Download as pdf or txt
Download as pdf or txt
You are on page 1of 233

Date: 30st June 2019 Bakliwal Tutorials-IIT

Comprehensive 2021 B-Test 1 YOUR STRATEGIC MOVE

Total Time: - 3.00 hrs Sub: - Physics, Chemistry & Math Max. Marks: -360

SET-A
Rules & regulations for BTEST

1. Immediately fill in the particulars on this page of the Test Booklet with Ball Point Pen provided in the examination
hall.

2. The Answer Sheet (OMR) is provided separately. Please fill in the particulars carefully with Ball Point Pen or
HB pencil only.

3. The test is of 3 hours duration.

4. The Test Booklet consists of 90 questions. The maximum marks are 360.

5. There are three parts in the question paper consisting of Physics, Chemistry and Mathematics having 30 questions
in each part of equal weightage. Each question is allotted 4 (four) marks for correct response.

6. Candidates will be awarded marks as stated above in instruction No. 5 for correct response of each question. ¼
(one-fourth) marks of the total marks allotted to the question (i.e. 1 mark) will be deducted for indicating incorrect
response of each question. No deduction from the total score will be made if no response is indicated for an item in
the answer sheet.

7. There is only one correct response for each question. Filling up more than one response in any question will be
treated as wrong response and marks for wrong response will be deducted accordingly as per instruction 6 above.

8. No candidate is allowed to carry any textual material, printed or written, bits of papers, pager, mobile phone, any
electronic device, etc. except the Admit Card inside the examination room/hall.

9. Rough work is to be done on the additional sheets provided

11. On completion of the test, the candidate must immediately hand over the Answer Sheet to the Invigilator on duty
in the Room/Hall. However, the candidates are allowed to take away this Test Booklet with them.

12. The CODE for this Booklet is A. Make sure that the CODE is marked properly on the answer sheet.

13. Do not fold or make any stray mark on the Answer Sheet.

Name of student

Roll No

Exam Hall
BT-IIT |Camp| FC Rd| PaudRoad|Wanowrie|Aundh|VimanNagar|Deccan|SataraRd|Pimple Saudagar|PCMC|www.bakliwaltutorialsiit.com 1
BT Tests Bakliwal Tutorials - IIT

BT-IIT |Camp| FC Rd| PaudRoad|Wanowrie|Aundh|VimanNagar|Deccan|SataraRd|Pimple Saudagar|PCMC|www.bakliwaltutorialsiit.com 2


BT Tests Bakliwal Tutorials - IIT
PHYSICS
SECTION - I
Single Correct Answer Type
This section contains 30 multiple choice questions. Each question has four choices (A), (B), (C) and (D) out of which
ONLY ONE is correct.

Q.1 The resultant of two vectors ⃗ and ⃗ is perpendicular to ⃗. Magnitude of resultant ⃗ is equal to magnitude of ⃗.
The angle between ⃗ and ⃗ is

(A) (B) (C) (D)

Solution: (C)

Q.2 Parallelogram law of vectors is applicable to the addition of :

(A) Any two vectors (B) Two scalars

(C) A vector and a scalar (D) Two vectors representing same physical quantity.

Solution: (D)

Q.3 Angle made by vector √3 + √2 ̂ − 2 with –ve y-axis is:

√ √ √
(A) cos (B) cos − (C) − cos − (D) None of these

Solution: (B)

√ ̂ √ ̂ ( ̂) √
cos = ( )
=


= cos

Q.4 The position vectors of four points A, B, C and D are ̂ + ̂ − 2 , 2 ̂ + 5 ̂ − 3 , ̂ − 6 ̂ − , − ̂ − 14 ̂ + then


select the correct relation:

(A) AB = CD (B) = (C) = (D) =

Solution: (B)

BT-IIT |Camp| FC Rd| PaudRoad|Wanowrie|Aundh|VimanNagar|Deccan|SataraRd|Pimple Saudagar|PCMC|www.bakliwaltutorialsiit.com 3


BT Tests Bakliwal Tutorials - IIT
⃗ = ̂+4 ̂− ⃗ = −2 ̂ − 8 ̂ + 2

⇒ = √1 + 16 + 1 = √18

= √4 + 64 + 4 = √72 = 2√18

Q.5 If ⃗, ⃗ & ⃗ + ⃗ are three non-zero vector. Such that ⃗ + ⃗ is perpendicular to ⃗ then which of one is correct:(A)
(A) ≥ (B) ≥ (C) > (D) >
√ √

Solution: (C)

Q.6 Vectors ⃗ = ̂ + ̂ − 2 and ⃗ = 3 ̂ + 3 ̂ − 6 are :

(A) Parallel (B) Antiparallel (C) Perpendicular (D) at acute angle with each
other

Solution: (A)

Since ⃗ = 3 ⃗, so both are parallel

Q.7 Component of 3 ̂ − 4 ̂ along ̂ − ̂ will be:

(A) (B) 7 (C) (D) 7√2


√ √

Solution: (C)

⃗. ⃗ ( ̂ ̂ ).( ̂ ̂)
Component = ⃗
= =
√ √

Q.8 For the figure shown

(A) ⃗ + ⃗ = ⃗ (B) ⃗ + ⃗ = ⃗ (C) ⃗ + ⃗ = ⃗ (D) ⃗ + ⃗ + ⃗ = 0

Solution: (C)

Apply triangle law of vector addition.

⃗ ⃗
If ⃗ and ⃗ are two non-zero vectors such that ⃗ + ⃗ = and ⃗ = 2 ⃗ then the angle between ⃗ and ⃗ is

(A) 37° (B) 53° (C) cos (−3/4) (D) cos (−4/3)

BT-IIT |Camp| FC Rd| PaudRoad|Wanowrie|Aundh|VimanNagar|Deccan|SataraRd|Pimple Saudagar|PCMC|www.bakliwaltutorialsiit.com 4


BT Tests Bakliwal Tutorials - IIT
Solution: (C)

( + +2 cos ) = ( + −2 cos )

⇒3 +3 + 10 cos =0 or 12 +3 + 10(2 ) ( ) cos =0

15 + 20 cos =0 cos =−

Q.10 If ̂ and ̂ are unit vectors along mutually perpendicular directions then the magnitude of ̂ − ̂ is :

(A) 0 (B) √2 (C) 1 (D) 2

Solution: (B)

| ̂ − ̂| − √1 + 1 + cos 90° = √2

Q.11 There are two vector ⃗ & ⃗. The x and y components of vector ⃗ are 4 m and 6m respectively. The x, y
components of vector ⃗ + ⃗ are 10 m and 9 m respectively. The magnitude of ⃗.

(A) √25 (B) √35 (C) √45 (D) √55

Solution: (C)

⃗ =4 ̂+6 ̂ ⃗ + ⃗ = 10 ̂ + 9 ̂

⇒ ⃗ = 6 ̂ + 3 ̂ ⃗ + ⃗ = √6 + 3 = √45

Q.12 Two vectors ⃗ and ⃗ inclined at an angle w.r.t. each other have a resultant ⃗ which makes an angle with ⃗. If
the direction of ⃗ and ⃗ are interchanged, then the resultant will have the same :

(A) Magnitude (B) Direction

(C) Magnitude as well as direction (D) Neither magnitude nor direction

Solution: (A)

In first case

=√ + +2 cos

In second case

=√ + +2 cos
BT-IIT |Camp| FC Rd| PaudRoad|Wanowrie|Aundh|VimanNagar|Deccan|SataraRd|Pimple Saudagar|PCMC|www.bakliwaltutorialsiit.com 5
BT Tests Bakliwal Tutorials - IIT
⇒ =

In first case ⃗ is at angle with horizontal and in second case ⃗ is at angle ( − ) with horizontal.

Q.13 If ⃗ = ̂ + ̂ and ⃗ = ̂ − ̂

The values of ⃗ + ⃗ . ⃗ − ⃗ is:

(A) √2 (B) 0 (C) (D) 2

Solution: (B)

⃗+ ⃗=2̂ ⇒ ⃗− ⃗=2 ̂

⇒ ⃗ + ⃗ . ⃗ − ⃗ = 4 ̂. ̂ = 0

Q.14 If ⃗ = 3 ̂ + 4 ̂ & ⃗ = 7 ̂ + 24 ̂ then the vector having the same magnitude as ⃗ and same direction as ⃗ is:

(A) 15 ̂ + 20 ̂ (B) 10 ̂ + 20 ̂ (C) 20 ̂ + 15 ̂ (D) 15 ̂ + 10 ̂

Solution: (A)
̂ ̂
= × √7 + 24 = 15 ̂ + 20 ̂

Q.15 Consider two vectors

⃗= ̂−5 ̂+3

⃗= ̂+ ̂+2

If ⃗ ⊥ ⃗ , find x.

(A) 2 or 3 (B) -2 or 3 (C) -2 or -3 (D) 2 or -3

Solution: (A)

⃗. ⃗ = 0

⇒ −5 +6= 0

⇒ ( − 2)( − 3) = 0

⇒ = 2 =3

Q.16 Vector sum of the forces of 5N and 4N cannot be:

(A) 10N (B) 4N (C) 3N (D) 5N

Solution: (A)
BT-IIT |Camp| FC Rd| PaudRoad|Wanowrie|Aundh|VimanNagar|Deccan|SataraRd|Pimple Saudagar|PCMC|www.bakliwaltutorialsiit.com 6
BT Tests Bakliwal Tutorials - IIT
Q.17 The resultant of two vectors of magnitudes 3A and A acting at angle is √13 . The value of angle ′ ′ is:

(A) 45° (B) 30° (C) 60° (D) 37°

Solution: (C)

(3 ) + ( ) + 2(3 )( ) cos = √13

∴ 10 +6 cos = 13

∴ = 60°

Q.18 If ⃗ = ̂ + ̂ − and ⃗ = ̂ − ̂ + , then unit vector along ⃗ − ⃗ is:

√ √ ̂ ̂
(A) ̂− (B) (C) (D)

Solution: (C)

⃗− ⃗= ̂+ ̂− − ̂− ̂+ =2 ̂−2

⃗ ⃗
∴ Unit vector along ⃗ − ⃗ = ⃗ ⃗

̂ ̂ ̂
= = = =
( ) ( ) √ √ √

⃗ ⃗
∴ ⃗ − ⃗ = ⃗ ⃗

̂
= = = =
( ) ( ) √ √ √

Q.19 A weight W is supported by two strings inclined at 60° and 30° to the vertical. The tensions in the strings are
& as shown. If these tensions are to be determined in terms of W using a triangle of forces, which of these
triangles should you draw? (block is in equilibrium)

(A) (B) (C) (D)

Solution: (D)

BT-IIT |Camp| FC Rd| PaudRoad|Wanowrie|Aundh|VimanNagar|Deccan|SataraRd|Pimple Saudagar|PCMC|www.bakliwaltutorialsiit.com 7


BT Tests Bakliwal Tutorials - IIT
⃗= ⃗, ⃗= ⃗ , ⃗= ⃗

⃗+ ⃗+ ⃗=0

Q.20 As shown in figure, mass M = 10 gms is placed on an inclined plane. In order to keep it at rest the value of m will
be-

(A) 5 gm (B) 10√3 gm (C) 0.10 gm (D) 1.0√3 gm

Solution: (A)

= sin 30°

= 5

Q.21 In which of the following cases the magnitude of acceleration of the block A will be maximum (neglect friction,
mass of pulley and string)

(A) (B)

BT-IIT |Camp| FC Rd| PaudRoad|Wanowrie|Aundh|VimanNagar|Deccan|SataraRd|Pimple Saudagar|PCMC|www.bakliwaltutorialsiit.com 8


BT Tests Bakliwal Tutorials - IIT

(C) (D)

Solution: (C)

(i) = = (ii) = =

(iii) = =2 (iv) =

Q.22 Two blocks A and B of masses 4 kg and 8 kg respectively are placed on a smooth plane surface. A force F of 12 N
is applied on A as shown. Find the force of contact (in N) between A and B?

(A) 8 N (B) 6 N (C) 10 N (D) None of these

Solution: (A)

= = = 1 /

⇒ 12 − = 4(1)

⇒ 12 − = 4(1)

=8

Q.23 The block of mass ‘m’ equal to 100 kg is being pulled by a horizontal force F = 2 mg applied to a string as shown
in figure (Take g = 10 / ). The pulley is massless and is fixed at the edge of an immovable table. What is the
value of force exerted by the supporting cable (rod) on the pulley (in Newton)

BT-IIT |Camp| FC Rd| PaudRoad|Wanowrie|Aundh|VimanNagar|Deccan|SataraRd|Pimple Saudagar|PCMC|www.bakliwaltutorialsiit.com 9


BT Tests Bakliwal Tutorials - IIT

(A) 3mg (B) 4 mg (C) 2√2 (D) 2mg

Solution: (C)

The F.B.D. of pulley is as shown.

Let ⃗ , ⃗ and ⃗ be the forces exerted by the horizontal string, vertical string and the support on the massless
pulley respectively. Then

⃗ + ⃗ + ⃗ =0

OR ⃗ = ⃗ + ⃗ = 2√2

(Tension in each string is ⃗ = ⃗ = 2 )

Q.24 F.B.D. of the rod of mass m as shown in figure (All surfaces are smooth)

(A) (B) (C) (D)

Solution: (A)

BT-IIT |Camp| FC Rd| PaudRoad|Wanowrie|Aundh|VimanNagar|Deccan|SataraRd|Pimple Saudagar|PCMC|www.bakliwaltutorialsiit.com 10


BT Tests Bakliwal Tutorials - IIT
Q.25 When a horse pulls a cart, the force that helps the horse to move forward is the force exerted by

(A) the card on the horse (B) the ground on the horse

(C) the ground on the cart (D) the horse on the ground

Solution: (B)

Q.26 Three weights are hanging over a smooth fixed pulley as shown in the figure. What is the tension in the string
connecting weights B and C?

(A) g (B) 4g/3 (C) 8g/9 (D) 10g/9

Solution: (B)

= = /3

− = /3
BT-IIT |Camp| FC Rd| PaudRoad|Wanowrie|Aundh|VimanNagar|Deccan|SataraRd|Pimple Saudagar|PCMC|www.bakliwaltutorialsiit.com 11
BT Tests Bakliwal Tutorials - IIT
⇒ = 4 /3

Q.27 A heavy particle of mass 1kg is suspended from a massless string attached to a roof . A horizontal force F is
applied to the particle such that in the equilibrium position the string makes an angle 30° with the vertical. The
magnitude of the tension T equals

(A) 10N (B) (C) 5N (D)


√ √

Solution: (B)

30 = 10


⇒ = 10

⇒ =

Q.28 A satellite is in orbit around the earth. Which one feels the greater force?

(A) The satellite because the earth is so much more massive

(B) The earth because the satellite has so little mass

(C) Earth and the satellite feel exactly the same force

(D) It depends on the distance of the satellite from Earth.

Solution: (C)

(Newtons third law)

Q.29 A vector makes an angle of 30° with the horizontal. If horizontal component of the vector is 250. Find its vertical
component?

BT-IIT |Camp| FC Rd| PaudRoad|Wanowrie|Aundh|VimanNagar|Deccan|SataraRd|Pimple Saudagar|PCMC|www.bakliwaltutorialsiit.com 12


BT Tests Bakliwal Tutorials - IIT

(A) 250 (B) 100/√3 (C) 250/√3 (D) None

Solution: (C)

Let vector is ⃗


= cos 30° = 250 =

⇒ =

= sin 30° = × =
√ √

Q.30 In the system shown in the adjoining figure, the tension is

(A) g (B) 2g (C) 5g (D) 6g

Solution: (C)

Balancing the forces we get T2 = 5g

BT-IIT |Camp| FC Rd| PaudRoad|Wanowrie|Aundh|VimanNagar|Deccan|SataraRd|Pimple Saudagar|PCMC|www.bakliwaltutorialsiit.com 13


BT Tests Bakliwal Tutorials - IIT
CHEMISTRY
SECTION - I
Single Correct Answer Type
This section contains 30 multiple choice questions. Each question has four choices (A), (B), (C) and (D) out of which
ONLY ONE is correct.

Q.1 8g of has amount of charge equal to : ( = 6.02 × 10 , = −1.6 × 10 )

(A) 5 (B) 2 (C) (D)

Solution: (C)

Q.2 The number of photons emitted in 16 hours by a 60 W sodium lamp ( of light = 620 nm) with 50% efficiency is :
Take hc = 12400 eVÅ

(A) 2.7 × 10 (B) 1.08 × 10 (C) 5.4 × 10 (D) 9 × 10

Solution: (C)

Q.3 Which the following order represents correct increasing order of mass for given samples.

(i) 1 g-atom of C (ii) mole of CH

(iii) 10 of water (density = 1 gm/ml) (iv) 3.011 × 10 atoms of oxygen

(A) ii < i < iii < iv (B) ii < iv < iii < i (C) iv < ii < iii < I (D) ii < i < iv < iii

Solution: (C)

BT-IIT |Camp| FC Rd| PaudRoad|Wanowrie|Aundh|VimanNagar|Deccan|SataraRd|Pimple Saudagar|PCMC|www.bakliwaltutorialsiit.com 14


BT Tests Bakliwal Tutorials - IIT

Q.4 An atom of an element has 10 electrons, 10 protons and 12 neutrons. The atomic mass of the element is-

(A) 32 u (B) 22 u (C) 20 u (D) None of these

Solution: (B)

Atomic mass of the element is 22 amu.

Q.5 The addition of a neutron to the nucleus of an atom results in-

(A) Increase in nuclear charge (B) Increase in atomic mass

(C) Formation of an isobar (D) None of above

Solution: (B)

FACT

Q.6 Which one of the following samples doesn’t contain 3 atoms:

(A) 22.4 L of CO at STP (B) 92 g of NO

(C) 0.375 moles of S (D) 0.5 N molecules of Ethene (C H )

Solution: (B)

Q.7 Three oxides of a certain hypothetical element contain 50%, 33.33% and 25% element by mass respectively. The
simplest ratio of the valencies of element in the three oxides is:

(A) 2 : 3 : 4 (B) 3 : 4 : 6 (C) 1 : 2 : 3 (D) None of these

Solution: (C)

BT-IIT |Camp| FC Rd| PaudRoad|Wanowrie|Aundh|VimanNagar|Deccan|SataraRd|Pimple Saudagar|PCMC|www.bakliwaltutorialsiit.com 15


BT Tests Bakliwal Tutorials - IIT

Q.8 When dinitrogen pentaoxide , a white solid, is heated, it decomposes into nitrogen dioxide and oxygen
according to the reaction:

N O (s) → NO (g) + O (g)

If a sample of produces 3.2 g of , then the mass of formed alongwith is:

(A) 4.6 g (B) 9.2 g (C) 13.8 g (D) 18.4 g

Solution: (D)

Q.9 A mixture of 32 g of oxygen and 6 g of hydrogen is heated to the formation of H O(g). The reaction is :

2H (g) + O (g) → 2H O(g)

What will be the maximum amount of produced by given reaction:

(A) 38 g (B) 54 g (C) 36 g (D) 18 g

Solution: (C)

BT-IIT |Camp| FC Rd| PaudRoad|Wanowrie|Aundh|VimanNagar|Deccan|SataraRd|Pimple Saudagar|PCMC|www.bakliwaltutorialsiit.com 16


BT Tests Bakliwal Tutorials - IIT
Q.10 If the vapour density of a volatile metal chloride is 130.5 and the valency of the metal involved is 4, then the gram
atomic mass of metal is:

(A) 66.75 g (B) ≈ 2 × 10 (C) 119 g (D) 59.5 g

Solution: (C)

Q.11 How many moles of will be needed to add to 810 gram of water to prepare a solution having 0.1 mole
fraction of .

(A) 1 (B) 5 (C) 4 (D) 9

Solution: (B)

Q.12 Volume of water required to convert 100 ml aqueous solution of 0.5M HCl to 0.2M HCl solution:

(A) 250 ml (B) 100 ml (C) 150 ml (D) 400 ml

Solution: (C)

Q.13 At STP, what volume of ( ) is needed to react completely with 9.03 × 10 molecules of CO(g) according to
given reaction ( = 6.02 × 10 )

CO(g) + 2H (g) → CH OH(g)

(A) 22.4 Litre (B) 44.8 Litre (C) 4.48 Litre (D) 6.72 Litre

BT-IIT |Camp| FC Rd| PaudRoad|Wanowrie|Aundh|VimanNagar|Deccan|SataraRd|Pimple Saudagar|PCMC|www.bakliwaltutorialsiit.com 17


BT Tests Bakliwal Tutorials - IIT
Solution: (D)

Q.14 A mixture of and contains 88% of by weight, what is the mole fraction of in the mixture:

(A) 0.12 (B) 0.25 (C) 0.75 (D) 0.18

Solution: (C)

Q.15 A gaseous mixture contains ( ) and ( ) in 2 : 5 ratio by mass. The ratio of the number of molecules of
( ) and ( ) is :

(A) 5 : 2 (B) 2 : 5 (C) 1 : 2 (D) 5 : 4

Solution: (B)

Q.16 Density of ozone relative to methane under the same temperature & pressure conditions is:

(A) 1/3 (B) 3 (C) 1.5 (D) 2.5

Solution: (B)

BT-IIT |Camp| FC Rd| PaudRoad|Wanowrie|Aundh|VimanNagar|Deccan|SataraRd|Pimple Saudagar|PCMC|www.bakliwaltutorialsiit.com 18


BT Tests Bakliwal Tutorials - IIT

Q.17 If the diameter of two different nuclei are in the ratio 1 : 3 then their atomic mass number are in the ratio:

(A) 1 : 9 (B) 27 : 1 (C) 1 : 8 (D) 1 : 27

Solution: (D)

Q.18 An -particle with initial speed is projected from infinity and it approaches up to distance from a nuclie.
Then, the initial speed of -particle, which approaches upto 2 distance from the nucleus, is :

(A) √2 (B) (C) 2 (D)


Solution: (B)

Q.19 For a wave, frequency is 10 Hz and wavelength is 2.5 m. How much linear distance will it travel in 40 seconds:

(A) 1000 m (B) 100 m (C) 10,000 m (D) 10 m

Solution: (A)

Q.20 The potential energy of the electron present in the ground state of ion is represented by:

(r = Radius of ground state)

3e 2 3e 3e 2 3e2
(A)  (B)  (C)  (D) 
40 r 40 r 40 r 2 40 r

Solution: (D)

KZe2
PE  
r

BT-IIT |Camp| FC Rd| PaudRoad|Wanowrie|Aundh|VimanNagar|Deccan|SataraRd|Pimple Saudagar|PCMC|www.bakliwaltutorialsiit.com 19


BT Tests Bakliwal Tutorials - IIT
Q.21 One quantum is absorbed per gaseous molecule of for converting into Br atoms. If light absorbed has
wavelength 5000Å, then the bond energy of is about …. / (1 / particle = 96 KJ/mol)

(A) 119 (B) 238 (C) 357 (D) 476

Solution: (B)

Q.22 Visible spectrum contains light of following colours “Violet – Indigo – Blue – Green – Yellow – Orange – Red”
(VIBGYOR)

Its frequency ranges from Violet (7.5 × 10 ) to Red (4 × 10 ). Find out the maximum wavelength in this
range.

(A) 400 Å (B) 750 Å (C) 4000 Å (D) 7500 Å

Solution: (D)

Q.23 A certain dye absorbs light of certain wavelength and then fluorescence light of wavelength 5000 Å. Assuming
that under given conditions, 50% of the absorbed energy is re-emitted out as fluorescence and the ratio of number
of quanta emitted out to the number of quanta absorbed is 5 : 8,

Find out the wavelength of absorbed light (in Å) : hc = 12400 eVÅ

(A) 4000 Å (B) 3000 Å (C) 2000 Å (D) 1000 Å

Solution: (A)

Q.24 Assume that 10 of light energy is needed by the interior of the human eye to see an object. How many
minimum photons of green light ( = 310 ) are needed to generate this energy?

(A) 14 (B) 13 (C) 16 (D) 17

BT-IIT |Camp| FC Rd| PaudRoad|Wanowrie|Aundh|VimanNagar|Deccan|SataraRd|Pimple Saudagar|PCMC|www.bakliwaltutorialsiit.com 20


BT Tests Bakliwal Tutorials - IIT
Solution: (C)

Q.25 If the radius of the first Bohr orbit of the H atom is r, then for , it will be

(A) 3r (B) 9r (C) r/3 (D) r/9

Solution: (C)

Q.26 For which of the following species, Bohr model is not valid:

(A) (B) (C) (D)

Solution: (D)

Only for single electron species.

Q.27 The ratio of radius of two different orbits in a H-atom is 4 : 9. Then, the ratio of the frequency of revolution of
electron in these orbits is:

(A) 2 : 3 (B) 27 : 8 (C) 3 : 2 (D) 8 : 27

Solution: (B)

Q.28 How would you prepare exactly 3 litre of 1M NaOH solution by mixing properties of stock solutions of 2.5 M
NaOH and 0.4 M NaOH, if no water is to be used. Find the ratio of the volume  v1 v 2 

(A) 1 : 3 (B) 3 : 7 (C) 2 : 5 (D) Data insufficient

Solution: (C)

BT-IIT |Camp| FC Rd| PaudRoad|Wanowrie|Aundh|VimanNagar|Deccan|SataraRd|Pimple Saudagar|PCMC|www.bakliwaltutorialsiit.com 21


BT Tests Bakliwal Tutorials - IIT

Q.29 75 mL of solution (specific gravity = 1.18), containing 49% by mass, is diluted to 590 mL.
Calculate molarity of the diluted solution

(A) 0.375 M (B) 1.5 M (C) 0.75 M (D) 5.9 M

Solution: (C)

Q.30 What volume of 0.25 (nitric acid) solution reacts with 50 Of 0.15 (sodium carbonate)
solution in the following reaction:

2HNO (aq) + Na CO (aq) → 2NaNO (aq) + H O(l) + CO (g)

(A) 30 (B) 60 (C) 120 (D) 100

Solution: (B)

MATHS
SECTION - I
Single Correct Answer Type
This section contains 30 multiple choice questions. Each question has four choices (A), (B), (C) and (D) out of which
ONLY ONE is correct.


Q.1 If = + √2, then a & b are respectively equal to

(A) , (B) , (C) , (D) ,

Solution: (A)
√ √ √
× =
√ √

BT-IIT |Camp| FC Rd| PaudRoad|Wanowrie|Aundh|VimanNagar|Deccan|SataraRd|Pimple Saudagar|PCMC|www.bakliwaltutorialsiit.com 22


BT Tests Bakliwal Tutorials - IIT
Q.2 The expression √ √ is simplified to

(A) (B) (C) (D)

Solution: (D)

√ =( ) = = √

Q.3 If ( )
= 1, then m =

(A) 0 (B) 1 (C) n (D) 2n

Solution: (D)

(2 ) (2 )2 = (2 ) 2

2 .2 =2 .2

Equate powers + +3 = + +2

We get m = 2n

Q.4 If − = 2, then the value of + is

(A) 4 (B) 8 (C) 12 (D) 34

Solution: (D)

− = 2 ⇒ + −2= 4

+ =6

Again acquiring , + + 2 = 36

Q.5 If + + = 0, the value of + + is ( ≠ 0)

(A) 1 (B) 3 (C) -3 (D) None of these

Solution: (B)

+ + = = =3

BT-IIT |Camp| FC Rd| PaudRoad|Wanowrie|Aundh|VimanNagar|Deccan|SataraRd|Pimple Saudagar|PCMC|www.bakliwaltutorialsiit.com 23


BT Tests Bakliwal Tutorials - IIT
(∴ + + = 0 ; + + =3 )

Q.6 If = 7 + 4√3 , then the value of √ + is


(A) 8 (B) 6 (C) 5 (D) 4

Solution: (D)

√ = 7 + 4√3 = 2 + √3 = 2 + √3

√ + √ = 2 + √3 + √
= 2 + √3 + 2 − √3 = 4

Q.7 If 2 −5 + + 2 = ( − 2) ( − − 1), for all values of , then a & b are respectively:

(A) 2, 1 (B) 2, -1 (C) 1, 2 (D) -1, ½

Solution: (A)

− −1 =

( )
− −1 = =2 − −1

= 2 ; = 1

Q.8 If = { : = 3 + 1, 2 ≤ ≤ 5, n ∈ N}, then number of subsets of A is:

(A) 4 (B) 16 (C) 8 (D) None of these

Solution: (B)

= {7, 10, 13, 16} ; No of subsets = 2 = 16

Q.9 If A is the set of the divisors of the number 15, B is the set of prime numbers smaller than 10 and C is the set of
even numbers smaller than 9, then ( ∪ ) ∩ is the following set

(A) {1, 3, 5} (B) {1, 2, 3} (C) {2, 3, 5} (D) {1, 2, 4}

Solution: (C)

= {1, 3, 5, 15} ; = {2, 3, 5, 7} , = {2, 4, 6, 8}

( ∪ )∩ = {2, 3, 5}

BT-IIT |Camp| FC Rd| PaudRoad|Wanowrie|Aundh|VimanNagar|Deccan|SataraRd|Pimple Saudagar|PCMC|www.bakliwaltutorialsiit.com 24


BT Tests Bakliwal Tutorials - IIT
Q.10 Out of 40 children, 30 can swim, 27 can play chess and only 5 can do neither. How many can swim only?

(A) 30 (B) 22 (C) 12 (D) 8

Solution: (D)

(∪) = 40

( ∪ ) = 35

(30 − ) + + (27 − ) = 35

= 22

n (Children who can swim only) = 30 − =8

Q.11 The value of x satisfying the equation =

(A) ab/c (B) 2ab/c (C) ab/3c (D) ab/2c

Solution: (A)
( ) ( ) ( )
( )
=
( )

Apply C & D, =

( ) ( )
Apply C & D =
( ) ( )

Apply C & D = =

Q.12 If 4 − ( − 1) + − 6 + 1 is divisible by (2 − 1), then ‘a’ is equal to

(A) 13 (B) -13 (C) 11 (D) -11

Solution: (A)

BT-IIT |Camp| FC Rd| PaudRoad|Wanowrie|Aundh|VimanNagar|Deccan|SataraRd|Pimple Saudagar|PCMC|www.bakliwaltutorialsiit.com 25


BT Tests Bakliwal Tutorials - IIT
= 0 ⇒ 4 − ( − 1) + −3+1= 0

Gives a = 13

Q.13 Number of composite numbers less than 20 which are coprime with 4 is/are

(A) 2 (B) 3 (C) 4 (D) 5

Solution: (A)

9, 15

Q.14 In a survey of 60 people, it was found that 25 people read Newspapers H, 26 read Newspapers T, 26 Newspaper I,
9 read both H and I, 11 read both H and T, 8 read both T and I, 3 read all the three newspapers. Find the number of
people who read at least one of the newspapers?

(A) 52 (B) 42 (C) 62 (D) None of these

Solution: (A)

( ∪ ∪ )= ( )+ ( )+ ( )− ( ∩ )− ( ∩ )− ( ∩ )+ ( ∩ ∩ )

= 25 + 26 + 26 − 11 − 8 − 9 + 3 = 52

Q.15 For a sequence < >, = 2 and = . Then ∑ is

(A) [4 + 19 × 3] (B) 3 1 − (C) (1 − 3 ) (D) None of these

Solution: (B)

Sequence is 2, ,

∑ =2+ + … … 20

=2 =3 1−

Q.16 The largest term common to the sequences 1, 11, 21, 31, ……… to 100 terms and 31, 36, 41, 46, …….to 100 term
is

(A) 381 (B) 471 (C) 281 (D) None of these

Solution: (D)

BT-IIT |Camp| FC Rd| PaudRoad|Wanowrie|Aundh|VimanNagar|Deccan|SataraRd|Pimple Saudagar|PCMC|www.bakliwaltutorialsiit.com 26


BT Tests Bakliwal Tutorials - IIT
Two given sequence

1, 11, 21, ..991 (last term = 1 + (100 - 1) 10 = 991)

31, 36, 41, ……526 (last term = 31 + (100 - 1) 5 = 521

Largest term common = 521

Q.17 Sum of infinite number of terms of a G.P. is 20 and sum of their squares is 100. The common ratio of the G.P. is

(A) 5 (B) 3/5 (C) 8/5 (D) 1/5

Solution: (B)

Sum of term = 20 ⇒ = 20

Sum of squares = +( ) +( ) …..= = 100

Solving 2 equations we get =

Q.18 In the sequence 1, 2, 2,4, 4, 4, 4, 8, 8, 8, 8, 8, 8, 8, 8…….., where n consecutive terms have the value n, the 1025th
term is

(A) 2 (B) 2 (C) 2 (D) 2

Solution: (B)

Let term at 1025th position = 2

So 1 + 2 + 2 + ⋯ 2 ≥ 1025

1 ≥ 1025 ; ≥ 10

Q.19 for non zero x the value of on simplifying is

(A) x (B) (C) 1 (D) -1

Solution: (C)

( )

= =1

Q.20 The sum of the n terms of two A.P series are in the ratio of (n + 1) : (n + 3). The ratio of their 4th terms is
BT-IIT |Camp| FC Rd| PaudRoad|Wanowrie|Aundh|VimanNagar|Deccan|SataraRd|Pimple Saudagar|PCMC|www.bakliwaltutorialsiit.com 27
BT Tests Bakliwal Tutorials - IIT
(A) (B) 4/5 (C) 1/2 (D) 1/3

Solution: (B)

Let Sn and Sn be sum of n terms of two series then

Sn

n 1

n  1  2  1  2 a1  n  1d1

Sn n3 n  1  2  2 2 a 2  n  1d 2

 a1 = 1, d1 = 1, a2 = 2, d2 = 1  T4  1  3  1  4 .
T4 2  3 1 5

Hence (B) is the correct answer.

24
Q.21 {bi } , i = 1, 2, ….n is arithmetic sequence. If b1 b5 b10 b15 b20 b24  225 then  b i is equal to
i 1

(A) 600 (B) 900 (C) 300 (D) None of these

Solution: (B)

b1  b5  b10  b15  b20  b24

= 6 b1 + (4d + 9d + 14d + 19d + 23d)

= 3 (2 b1 + 23d)

= 225 (given)
24
24
Now,  b 1  (2 b1 + 23d)
i 1 2

= 12  75 = 900

Q.22 Given p A.P's, each of which consists of n terms. If their first terms are 1, 2, 3, …, p and common differences are
1, 3, 5, ---, 2p –1 respectively, then sum of the terms of all the progressions is
1 1
(A) np(np+1) (B) n(p+1) (C) np(n+1) (D) none of these .
2 2

Solution: (A)

n
The rth A. P. has first term r and common difference 2r – 1. Hence sum of its n terms = 2r  n  12r  1 .
2
p
n
The required sum =  2 2r  n  12r  1
r 1

BT-IIT |Camp| FC Rd| PaudRoad|Wanowrie|Aundh|VimanNagar|Deccan|SataraRd|Pimple Saudagar|PCMC|www.bakliwaltutorialsiit.com 28


BT Tests Bakliwal Tutorials - IIT
p p
npp  1 nn  1
p
= n r  n n  1  2 r   1 = pp  1  p = np np  1 .
 

r 1 2  r  1 
r 1  2 2 2

Hence (A) is the correct answer.

Q.23 The third term of a G.P. is 4, the product of the first five terms is

(A) 43 (B) 45 (C) 44 (D) none of these

Solution: (B)

Let the first five terms of the given G.P. be a1 , a2 , a3 , a4, a5 .

Hence a3 = 4 . Now a1 a5 = a2 a4 = a32  a1 a2 a3 a4 a5 = 45.

Hence (B) is the correct answer.

ba bc
Q.24 If a, b, c are in H.P. , then the value of  is
ba b c

(A) 0 (B) 1 (C) 2 (D) 3

Solution: (C)

2ac b 2c b  a 3c  a
a, b, c are in H.P.  b =     . . . . (A)
ac a ac ba c a

Again a, b, c are in H.P.

2ac b 2a b  c 3a  c
 b=     . . . . (B)
ac c ac bc ac

b  a b  c 3c  a 3a  c
From (A) and (B)    =2.
ba bc c a ac

Hence (C) is the correct answer.

Q.25 150 workers were engaged to finish a piece of work in a certain number of days. 4 workers dropped the second
day, 4 more workers dropped the third day and so on. It takes eight more days to finish the work now. The number
of days in which the work was completed is

(A) 15 (B) 20 (C) 25 (D) 30

Solution: (C)

BT-IIT |Camp| FC Rd| PaudRoad|Wanowrie|Aundh|VimanNagar|Deccan|SataraRd|Pimple Saudagar|PCMC|www.bakliwaltutorialsiit.com 29


BT Tests Bakliwal Tutorials - IIT
Let the work was to be finished in x days.  Work of 1 worker in a day 1 Now the work will be finished in (x +

150 x

8) days.  Work done = Sum of the fraction of work done


1 1 1 to (x + 8) terms
1  150  (150  4 )  (150  8 )  .......
150 x 150 x 150 x

x 8  150  4 
 1 2   ( x  8  1)    150 x  (x  8){150  2(x  7)}  (x  8)(x  7)  600  0
2  150 x  150 x 

 (x  8)(x  7)  25  24 ,  x  8  25
Hence work completed in 25 days.

100 100
Q.26 Let a n be the nth term of the G.P. of positive numbers. Let a 2n  and a 2 n 1   , such that   , then the
n 1 n 1

common ratio is

   
(A) (B) (C) (D)
   

Solution: (A)
100
Let x be the first term and y, the common ratio of the G.P. Then,   a 2n  a 2  a 4  a 6  ....  a 200 and
n 1

100
  a
n 1
2 n 1  a1  a 3  a 5  ......  a199

1  (y 2 )100  1  y 200 
   xy  xy 3  xy 5  .....  xy 199  xy 2
 xy  2


1y  1y 

1  (y 2 )100  1  y 200 
  x  xy 2  xy 4  .....  xy 198  x  2
 x   2


1y  1y 
 
  y . Thus, common ratio 
 

Q.27 If a, b, c are in A.P. b – a, c – b and a are in G.P., then a : b : c is

(A) 1 : 2 : 3 (B) 1 : 3 : 5 (C) 2 : 3 : 4 (D) 1 : 2 : 4

Solution: (A)

Given, a, b, c are in A.P.  2b = a + c


b – a, c – b, a are in G.P. So (c  b )2  a (b  a )
  2b  a  c 
 
 2
(b  a )  (b  a ) a   b  b  a  c
  b  a  c  b 

 b  2a [∵ b  a]
Put in 2b = a + c, we get c = 3a. Therefore a : b : c = 1 : 2 : 3

BT-IIT |Camp| FC Rd| PaudRoad|Wanowrie|Aundh|VimanNagar|Deccan|SataraRd|Pimple Saudagar|PCMC|www.bakliwaltutorialsiit.com 30


BT Tests Bakliwal Tutorials - IIT
Q.28 If the first two terms of an H.P. be 2 and 12 then the largest positive term of the progression is the
5 23

(A) 6th term (B) 7th term (C) 5th term (D) 8th term

Solution: (C)

For the corresponding A.P., the first two terms are 5 and 23 i.e. 30 and 23
2 12 12 12

Common difference 7

12

 The A.P. will be 30 23 16 9 2 5


, , , , , , ......
12 12 12 12 12 12

The smallest positive term is 2 , which is the 5th term.  The largest positive term of the H.P. will be the
12
5th term.

Q.29 3 5 7 is equal to
1   3  ...... 
2 22 2

(A) 3 (B) 6 (C) 9 (D) 12

Solution: (B)

3 5 7
S 1    ....... 
2 22 23
1 1 3 5
S     ....... 
2 2 22 23
1 2 2 2
S  1   2  3  ........ (on subtractin g)
2 2 2 2

S 1 1 1  S  1/2 
  1  2   2  3  ....     1  2 3. Hence S = 6
2  2 2 2  2  1 1 / 2 

Q.30 Sum of the series 1  2.2  3.2 2  4.23  .....  100.299 is

(A) 100 .2100  1 (B) 99.2100  1 (C) 99.2100  1 (D) 100 .2100  1

Solution: (B)

Let S  1  2.2  3.2 2  4.23  ....  100.299 …..(i)


2S = 1.2  2.2 2  3.23  .....  99.299  100.2100 …..(ii)
Equation (i) – Equation (ii) gives,
2(299  1)
 S  1  (1.2  1.2 2  1.23  ..... upto 99 terms)  100 .2100  1   100 .2100
2 1
 S  1  2100  2  100.2100  1  99.2100

BT-IIT |Camp| FC Rd| PaudRoad|Wanowrie|Aundh|VimanNagar|Deccan|SataraRd|Pimple Saudagar|PCMC|www.bakliwaltutorialsiit.com 31


Date: 25th August 2019 Bakliwal Tutorials-IIT
Comprehensive 2021 B-Test 3 YOUR STRATEGIC MOVE

Total Time: - 3 hrs. Sub: - Physics, Chemistry & Math Max. Marks: -360

SET-A
Rules & regulations for BTEST

1. Immediately fill in the particulars on this page of the Test Booklet with Ball Point Pen.

2. The Answer Sheet (OMR) is provided separately. Please fill in the particulars carefully with Ball Point Pen.

3. The test is of 3 hours duration.

4. The Test Booklet consists of 90 questions. The maximum marks are 360.

5. There are three parts in the question paper consisting of Physics, Chemistry and Mathematics having 30 questions
in each part of equal weightage. Each question is allotted 4 (four) marks for correct response.

6. Candidates will be awarded marks as stated above in instruction No. 5 for correct response of each question. ¼
(one-fourth) marks of the total marks allotted to the question (i.e. 1 mark) will be deducted for indicating incorrect
response of each question. No deduction from the total score will be made if no response is indicated for an item in
the answer sheet.

7. There is only one correct response for each question. Filling up more than one response in any question will be
treated as wrong response and marks for wrong response will be deducted accordingly as per instruction 6 above.

8. No candidate is allowed to carry any textual material, printed or written, bits of papers, pager, mobile phone, any
electronic device, etc. except the Admit Card inside the examination room/hall.

9. Rough work is to be done on the additional sheets provided

11. On completion of the test, the candidate must immediately hand over the Answer Sheet to the Invigilator on duty
in the Room/Hall. However, the candidates are allowed to take away this Test Booklet with them.

12. The CODE for this Booklet is A. Make sure that the CODE is marked properly on the answer sheet.

13. Do not fold or make any stray mark on the Answer Sheet.

Name of student

Roll No

Exam Hall

BT-IIT |Camp| FC Rd| PaudRoad|Wanowrie|Aundh|VimanNagar|Deccan|SataraRd|Pimple Saudagar|PCMC|www.bakliwaltutorialsiit.com 1


BT Tests Bakliwal Tutorials - IIT
PHYSICS
SECTION - I
Single Correct Answer Type
This section contains 30 multiple choice questions. Each question has four choices (A), (B), (C) and (D) out of
which ONLY ONE is correct.

Q.1 A rescue plane is flying horizontally at a speed of 30 m/s and at an altitude of 125 m when it drops a
warning flare. Neglecting air resistance and assuming that the plane does not change its course, speed,
altitude, how far from the plane is the flare when it hits the ground?

(A) 146 m (B) 195 m (C) 125 m (D) 150 m

Solution: (C)

Since horizontal velocity of both are same so plane will always remain vertically above packet. Hence it
will be 125 m.

Q.2 A particle starts from the origin at t = 0 and moves in the x-y plane with constant acceleration a in the y-
direction. Acceleration of particle in x direction is zero. Its equation of motion is given by y  bx 2 . The
x-component of its velocity is

 2a  a a
(A) Variable (B)   (C) (D)  
 b  2b b

Solution: (C)

y  bx 2
dy dx
 2bx
dt dt

On differentiating again, we get

BT-IIT |Camp| FC Rd| PaudRoad|Wanowrie|Aundh|VimanNagar|Deccan|SataraRd|Pimple Saudagar|PCMC|www.bakliwaltutorialsiit.com 2


BT Tests Bakliwal Tutorials - IIT
2 2
d y  d x  dx  dx  
2
 2b  x 2    
dt  dt  dt  dt  
d2y   dx 2  d 2 x
 2b 0     as 2  0
dt 2   dt   dt
2
 dx 
a  2b  
 dt 
dx a
v 
dt 2b

PARAGRAPH FOR QUESTION NO. 3 & 4

An inclined plane makes an angle   45 with horizontal. A stone is projected normally from the inclined
plane, with speed u m/s at time t = 0 the x and y axis are drawn from the point of projection along and normal to
inclined plane as shown in the Figure. The length of incline is sufficient for stone to land on it and neglect air
friction.

Q.3 The instant of time at which velocity of stone is parallel to x-axis

2 2u 2u 2u u
(A) (B) (C) (D)
g g g 2g

Solution: (C)

vy  0
 u y  dyt  0
 u  g cos  t  0,
u u 2
t 
g cos  g

Q.4 The instant of time at which velocity of stone makes an angle   45 with positive x-axis is

2 2u 2u 2u u
(A) (B) (C) (D)
g g g 2g

BT-IIT |Camp| FC Rd| PaudRoad|Wanowrie|Aundh|VimanNagar|Deccan|SataraRd|Pimple Saudagar|PCMC|www.bakliwaltutorialsiit.com 3


BT Tests Bakliwal Tutorials - IIT
Solution: (D)

v y  vx
 u  g cos  t  g sin 6t
u u
t  
g(cos   sin ) g 2

Q.5 A particle is projected from a point O with a velocity u in a direction making an angle  with the
horizontal. At point P particle is moving at right angles to its initial direction. Its velocity at point P is

(A) u tan  (B) u cot  (C) u cos ec  (D) u sec 

Solution: (B)

v cos (90   )  u cos 


v  u cot 

Q.6 A ball is projected from a point at two different angles with same speed u and land at the same point in
both the cases. Then

(A) the difference between two angles of projection is 90

(B) The maximum height attained by the ball in both the cases is equal

(C) the sum of maximum heights for the two cases is u 2 /  2g 

(D) the maximum height attained by the ball in one case must be twice of the maximum height attained
by the ball in second case

Solution: (C)

(a) sum of angle is 90° not difference

(b) Max. height ∝ sin

BT-IIT |Camp| FC Rd| PaudRoad|Wanowrie|Aundh|VimanNagar|Deccan|SataraRd|Pimple Saudagar|PCMC|www.bakliwaltutorialsiit.com 4


BT Tests Bakliwal Tutorials - IIT
2 2
u sin 
(c) H1 
2g

 
u 2 sin 2     2 2
H2  2   u cos 
2g 2g
 H1  H 2  u 2 / (2g)

Q.7 At time t = 0 a particle leaves the origin with a velocity of 6 m/s in the positive y direction. Its
acceleration is given by ⃗ = 2 ̂ − 3 ̂ / . The x and y coordinates of the particle at the instant the
particle reaches maximum y coordinate are

(A) 2 m, 3m (B) 4 m, 6 m (C) 1 m, 3 m (D) 2 m, 6 m

Solution: (B)

Q.8 Jai is standing on the top of a building of height 25 m, he wants to throw his gun to Veeru who stands on
top of another building of height 20 m at a distances 15 m from the first building. For which horizontal
speed of projection, it is possible

(A) 5ms 1 (B) 10 ms 1 (C) 15ms 1 (D) 20 ms 1

Solution: (C)

BT-IIT |Camp| FC Rd| PaudRoad|Wanowrie|Aundh|VimanNagar|Deccan|SataraRd|Pimple Saudagar|PCMC|www.bakliwaltutorialsiit.com 5


BT Tests Bakliwal Tutorials - IIT

Q.9 In a rescue attempt, a stationary helicopter in air drops a life preserver to a drowning man (Velocity of
man with respect to river is zero) being swept downstream by a river current of constant velocity v. The
helicopter is at a height of 45 m. The man is 6.0 m upstream from a point directly under the helicopter
when the life preserver is released. It lands 3.0 m in front of the man. How fast is the current flowing?
Neglect air resistance.  g  10 m / s 2 

(A) 1m / s (B) 2m / s (C) 3m / s (D) 4m / s

Solution: (A)

2h
t  3s
g
x  vt
3  v3
 v  1m / s

Q.10 Two guns on a battleship simultaneously fires two shells with same speed at enemy ships. If the shells
follow the parabolic trajectories as shown, which ship will get hit first?

BT-IIT |Camp| FC Rd| PaudRoad|Wanowrie|Aundh|VimanNagar|Deccan|SataraRd|Pimple Saudagar|PCMC|www.bakliwaltutorialsiit.com 6


BT Tests Bakliwal Tutorials - IIT
(A) A (B) B (C) Both at same time (D) Need more information

Solution: (B)

Time of flight depends upon maximum height and maximum height for A is large in comparison to B.

Q.11 A student is standing on a train travelling along a straight horizontal track at a speed of 10 m/s. The
student throws a ball into the air along a path, that he sees to make an initial angle of 60 with the
horizontal along the track. The professor standing on the ground observes the ball to rise vertically, the
maximum height reached by the ball with respect to professor is

(A) 15 / 4m (B) 15 3 (C) 5 3 (D) 15 m

Solution: (D)

Vbt cos 60  10


Vbt sin 60  Vbg
Vbg  10 3
Vbg2 100  3
H   15 m
2g 20

Q.12 A swimmer swims in still water at a speed = 5 km/h. He enters a 200-m-wide river, having river flow
speed = 4 km/h, at a point A and proceeds to swim at an angle of 127  sin 37  0.6  with the river flow
direction. Another point B is located directly across A on the other side. The swimmer lands on the other
bank at a point C, from which he walks the distance CB with a speed = 3 km/h. The total time in which
he reaches from A to B is

(A) 5 min (B) 4 min (C) 3 min (D) None of these

Solution: (B)

BT-IIT |Camp| FC Rd| PaudRoad|Wanowrie|Aundh|VimanNagar|Deccan|SataraRd|Pimple Saudagar|PCMC|www.bakliwaltutorialsiit.com 7


BT Tests Bakliwal Tutorials - IIT
Given: Vbr  5km / h

v r  4 km / h
d  0.2 km
0.2
t
4
 0.05 h  3 min

BC  1km / h  0.05 h
0.05
t2   60
3
 1min
t1  t 2  4 min

Q.13 A boat having a speed of 5 km/h in still water crosses a river of width 1 km along the shortest possible
path in 15 min. The speed of the river in km/h is.

(A) 1 (B) 3 (C) 4 (D) 41

Solution: (B)

Vbr  5km / h
vr
sin  
5
d 4
t  cos  
v br cos  5
   37
vr
sin 37 
5
 v r  3km / h

Q.14 A river flows with a speed more than the maximum speed with which a person can swim in still water.
He intends to cross the river by the shortest possible path (i.e., he wants to reach the point on the opposite
bank which is directly opposite to the starting point). Which of the following is correct?
BT-IIT |Camp| FC Rd| PaudRoad|Wanowrie|Aundh|VimanNagar|Deccan|SataraRd|Pimple Saudagar|PCMC|www.bakliwaltutorialsiit.com 8
BT Tests Bakliwal Tutorials - IIT
(A) He should start normal to the river bank

(B) He should start in such a way that he moves normal to the bank, relative to the bank

(C) He should start in a particular (calculated) direction making an obtuse angle with the direction of
water current

(D) The man cannot cross the river in the desired way

Solution: (D)

We know that to cross the river by the shortest path,

u
sin  
v

But u  v  sin   1 , which is not possible

PARAGRAPH FOR QUESTION NO. 15 & 16

A car is moving towards south with a speed of 20 ms 1 . A motorcyclist is moving towards east with a speed
of 15ms 1 . At a certain instant, the motorcyclist is due south of the car and is at a distance of 50 m from the car.

Q.15 The shortest distance between the motorcyclist and the car is

(A) 10 m (B) 20 m (C) 30 m (D) 40 m

Solution: (C)

Taking N as + y-axis and E as + x Axis

Imagine yourself as an observer sitting inside the car. You will regard the car as being at rest (at C).
Relative to you, the speed of the motorcyclist is obtained by imposing the reversed velocity of the car on
motorcyclist as shown in Figure

vm  15ms1 , vc  20ms 1

BT-IIT |Camp| FC Rd| PaudRoad|Wanowrie|Aundh|VimanNagar|Deccan|SataraRd|Pimple Saudagar|PCMC|www.bakliwaltutorialsiit.com 9


BT Tests Bakliwal Tutorials - IIT

v mC  152  202  25 ms 1

Q.16 The time after which they are closest to each other

(A) 1/3 s (B) 8/3 s (C) 1/5 s (D) 8/5 s

Solution: (D)

 20 
  tan 1    15 with x  axis
 15 

The motorcyclist appears to move along the line MP with speed 25

The shortest distance = perpendicular distance of MP from

 C  d  d  50cos53  d  30 m

Time taken to come closest = time taken by motorcyclist to reach B

MB 50 sin 53
t   t  1.6 s
v mc 25

Q.17 For a man moving on a horizontal road at a uniform speed of 6 km/hr, rain appears to fall vertically
downwards. When he increases his speed to 12 km/hr rain appears to make 30 with vertical. Find speed
of rain with respect to ground

(A) 12 km/hr (B) 14 km/hr

(C) 16 km/hr (D) 10 km/hr

Solution: (A)

BT-IIT |Camp| FC Rd| PaudRoad|Wanowrie|Aundh|VimanNagar|Deccan|SataraRd|Pimple Saudagar|PCMC|www.bakliwaltutorialsiit.com 10


BT Tests Bakliwal Tutorials - IIT

Q.18 The acceleration of a particle is increasing linearly with time t as a   t . If the particle starts from the
origin with initial velocity u, the distance travelled by it in t second is

1 1 1  t3
(A) ut   t 3 (B) ut   t 3
2
(C) ut   t 3 (D) ut 
2 2 3 6

Solution: (D)

a  t
dv  t.dt
t
v  t2 
 vu   
 2 0
t 2
vu 
2
t 2
v u
2
 t 2 
dx    u  .dt
 2 
 t 3 
x  ut 
 6 

Q.19 The position-time graph of a moving particle is shown in the figure. Shape of curves from t=0 to t=1 and
t=1 to t=2 is parabolic. The velocity-time graph is best given by

BT-IIT |Camp| FC Rd| PaudRoad|Wanowrie|Aundh|VimanNagar|Deccan|SataraRd|Pimple Saudagar|PCMC|www.bakliwaltutorialsiit.com 11


BT Tests Bakliwal Tutorials - IIT

(A) (B)

(C) (D)

Solution: (A)

From figure

For time interval t = 0 to t = 1s

Slope of x-t graph is negative and increasing, so velocity increases in negative direction

For t = 1 to 2 s

The slope is +ve and decreasing, so velocity is decreasing in +ve direction and become zero at t = 2

Q.20 The acceleration-time graph of a moving particle is shown in the figure. The velocity-time graph is given
by

BT-IIT |Camp| FC Rd| PaudRoad|Wanowrie|Aundh|VimanNagar|Deccan|SataraRd|Pimple Saudagar|PCMC|www.bakliwaltutorialsiit.com 12


BT Tests Bakliwal Tutorials - IIT

(A) (B)

(C) (D)

Solution: (A)

As acceleration is increasing slope of v-t curve continuously increases with velocity increasing as
acceleration is decreasing but velocity is increasing though slope is decreasing.

Q.21 The time when relative motion between the blocks will stop is

(A) 1 s (B) 2 s (C) 3 s (D) 4 s

Solution: (A)

Q.22 A block of mass 2 kg is on a horizontal surface. The co-efficient of static and kinetic frictions are 0.6 and
0.2. The minimum horizontal force required to start the motion is applied and if it is continued, the
velocity acquired by the body at the end of the 2nd second is  g  10 ms 2 

(A) 8ms 1 (B) 4 ms 1 (C) 2 ms 1 (D) Zero

Solution: (A)

BT-IIT |Camp| FC Rd| PaudRoad|Wanowrie|Aundh|VimanNagar|Deccan|SataraRd|Pimple Saudagar|PCMC|www.bakliwaltutorialsiit.com 13


BT Tests Bakliwal Tutorials - IIT
F  s mg  12N
f k  4N
F  fk
a  4ms 2
m
v  at  8 m / s1

Q.23 A given object takes m times as much time to slide down a 45 rough incline as it takes to slide down a
perfectly smooth 45 incline. The coefficient of kinetic friction between the object and the incline is
given by

1 1 1 1
(A)  k  (B)  k  1  (C)  k  1  (D)  k 
1  m 2  m2 m2 1  m2

Solution: (B)

1 g  2
S  t ;
2 2
1 g g  2
S    (t m )
2 2 2
1
1  1   k  m 2   k  1 
m2

Q.24 A particle is moving along x-axis with v  t 2  7t  10 . The distance it covers between t = 1s and t = 3s is

(A) 3m (B) 1.5m (C) -1.5m (D) 0m

Solution: (A)

Q.25 Acceleration of a particle moving along a straight line is given by a = 2t – 1. If its velocity at t = 1s is -
20m/s then its velocity at t = 6 s is

(A) -10 m/s (B) 10 m/s (C) 35 m/s (D) 46 m/s

Solution: (B)

BT-IIT |Camp| FC Rd| PaudRoad|Wanowrie|Aundh|VimanNagar|Deccan|SataraRd|Pimple Saudagar|PCMC|www.bakliwaltutorialsiit.com 14


BT Tests Bakliwal Tutorials - IIT
Q.26 The velocity-time graph of a body moving in a straight line is shown in the figure. The ratio of
displacement to the distance travelled by the body in first 6 second is :

(A) 1 : 1 (B) 1 : 2 (C) 1 : 3 (D) 1 : 4

Solution: (B)

Q.27 Two particles 1 and 2 are moving with velocities ⃗ = (4 ̂ − 3 ̂) / and ⃗ = ( ̂ − ̂) /


respectively. The position vectors of the particles at time t = 0 are ⃗ = (5 ̂ + 2 ̂) and

⃗ = (−4 ̂ − 4 ̂) . If they collide at t = 3s, the value of b is:

10
(A) (B) 5 (C) -1 (D) 7
3

Solution: (D)

⃗ +⃗ = ⃗ +⃗

(5 ̂ + 2 )̂ + (4 ̂ − 3 ̂)(3)

= (−4 ̂ − 4 ̂) + ( ̂ − ̂)(3)

17 ̂ − 7 ̂ = (−4 + 3 ) ̂ − 7 ̂

So, (3 − 4) = 17 ⇒ =7

Q.28 A man throws a ball making an angle of 60 with the horizontal. He runs on a level ground with constant
velocity and catches the ball at the same horizontal level as it was thrown. If he had thrown the ball with
speed 4 m/s, then his speed of running must be

(A) 4 m/s (B) 2 m/s (C) 8 m/s (D) 2 3 m / s

Solution: (B)

BT-IIT |Camp| FC Rd| PaudRoad|Wanowrie|Aundh|VimanNagar|Deccan|SataraRd|Pimple Saudagar|PCMC|www.bakliwaltutorialsiit.com 15


BT Tests Bakliwal Tutorials - IIT
Vmax  Vballin horizontal
 Vball cos 60
 4 cos 60
 2 ms -1

Q.29 Two trucks are moving on parallel tracks. A person on one truck projects a ball vertically upward then
path of the ball as seen by four observers : from the ground, from the second truck moving with same
velocity as that first truck, from the second truck moving with speed greater than first one in same
direction and from the second truck moving with speed less than the first truck in same direction are:

(A) Parabola, parabola, parabola and parabola

(B) straight line, straight line, parabola and parabola

(C) parabola, straight line, parabola and parabola

(D) None of the above

Solution: (C)

For the first, third and fourth observer velocity ball and g are neither parallel or anti-parallel so that the
path will be parabola. For the second observer velocity of the ball and g are parallel so that the path will
be straight line

Q.30 An airplane flies northwards from town A to town B and then back again. There is a steady wind blowing
towards the north so that for the first stage of the trip, the airplane is flying in the same direction as the
wind and for the return half of the journey, the airplane is flying directly into the wind. The total trip time
Tw , as compared to the total trip time in the absence of any wind To , obeys:

(A) Tw  To (B) Tw  To (C) Tw  2To (D) Tw  To

Solution: (D)

d d 2du
Tw    2
v  u v  u v  u2
v  speed of plane
u  speed of wind
2d 2d
Tw  2
TO  Clearly
 2 u  v
v  
 v 
Tw  TO

BT-IIT |Camp| FC Rd| PaudRoad|Wanowrie|Aundh|VimanNagar|Deccan|SataraRd|Pimple Saudagar|PCMC|www.bakliwaltutorialsiit.com 16


BT Tests Bakliwal Tutorials - IIT
CHEMISTRY
SECTION - I
Single Correct Answer Type
This section contains 30 multiple choice questions. Each question has four choices (A), (B), (C) and (D) out of
which ONLY ONE is correct.

Q.1 The vapour density of a mixture containing equal number of moles of methane and ethane at STP is

(A) 11.5 (B) 11.0 (C) 23 (D) 12.0

Solution: (A)

Molar mass of mixture will be

n  Molar mass of CH 4  n  Molar mass of C 2 H 6


M mix 
nn
n 16  30 
M mix   23
2n
M 23
Vapour density  mix   11.5
2 2

Q.2 Which of the following has maximum ionic character?

(A) LiCl (B) NaCl (C) KCl (D) CsCl

Solution: (D)

Cl is common hence larger the cation more will be the ionic character (Fajan’s rule). Hence CsCl
will have highest ionic character.

Q.3 In which of the following reactions hybridization of underlined species is not changing?

(A) NH 3  H   NH 4 (B) BF3  F   BF4 (C) C2 H 2  H2  C2 H6 (D) All of these

Solution: (A)

NH 3  H   NH 4
sp3 sp3

BT-IIT |Camp| FC Rd| PaudRoad|Wanowrie|Aundh|VimanNagar|Deccan|SataraRd|Pimple Saudagar|PCMC|www.bakliwaltutorialsiit.com 17


BT Tests Bakliwal Tutorials - IIT
Q.4 A partially dried clay mineral contains 8% water. The original sample contains 12% water and 45%
silica. The % of silica in the partially dried sample is nearly.

(A) 50% (B) 49% (C) 55% (D) 47%

Solution: (D)

Let original sample weighs  g

12 45
water  , silica  x
100 100
43
Im purities  x
100

Partially dried sample weight y g

8
Water  y
100

Since no evaporation of silica & impurities. So,

45 43 92 92
 x yx  y
100 100 100 88

45
x
% of silica in partially dried sample  100  100  47%
y

1 1
Q.5 If the mole fraction of a solute is changed from to by adding some solute in the 800 g of H 2 O
4 2
solvent, then the ratio of molality of two solutions will be:

(A) 1 : 3 (B) 1 : 4 (C) 2 : 3 (D) 1 : 2

Solution: (A)

x A 1000
Molality 
1  x A  mB
0.25 1000
First solution’s molality 
0.75 18

0.5 1000 1
Second solution’s molality   Molality ratio 
0.5 18 3

BT-IIT |Camp| FC Rd| PaudRoad|Wanowrie|Aundh|VimanNagar|Deccan|SataraRd|Pimple Saudagar|PCMC|www.bakliwaltutorialsiit.com 18


BT Tests Bakliwal Tutorials - IIT
Q.6 Which electronic level would allow the hydrogen atom to absorb a photon but not to emit a photon
(A) 3s (B) 2p (C) 2s (D) 1s

Solution: (D)

Fact based.

Q.7 Which does not favour the formation of ionic compound:

(A) the ionization energy of the metal atom should be low

(B) the lattice energy of the compound formed must be low

(C) the electron affinity of the non-metal should be high

(D) the lattice energy of the compound formed must be high

Solution: (B)

Q.8 In C3 O2 the hybridization state of carbon is:

(A) sp (B) sp 2 (C) sp3 (D) Both sp and sp 2

Solution: (A)

OCCCO

Q.9 A particle initially at rest having charge q coulomb. & mass m kg is accelerated by a potential difference
of V volts, what would be its K.E. & de-broglie wavelength respectively after acceleration

h h h h
(A) qV, (B) , qV (C) qV, (D) , qV
2qVm 2qVm mV mV

Solution: (A)

Fact based.

Q.10 The electron affinity of a hypothetical element ‘A’ is 3eV per atom. How much energy in kcal is
released when 10g of ‘A’ is completely converted to A  ion in a gaseous state?

( 1ev per atom  23 kcal mol 1 , Molar mass of A = 30)

BT-IIT |Camp| FC Rd| PaudRoad|Wanowrie|Aundh|VimanNagar|Deccan|SataraRd|Pimple Saudagar|PCMC|www.bakliwaltutorialsiit.com 19


BT Tests Bakliwal Tutorials - IIT
(A) 23 kcal (B) 46 kcal (C) 50 kcal (D) 52 kcal

Solution: (A)

A  e  A   3eV
30 3  23kcal

3  23
 Energy released for conversion of 10g gaseous A into A  ions  10  23kcal
30

Q.11 Which of the following is paramagnetic?

(A) O 2 (B) CN  (C) CO (D) NO

Solution: (A)

It has one unpaired electron.

Q.12 The distance between 4th and 3rd Bohr orbits of He  is:

(A) 2.645 1010 m (B) 1.322 1010 m (C) 1.8511010 m (D) None

Solution: (C)

n2 
r  0.529 A
Z
 16 9  
r4  r3  0.529    A  1.851 10 10 m
 2 2

Q.13 First, second & third ionization energies are 737, 1045 & 7733 KJ/mol respectively. The element can be:

(A) Na (B) B (C) Al (D) Mg

Solution: (D)

Mg has two electrons in its s-subshell after removing these electrons it will obtain inert gas configuration.
So, a big jump in I.E. will be there

BT-IIT |Camp| FC Rd| PaudRoad|Wanowrie|Aundh|VimanNagar|Deccan|SataraRd|Pimple Saudagar|PCMC|www.bakliwaltutorialsiit.com 20


BT Tests Bakliwal Tutorials - IIT
2
Q.14 The correct order of hybridization of the central atom in the following species NF3 , BF3 , PF5 and SiF6 
is:

(A) sp 2 ,sp3 , sp3d 2 ,sp3d (B) sp3 , sp 2 ,sp3d,sp 3d 2

(C) sp 2 ,sp3 ,sp3d,sp 3d 2 (D) sp3 ,sp 2 ,sp3d,sp 3d 3

Solution: (B)

Q.15 Which of the following arrangements is correct on the basis of the increasing p-character of the hybrid
orbitals of the central atoms in the following:

(I) CIO 2 (II) CS2 (III) SnCl2

(A) I > III > II (B) II > I > III (C) I > II > III (D) III > I > II

Solution: (A)

Q.16 Which of the following are isostructural?

(I) NO3 (II) CO32  (III) CIO3 (IV) SO 3' (V) XeO3

(A) (I) and (IV) (B) (II) and (V) (C) (III) and (IV) (D) (IV) and (V)

Solution: (A)

BT-IIT |Camp| FC Rd| PaudRoad|Wanowrie|Aundh|VimanNagar|Deccan|SataraRd|Pimple Saudagar|PCMC|www.bakliwaltutorialsiit.com 21


BT Tests Bakliwal Tutorials - IIT

Q.17 Which of the following is a planar molecule?

(A) XeO 2 F2 (B) XeOF3 (C) XeF4 (D) XeF6

Solution: (C)

Q.18 Which one of the following represents the INCORRECT decreasing order of bond angles?

(A) CO 2  BF3  CH 4  H 2 O (B) NO 2  NO 2  NO 2

(C) BCl3  PCl3  AsCl3  BiCl3 (D) IO3  BrO3  CIO3

Solution: (D)

BT-IIT |Camp| FC Rd| PaudRoad|Wanowrie|Aundh|VimanNagar|Deccan|SataraRd|Pimple Saudagar|PCMC|www.bakliwaltutorialsiit.com 22


BT Tests Bakliwal Tutorials - IIT

Q.19 Which set does not show correct matching?

(A) Sc3  Ne 3s 2 3p6 zero group (B) Fe2  Ar  3d6 8th group

(C) Cr  Ar  3d5 4s1 6th group (D) All of the above

Solution: (A)
18 18
21 Sc3 ;  Ar  3d 0 4s 0 and 21 Sc;  Ar  3d1 4s 2

As last electron enters in d-subshell so it belongs to d-block and thus its group number = 2 + 1 = 3

Sc3 belong to 3rd group of d-block, not zero group.

Q.20 Among the following, the pair in which the two species are not isostructural is

(A) IO 3 and XeO 3 (B) AlH 4 and PH 4 (C) AsF6 and SF6 (D) SiF4 and SeF4

Solution: (D)

Q.21 The maximum number of atoms which lie in the same plane in B2 H 6 molecule is:

(A) 5 (B) 6 (C) 4 (D) 8

BT-IIT |Camp| FC Rd| PaudRoad|Wanowrie|Aundh|VimanNagar|Deccan|SataraRd|Pimple Saudagar|PCMC|www.bakliwaltutorialsiit.com 23


BT Tests Bakliwal Tutorials - IIT
Solution: (B)

Four terminal hydrogen atoms and the two boron atoms lie in one plane in B2 H 6 .

Q.22 Arrange the following compounds in increasing order of their ionic character

SnCl2 ,SnCl 4 ,SiCl 4 ,SnF4 ,SnF2

(A) SnF2  SnCl2  SnF4  SnCl 4  SiCl 4 (B) SnF2  SnCl2  SnF4  SiCl 4  SnCl 4

(C) SiCl4  SnCl4  SnF4  SnCl 2  SnF2 (D) SnCl4  SnF4  SnCl 2  SnF2  SiCl 4

Solution: (C)

As the size of the cations increases in the order Si 4  Sn 4  Sn 2 and the size of anions, F  Cl

So, the order of increasing ionic character is, SiCl4  SnCl4  SnF4  SnCl 2  SnF2 .

Q.23 Which of the following statements are correct?

(I) In ICl2 , CIF3 and TeCl4 , the number of lone pair(s) of electron on central atoms are 3, 2 and 1
respectively.

(II) Amongst CO, CO 2 , CO 32  , CH 3OH , the correct order from the weakest to the strongest

carbon-oxygen bond is CH 3OH  CO 32  CO 2  CO

(III) The hybridization of boron in BF3 is the same which nitrogen has in CINO molecule.

(A) Only I and III (B) II (C) Only I & II (D) I, II & III

Solution: (D)

BT-IIT |Camp| FC Rd| PaudRoad|Wanowrie|Aundh|VimanNagar|Deccan|SataraRd|Pimple Saudagar|PCMC|www.bakliwaltutorialsiit.com 24


BT Tests Bakliwal Tutorials - IIT

Q.24 The correct order of strength of H-bond in the following compound:

(A) H 2 O  H 2 O 2  HF  H 2S (B) HF  H 2 O 2  H 2 O  H 2S

(C) HF  H 2 O  H 2S  H 2O 2 (D) HF  H 2 O  H 2O 2  H 2S

Solution: (D)

Strength of H-bonding is higher in H 2 O , than H 2 O2 because the amount of formal negative charge on
oxygen atom in case of water is more than that of H 2 O2 .

Q.25 Give the correct order of initials T or F for following statements. Use T if statements is true and F it is
false:

(I) The order of repulsion between different pair of electrons is lp  lp  l p  b p  b p  b p

(II) In general, as the number of lone pair of electrons on central atom increases, value of bond angle
from normal bond angle also increases

(III) The number of lone pair on O in H 2 O is 2 while in N in NH 3 is 1

(IV) The structures of xenon fluorides and xenon oxy fluorides could not be explained on the basis of
VSEPR Theory.

(A) TTTF (B) TFTF (C) TFTT (D) TFFF

Solution: (B)

(II) (F) In general as the number lone pair of electrons on central atom increases, value of bond angle
from normal bond angle decreases due to lp – lp > lp – bp.

(iv) (F) Structure of xenon fluorides and xenon oxy fluoride are explained on the basis of VSEPR theory

Q.26 In a molecule, the ionic charge is 4.8 × 10 esu. If the interionic distance is 1 unit, then the dipole
moment is

(A) 0.48 debye (B) 4.18 debye (C) 4.8 debye (D) 41.8 debye

Solution: (C)

BT-IIT |Camp| FC Rd| PaudRoad|Wanowrie|Aundh|VimanNagar|Deccan|SataraRd|Pimple Saudagar|PCMC|www.bakliwaltutorialsiit.com 25


BT Tests Bakliwal Tutorials - IIT

Q.27 Two types of carbon – carbon covalent bond lengths are present in

(A) Carbonate Ion (B) (C) Propene (D) Benzene

Solution: (C)

Q.28 Which of the following atomic orbitals overlapping are not allowed for bonding?

(A) All (B) (i) (ii) (iii) (C) (i)(iii)(v) (D) (ii) only

Solution: (B)

Fact based.

Q.29 The mole fraction of the solute in the 12 molal solution of Na 2 CO3

(A) 0.822 (B) 0.177 (C) 1.77 (D) 0.017

Solution: (B)

m Na 2CO3  12
n Na 2 CO3  12
1000
n H 2O   55.55
18
12
x Na 2CO3   0.177
12  55.55

BT-IIT |Camp| FC Rd| PaudRoad|Wanowrie|Aundh|VimanNagar|Deccan|SataraRd|Pimple Saudagar|PCMC|www.bakliwaltutorialsiit.com 26


BT Tests Bakliwal Tutorials - IIT
Q.30 Number of electrons having + value equal to zero in 26 Fe may be

(A) 13 (B) 18 (C) 7 (D) 12

Solution: (A)

MATHS
SECTION - I
Single Correct Answer Type
This section contains 30 multiple choice questions. Each question has four choices (A), (B), (C) and (D) out of
which ONLY ONE is correct.


be the set of integers. If A  x  Z : 2  x  2  x 
 B   x  Z: 3  2x 1  9 , then the
2
5 x  6
Q.1 Let  1 and

number of subsets of the set ∪ , is:

(A) 2 (B) 2 (C) 2 (D) 2

Solution: (C)

n 10
1
Q.2 If Sn   Tr  n  n  1 n  2 n  3, then  T is equal to:
r 1 r 1 r

(A) 65 (B) 75 (C) 65 (D) 58


1056 1056 528 528

Solution: (A)

3 13 5  1  2  7  1  2  3 
3 3 3 3 3

Q.3 The Sum 2


 2 2
 2 2 2
 ....... up to 10th term, is
1 1 2 1  2 3

(A) 680 (B) 600 (C) 660 (D) 620

Solution: (C)

BT-IIT |Camp| FC Rd| PaudRoad|Wanowrie|Aundh|VimanNagar|Deccan|SataraRd|Pimple Saudagar|PCMC|www.bakliwaltutorialsiit.com 27


BT Tests Bakliwal Tutorials - IIT

Q.4 Let , be positive real numbers and , positive integers. The maximum value of the expression
m n
x y
1  x 1  y 2 n 
2m

(A) 1 (B) 1 (C) 1 (D) m  n


2 4 6mn

Solution: (C)
3 3 3 3
3  1  1  3
Q.5 If the sum of the first 15 terms of the series     1    2   33   3   ...... is equal to 225 ,
4  2  4  4
is equal to:

(A) 108 (B) 27 (C) 54 (D) 9

Solution: (B)

Q.6 The sum of the solutions of the equation x 2  x  


x  4  2  0,  x  0  is equal to:

(A) 9 (B) 12 (C) 4 (D) 10

Solution: (D)

Q.7 Let and be the roots of the quadratic equation x 2sin   x  sin  cos    cos     0    o  , and
 n  1n 

< . Then    n  is equal to:
n 0
  

1 1 1 1
(A)  (B) 
1  cos  1  sin  1  cos  1  sin 

1 1 1 1
(C)  (D) 
1  cos  1  sin  1  cos  1  sin 

Solution: (C)

Q.8 If three distinct numbers a, b, c are in G.P. and the equations ax 2  2bx  c  0 and dx 2  2ex  f  0
have a common root, then which one of the following statements is correct?

(A) d , e , f are in A.P. (B) d, e, f are in A.P.


a b c
BT-IIT |Camp| FC Rd| PaudRoad|Wanowrie|Aundh|VimanNagar|Deccan|SataraRd|Pimple Saudagar|PCMC|www.bakliwaltutorialsiit.com 28
BT Tests Bakliwal Tutorials - IIT
(C) d, e, f are in G.P. (D) d , e , f are in G.P.
a b c

Solution: (A)

Q.9 The number of integral values of m for which the quadratic expression,
1 2m x2  2 1  3m x  4 1 m , x  R , is always positive, is:
(A) 3 (B) 8 (C) 7 (D) 6

Solution: (C)

such that sum of the squares of the roots of the quadratic equation , x   3  x  2  
2
Q.10 The values of
has the least value is:

(A) 15 (B) 1 (C) 4 (D) 2


8 9

Solution: (D)

Q.11 Let f k  x   1  sin k x  cos k x  for = 1, 2, 3, …. Then for all xR , the value of f4  x  f6  x  is equal
k
to:

(A) 1 (B) 1 (C) 1 (D) 5


12 4 12 12

Solution: (A)

Q.12 If cos       3 , sin       5 and 0      then tan  2 is equal to:


5 13 4

(A) 63 (B) 63 (C) 21 (D) 33


52 16 16 52

Solution: (B)

Q.13 The least value of the expression 2log10 x  logx 0.01, for x  1, is _______.

BT-IIT |Camp| FC Rd| PaudRoad|Wanowrie|Aundh|VimanNagar|Deccan|SataraRd|Pimple Saudagar|PCMC|www.bakliwaltutorialsiit.com 29


BT Tests Bakliwal Tutorials - IIT
(A) 10 (B) 4 (C) 2 (D) −2

Solution: (B)

Q.14 If sin   cos ec   2, the value of sin 10   cos ec 10  is

(A) 10 (B) 2 (C) 2 (D) 2

Solution: (D)

Q.15 Evaluate log5 log2 log3 log2 512

(A) 1 (B) 0 (C) 2 (D) −1

Solution: (B)

Q.16 The value of cos15 o  sin 15 o is equal to

1 1
(A) (B) 1 (C)  (D) 0
2 2 2

Solution: (A)

Q.17 If 1, log 9  31 x  2  , log 3  4.3x  1 are in A. P. then equals

(A) log3 4 (B) 1log3 4 (C) 1 log4 3 (D) log4 3

Solution: (B)

cos17o  sin17o
Q.18 
cos17o  sin17o
(A) tan 62 o (B) tan 56 o (C) tan 54 o (D) tan 73 o

Solution: (A)

BT-IIT |Camp| FC Rd| PaudRoad|Wanowrie|Aundh|VimanNagar|Deccan|SataraRd|Pimple Saudagar|PCMC|www.bakliwaltutorialsiit.com 30


BT Tests Bakliwal Tutorials - IIT
3 1
Q.19    , then 2cot   is equal to
4 sin 2 

(A) 1  cot  (B) 1  cot  (C) 1  cot  (D) 1  cot 

Solution: (B)

sin 3  sin 5   sin 7   sin 9 


Q.20 is equal to
cos 3  cos 5   cos 7   cos 9 

(A) tan3 (B) cot3 (C) tan 6 (D) cot6

Solution: (C)

Q.21 The value of tan27o  tan18o  tan27o tan18o , is

(A) An irritational number

(B) Rational which is not integer

(C) Integer which is prime

(D) Integer which is not a prime

Solution: (D)

Q.22 If tan A and tan B are the roots of the quadratic equation x2 ax b  0, then the value sin2  A  B is:

a2 a2 a2 a2
(A) (B) 2 2 (C) (D)
a 2  1  b 
2
a b b  a 
2
b 2 1  a 
2

Solution: (A)

Q.23   
Let P   : sin   cos   2 cos  and Q   : sin   cos   2 sin  be two sets. Then:
(A) P  Q Q  P   (B) Q  P (C) P  Q (D) P  Q

Solution: (D)

BT-IIT |Camp| FC Rd| PaudRoad|Wanowrie|Aundh|VimanNagar|Deccan|SataraRd|Pimple Saudagar|PCMC|www.bakliwaltutorialsiit.com 31


BT Tests Bakliwal Tutorials - IIT

tan A cot A
Q.24 The expression  can be written as:
1  cot A 1  tan A

(A) sin A cos A  1 (B) sec A cos ec A  1 (C) tan A  cot A (D) sec A  cos ec A

Solution: (B)

Q.25 If      and      , then tan equals


2

(A) 2  tan   tan   (B) tan   tan  (C) tan    tan  (D) 2 tan   tan 

Solution: (C)

Q.26 Let        . Suppose 1 and 1 are the roots of the equation x 2  2x sec   1  0 and 2 and 2
6 12
are the roots of the equation x 2  2x tan      . If 1 1 and 2 2, then 1 2 equals

(A) 2  sec   tan   (B) 2 sec  (C) 2 tan  (D) 0

Solution: (C)

such that f  x   x  px  q has a factor x 2  3x  2 is


4 2
Q.27 The value of and

(A) = 4, = 5 (B) = −1, = −4 (C) = −5, = −4 (D) = 5, =4

Solution: (D)

Q.28 If   are the roots of ax 3  bx  c  0 , then the equation whose roots are          is

(A) ax 3  bx  c  0 (B) ax 3  bx  c  0 (C) cx 3  bx  c  0 (D)  ax 3  bx 2  c  0

Solution: (B)

such that  n  2 x  8x  n  4  0 x  R where n  N .


2
Q.29 The least value of

(A) 5 (B) 6 (C) 4 (D) 7


BT-IIT |Camp| FC Rd| PaudRoad|Wanowrie|Aundh|VimanNagar|Deccan|SataraRd|Pimple Saudagar|PCMC|www.bakliwaltutorialsiit.com 32
BT Tests Bakliwal Tutorials - IIT
Solution: (A)

Q.30 Number of solutions of log4  x 1  log2  x  3 is

(A) 3 (B) 1 (C) 2 (D) 0

Solution: (B)

BT-IIT |Camp| FC Rd| PaudRoad|Wanowrie|Aundh|VimanNagar|Deccan|SataraRd|Pimple Saudagar|PCMC|www.bakliwaltutorialsiit.com 33


Date: 22nd Sep 2019 Bakliwal Tutorials-IIT
Comprehensive 2021 B-Test 4 YOUR STRATEGIC MOVE

Total Time: - 3 hrs. Sub: - Physics, Chemistry & Math Max. Marks: -240

Rules & regulations for BTEST

1. Immediately fill in the particulars on this page of the Test Booklet with Ball Point Pen.
2. The Answer Sheet (OMR) is provided separately. Please fill in the particulars carefully with Ball Point Pen
only.
3. The test is of 3 hours duration.
4. The Test Booklet consists of 60 questions. The maximum marks are 240.
5. There are three parts in the question paper consisting of Physics, Chemistry and Mathematics having 20
questions in each part. Question pattern of question paper is as follows,
Advance Pattern
Single Correct Answer Type -
This section contains 5 multiple choice questions. (Marking +3, -1)

Multiple Correct Answer Type -


This section contains 6 multiple choice questions. (Marking +4, 0)

Integer Answer Type - Answer should be 0 to 9


This section contains 5 questions. (Marking +5, 0)

Matrix Answer Type –


This section contains 4 questions. (Marking +4, 0)
Each question has four statements (A, B C and D) given in Column I and four or five statements (P, Q, R, S
and T) in column II, in which ONE OR MORE may be correct.

6. No candidate is allowed to carry any textual material, printed or written, bits of papers, pager, mobile phone,
any electronic device, etc. except the BT ID Card inside the examination room/hall.
7. Rough work is to be done on the additional sheets provided
8. On completion of the test, the candidate must immediately hand over the Answer Sheet to the Invigilator on
duty in the Room/Hall. However, the candidates are allowed to take away this Test Booklet with them.
9. The CODE for this Booklet is A. Make sure that the CODE is marked properly on the answer sheet.
10. Do not fold or make any stray mark on the Answer Sheet.

BT-IIT |Camp| FC Rd| PaudRoad|Wanowrie|Aundh|VimanNagar|Deccan|SataraRd|Pimple Saudagar|PCMC|www.bakliwaltutorialsiit.com 1


BT Tests Bakliwal Tutorials - IIT

PHYSICS
SECTION - I
Single Correct Answer Type
This section contains 5 multiple choice questions. Each question has four choices (A), (B), (C) and (D) out of which
ONLY ONE is correct.

Q.1 Two beads A and B move along a semicircular wire frame as shown in figure. The beads are connected
by an inelastic string which always remains tight. At an instant the speed of A is u. ∠BAC =
45° and ∠BOC = 75° , where O is the centre of the semicircular arc. The speed of bead B at that
instant is:

(A) √2 (B) (C) (D)



Solution: (A)

Q.2 A large mass M hangs stationary at the end of a light string that passes through a smooth fixed tube to
a small mass m that moves around in a horizontal circular path. It ℓ is the length of the string from m
to the top end of the tube and is angle between this part and vertical part of the string as shown in
the figure, then time taken by m to complete one circle is equal to

ℓ ℓ ℓ ℓ
(A) 2 (B) 2 (C) 2 (D) 2

Solution: (D)

BT-IIT |Camp| FC Rd| PaudRoad|Wanowrie|Aundh|VimanNagar|Deccan|SataraRd|Pimple Saudagar|PCMC|www.bakliwaltutorialsiit.com 2


BT Tests Bakliwal Tutorials - IIT

Q.3 In the figure shown a lift goes downwards with a constant retardation. An observer in the lift observers
a conical pendulum in the lift, revolving in a horizontal circle with time period 2 seconds. The distance
between the centre of the circle and the point of suspension is 2.0 m. Find the retardation of the lift in
/ .

(A) 10 (B) 5 (C) 2 (D) 4


Solution: (A)

Q.4 System is shown in the figure. Assume that cylinder remains in contact with the two wedges. The
velocity of cylinder is –


(A) 19 − 4√3 / (B) / (C) √3 / (D) √7 /
Solution: (D)

BT-IIT |Camp| FC Rd| PaudRoad|Wanowrie|Aundh|VimanNagar|Deccan|SataraRd|Pimple Saudagar|PCMC|www.bakliwaltutorialsiit.com 3


BT Tests Bakliwal Tutorials - IIT

Q.5 A small ring P is threaded on a smooth wire bent in the form of a circle of radius a and center O. The
wire is rotating with constant angular speed about a vertical diameter XY, while the ring remains at
rest relative to the wire at a distance a/2 from XY. Then is equal to


(A) (B) (C) (D)

Solution: (C)

BT-IIT |Camp| FC Rd| PaudRoad|Wanowrie|Aundh|VimanNagar|Deccan|SataraRd|Pimple Saudagar|PCMC|www.bakliwaltutorialsiit.com 4


BT Tests Bakliwal Tutorials - IIT

SECTION – II
(Multiple Correct Answer(s) Type)
This section contains 6 multiple choice questions. Each question has four choices (A), (B), (C) and (D)
Out of Which ONE OR MORE may be correct.

Q.6 A block of mass m is placed on a wedge. The wedge can be accelerated in four manners marked as (1)
(2), (3) and (4) as shown. If the normal reactions in situation (1), (2), (3) and (4) are N , N , N and
N respectively and acceleration with which the block slides on the wedge in situations are
b ,b ,b b respectively then:

(A) > > > (B) > > >


(C) > > > (D) > > >
Solution: (AC)

BT-IIT |Camp| FC Rd| PaudRoad|Wanowrie|Aundh|VimanNagar|Deccan|SataraRd|Pimple Saudagar|PCMC|www.bakliwaltutorialsiit.com 5


BT Tests Bakliwal Tutorials - IIT

Q.7 A bob is attached to a string as shown in the figure below. At the lowest point, it is given a horizontal
velocity u. The bob rotates complete circle in vertical plane. For different values of u, it can have
different total acceleration at different points. For each of the points marked in column-I, match the
possible direction(s) of the total acceleration given by statements (i), (ii), (iii), (iv), and (v)

Column I Column II
(I)
(II)

(III)

BT-IIT |Camp| FC Rd| PaudRoad|Wanowrie|Aundh|VimanNagar|Deccan|SataraRd|Pimple Saudagar|PCMC|www.bakliwaltutorialsiit.com 6


BT Tests Bakliwal Tutorials - IIT
(IV)

(V)

Match the situation of the particle with the statement given in Column-II
(A) A-II (B) B-I, IV, V (C) C-I, II (D) D-III, IV
Solution: (ABD)
Q.8 In the figure shown all the surface are smooth. All the blocks A, B and C are movable, x-axis is
horizontal and y-axis vertical as shown. Just after the system is released from the position as shown.

(A) Acceleration of ‘A’ relative to ground is in negative y-direction


(B) Acceleration of ‘A’ relative to B is in positive x-direction
(C) The horizontal acceleration of ‘B’ relative to ground is in negative x-direction
(D) The acceleration of ‘B’ relative to ground directed along the inclined surface of ‘C’ is greater than
sin
Solution: (ABCD)
Q.9 Two particles move on a circular path (one just inside and the other just outside) with angular velocities
and 5 starting from the same point. Then:
(A) they cross each other at regular intervals of time when their angular velocities are oppositely
directed
(B) they cross each other at points on the path subtending an angle of 60° at the centre if their angular
velocities are oppositely directed
(C) they cross at intervals of time if their angular velocities are oppositely directed
(D) they cross each other at points on the path subtending 90° at the centre if their angular velocities
are in the same sense
Solution: (BCD)
Q.10 Figure shows two blocks A and B connected to an ideal pulley string system. In this system when
bodies are released then : (neglect friction and take = 10 / )

(A) Acceleration of block A is 1 /


(B) Acceleration of block A is 2 /
BT-IIT |Camp| FC Rd| PaudRoad|Wanowrie|Aundh|VimanNagar|Deccan|SataraRd|Pimple Saudagar|PCMC|www.bakliwaltutorialsiit.com 7
BT Tests Bakliwal Tutorials - IIT
(C) Tension in string connected to block B is 40 N
(D) Tension in string connected to block B is 80 N
Solution: (BD)

Q.11 A car moving along a circular track of radius 50.0 m, accelerates from rest at 3.00 . Consider a
situation when the car’s centripetal acceleration equals its tangential acceleration.
(A) The angle around the track does the car travel is 1 rad
(B) The magnitude of the car’s total acceleration at that instant is 3√2
(C) Time elapses before this situation is
(D) the distance travelled by the car during this time 25 m.
Solution: (BCD)

= =
.
3= ⇒ =
The angular acceleration of car = =
The angle rotated by car
= = ×
Distance travelled by car upto this instant is
= . = × 50 = 25
Net acceleration of the car is a
Total = + = √3 + 3 = 3√2

SECTION -III
(One Integer Value Correct Type)
This section contains 5 questions. Each question, when worked out will result in one integer from 0 to 9. (Both
inclusive)

Q.12 A wire frame is in the shape of a parabola = 5 . It is being rotated about y-axis with an angular
velocity . A small bead of mass (m = 1 kg) is at point P and is in equilibrium with respect to the
frame. Then the angular = 5 , find n

BT-IIT |Camp| FC Rd| PaudRoad|Wanowrie|Aundh|VimanNagar|Deccan|SataraRd|Pimple Saudagar|PCMC|www.bakliwaltutorialsiit.com 8


BT Tests Bakliwal Tutorials - IIT
Solution: (2)

Q.13

Find tension in string connecting the pulleys


Solution: (0)
Q.14 In the figure shown all contact surfaces are smooth. Acceleration of B block will be

Solution: (1)

BT-IIT |Camp| FC Rd| PaudRoad|Wanowrie|Aundh|VimanNagar|Deccan|SataraRd|Pimple Saudagar|PCMC|www.bakliwaltutorialsiit.com 9


BT Tests Bakliwal Tutorials - IIT
Q.15 A rod AB is shown in figure. End A of the rod is fixed on the ground. Block is moving with velocity
√3 / towards right. The velocity of end B of rod when rod makes an angle of 60° with the ground
is:

Solution: (2)

Q.16 A car travelling on a smooth road passes through a curved portion of the road in the form of an arc of
circle of radius 10 m. If the mass of car is 120 kg, find the reaction (in ) on car at lowest point P
where its speed is 20 .

Solution: (6)

SECTION –IV (Match the Column)


This section contains 4 questions. Each question has four statements (A, B, C, and D) given in Column I and four
statements (P, Q, R, S & T) in column II, in which ONE OR MORE may be correct.
Q.17 A man pushes a block of 30 kg along a level floor at a constant speed with a force directed at 45°
below the horizontal. If the coefficient of friction is 0.20, then match the following.
Column I Column II
(A) Work done by all forces exerted by the (P) Zero
surface on the block in 20 m
(B) Work done by the force of gravity (Q) -1500 J
(C) Work done by the man one the block in (R) 750 J
pushing it through 10 m
(D) Net force on the block (S) 30 J

Solution: [(A) –Q; (B) –P; (C) –R; (D) -P ]

BT-IIT |Camp| FC Rd| PaudRoad|Wanowrie|Aundh|VimanNagar|Deccan|SataraRd|Pimple Saudagar|PCMC|www.bakliwaltutorialsiit.com 10


BT Tests Bakliwal Tutorials - IIT

Q.18 In the system shown in Figure masses of the blocks are such that when system is released, the
acceleration of pulley is a upwards and the acceleration of block 1 is upwards. It is found that
the acceleration of block 3 is same as that of 1 both in magnitude and direction.
Given that > > /2. Math the following:

Column I Column II
(A) Acceleration of 2 (P) 2 +
(B) Acceleration of 4 (Q) 2 −
(C) Acceleration of 2 w.r.t. 3 (R) Upwards
(D) Acceleration of 2 w.r.t. 4 (S) Downwards

Solution: [(A) – QR; (B) – PS; (C) – S; (D) - R]

BT-IIT |Camp| FC Rd| PaudRoad|Wanowrie|Aundh|VimanNagar|Deccan|SataraRd|Pimple Saudagar|PCMC|www.bakliwaltutorialsiit.com 11


BT Tests Bakliwal Tutorials - IIT

Q.19

Column I Column II
(A) (P) = cos

Conical pendulum
(B) (Q) cos =

Pendulum is swinging angular position


is the extreme position T is tension in
extreme position
(C) (R) Speed of ball with respect to ground is
constant

The car is moving with constant


acceleration. The ball is at rest with
respect to car
(D) (S) Velocity of ball with respect to ground is
changing conitnuously

BT-IIT |Camp| FC Rd| PaudRoad|Wanowrie|Aundh|VimanNagar|Deccan|SataraRd|Pimple Saudagar|PCMC|www.bakliwaltutorialsiit.com 12


BT Tests Bakliwal Tutorials - IIT
The car is moving with constant velocity.
The ball is at rest with respect to car
Solution: [(A) – QRS; (B) – PS; (C) – QS; (D) - R]

Q.20 A particle is moving with speed = 2 on the circumference of circle of radius R. Match the
quantities given in column I with corresponding results in Column II
Column I Column II
(A) Magnitude of tangential acceleration of (P) Decreases with time
particle
(B) Magnitude of Centripetal acceleration of (Q) Increases with time
particle
(C) Magnitude of angular speed of particle (R) Remains constant
with respect to centre of circle
(D) Angle between the total acceleration (S) Depends on the value of radius R
vector and centripetal acceleration vector
of particle
Solution: [(A) – Q; (B) – QS; (C) – QS; (D) - PS]

CHEMISTRY
SECTION - I
Single Correct Answer Type
This section contains 5 multiple choice questions. Each question has four choices (A), (B), (C) and (D) out of which
ONLY ONE is correct.

Q.1 Radiations corresponding to the transition n = 3 to n = 1 in hydrogen atom falls on a certain metal of
work function 3 :
(I) The maximum kinetic energy of the photoelectron will be 9.09
(II) Stopping potential will be 9.09
(III) The maximum kinetic energy of the photoelectron will be 8.09
(IV) Stopping potential will be 8.09
Select the correct option:
(A) I, IV (B) I, II (C) II, III (D) III, IV
Solution: (B)

BT-IIT |Camp| FC Rd| PaudRoad|Wanowrie|Aundh|VimanNagar|Deccan|SataraRd|Pimple Saudagar|PCMC|www.bakliwaltutorialsiit.com 13


BT Tests Bakliwal Tutorials - IIT

Q.2 In the reaction ( )+ ( ) → ( )+ (ℓ)


When 1 mol of ammonia and 1 mol of are made to react:
(A) 2 moles of NO is produced (B) All the oxygen will be consumed
(C) All the ammonia will be consumed (D) None is limiting reagent
Solution: (B)

Q.3 and are converted to monocations respectively, which is wrong statement:


(A) In , the N – N bond weakens (B) In , the O – O bond order increases
(C) In , the paramagnetism decreases (D) becomes diamagnetic
Solution: (D)

Q.4 100 ml of an aqueous solution of (density = 1.8 g/ml) is diluted such that its final density
becomes 1.2 g/ml, volume of added is (density of =1 / )
(A) 150 ml (B) 300 ml (C) 600 ml (D) None
Solution: (B)
Let x ml of is added
= 1.2
= 300

Q.5 The rates of effusion of two gases A and B under the same condition of temperature and pressure are
in the ratio ∶ = 2 ∶ 1. What would be the ratio of the rms speeds of molecules of A and B at
temperature and respectively if ∶ = 2 ∶ 1?
(A) 2√2 ∶ 1 (B) 2 : 1 (C) 4 : 1 (D) √3 ∶ 1
Solution: (A)

SECTION – II
(Multiple Correct Answer(s) Type)
BT-IIT |Camp| FC Rd| PaudRoad|Wanowrie|Aundh|VimanNagar|Deccan|SataraRd|Pimple Saudagar|PCMC|www.bakliwaltutorialsiit.com 14
BT Tests Bakliwal Tutorials - IIT
This section contains 6 multiple choice questions. Each question has four choices (A), (B), (C) and (D)
Out of Which ONE OR MORE may be correct.
Q.6 For similar nature of below graph where ≤ 5, select correct statement(s).

(A) Angular node present in orbital may be 1


(B) Angular nodes present in orbital may be 3
(C) For possible orbitals magnetic quantum number may be 2
(D) For possible orbital value of total nodes must be less than 4
Solution: (AC)

Q.7 The outermost electronic configuration of 7 N in ground state can be:

(A) (B)

(C) (D)
Solution: (AB)

Q.8 Indicate the correct statement for equal volumes of ( ) ( ) at 25° and 1 atm.
(A) The average translational KE per molecule is the same for and
(B) The rms speed remains same for both and
(C) The density of is less than that of
(D) The total translational KE of both and is the same
Solution: (ACD)

Q.9 The pressure of 11 gm of a gas which is placed in a 4 litres container at 127° is 2 atm, then the gas
would be? (Take R = 0.08 ltr. atm/mol K)
(A) (B) (C) (D)
Solution: (AD)

BT-IIT |Camp| FC Rd| PaudRoad|Wanowrie|Aundh|VimanNagar|Deccan|SataraRd|Pimple Saudagar|PCMC|www.bakliwaltutorialsiit.com 15


BT Tests Bakliwal Tutorials - IIT
=
× . ×
= = ×
= 44
Gases are &

Q.10 If the rms velocity of nitrogen and oxygen molecule are same at two different temperature and same
pressure then
(A) Average speed of molecules is also same
(B) Density (gm/lt) of nitrogen and oxygen is also equal
(C) Number of moles of each gas is also equal
(D) most probable velocity of molecules is also equal
Solution: (ABD)

Q.11 Which is true about , , ?


(A) Hybridization of N is same
(B) No. of lone pair of electron on N are same
(C) Molecular geometry (i.e. shape) is different
(D) Bond angle is same
Solution: (AC)

SECTION -III
(One Integer Value Correct Type)
This section contains 5 questions. Each question, when worked out will result in one integer from 0 to 9. (Both
inclusive)

Q.12 If the ratio of and RT for a real gas is and at a temperature where = 0 then value of x will
be
Solution: (9)
At critical temperature & pressure =
=
=9

Q.13 How many maximum numbers of atoms are present in one plane in Diborane (B2H6)?
Solution: (06)

Q.14 In how many following species, the central atoms have two lone pairs of electrons?

Solution: (7)

BT-IIT |Camp| FC Rd| PaudRoad|Wanowrie|Aundh|VimanNagar|Deccan|SataraRd|Pimple Saudagar|PCMC|www.bakliwaltutorialsiit.com 16


BT Tests Bakliwal Tutorials - IIT
Q.15 How many of following molecules have − bonding?

Solution: (07)
In II, III, IV, V, III, IX, & X

Q.16 The maximum number of halogen atoms which are present in one plane for dimer of { } is
……
Solution: (06)

SECTION –IV (Match the Column)


This section contains 4 questions. Each question has four statements (A, B, C, and D) given in Column I and four
statements (P, Q, R, S & T) in column II, in which ONE OR MORE may be correct.
Q.17 MATCH LISTING
LIST-I LIST-II
(A) Ratio of ionisation energy for (P) 2:1
and , respectively
(B) Ratio of total energy of electron in 1st (Q) 1:2
excited state of and potential
th
energy of electron in 5 excited state of
, respectively
(C) Ratio of time period of revolution of (R) 1:4
electron in 1 st orbit of and 2 nd
orbit of , respectively
(D) Ratio of radius of 3 rd excited state of (S) 1:1
and 1st excited state of H-atom
respectively

Solution: [(A) – R; (B) – P; (C) – Q; (D) - P]

BT-IIT |Camp| FC Rd| PaudRoad|Wanowrie|Aundh|VimanNagar|Deccan|SataraRd|Pimple Saudagar|PCMC|www.bakliwaltutorialsiit.com 17


BT Tests Bakliwal Tutorials - IIT
Q.18 Match the compounds listed in column I with characteristic(s) listed in column II
Column-I Column-II
(A) (P) Tetrahedral hybridization
(B) (Q) Trigonal planar hybridization
(C) ( ) (R) Empty orbital(s) of central atom
participate in hybridization
(D) ( ) (S) Two types of bonds
(T) Two types of bond angles.

Solution: [(A) – P,R,S,T; (B) – P,R,S,T; (C) – P,R,S,T; (D) - Q]

Q.19 Match the following:


Column I Column II
(A) H (a = 0.244 litre atm mole ; b (P) Maximum boiling point
= 0.02 litre mole )
(B) He (a = 0.03412 litre atm mole ; b (Q) Minimum boiling point
= 0.02370 litre mole )
(C) ( = 3.592 ; (R) Critical temperature < Boyle temp.
= 0.0426 )
(D) ( = 5.464 ; (S) Minimum critical temperature
= 0.03049 )
(T) Minimum Boyle temperature

Solution: [(A) –R ; (B) –QRST ; (C) – R; (D) - PR]

Q.20 Match the characteristics mentioned in column II with the process in Column I
Column-I Column-II
(A) ( )+ → ( ) (P) Positive Electron gain enthalpy
(B) ( )+ → ( ) (Q) Negative Electron gain enthalpy
(C) ( ) → ( )+ (R) Exothermic
(D) ( )+ → ( ) (S) Endothermic

Solution: [(A) – QR; (B) – PS; (C) – S; (D) – QR]

MATHS
SECTION - I
Single Correct Answer Type
This section contains 5 multiple choice questions. Each question has four choices (A), (B), (C) and (D) out of which
ONLY ONE is correct.

Q.1 If 0 ≤ ≤ then simplify 2 + 2 + √2 + 2 cos 32


(A) 2 cos (B) 2 cos 2 (C) 2 cos 4 (D) 2 cos 8
Solution: (C)

BT-IIT |Camp| FC Rd| PaudRoad|Wanowrie|Aundh|VimanNagar|Deccan|SataraRd|Pimple Saudagar|PCMC|www.bakliwaltutorialsiit.com 18


BT Tests Bakliwal Tutorials - IIT
Q.2 If , are the roots of + + = 0 + + = 0 and if ( / ), ( / ) are the
roots of + 1 + ( + 1) = 0, then (∈ )
(A) must be an odd integer (B) may be any integer
(C) must be an even integer (D) cannot say anything
Solution: (C)

Q.3 The least value of 18 sin + 2 − 3 (wherever defined) is


(A) −15 (B) −12 (C) 0 (D) 9
Solution: (D)

Q.4 The number of solutions lying in ≤ 0 for 10 sin = is /are


(A) Seven (B) Two (C) Three (D) Four
Solution: (D)

Q.5 The value of ∑ ∑ ∑ (1) = 220, then the value of n equals


(A) 11 (B) 12 (C)10 (D) 9
Solution: (C)

SECTION – II
(Multiple Correct Answer(s) Type)
This section contains 6 multiple choice questions. Each question has four choices (A), (B), (C) and (D)
Out of Which ONE OR MORE may be correct.

Q.6 For the series, = 1+( (1 + 2) + ( 1 + 2 + 3) + (1 + 2 + 3 + 4 ) + ⋯


) ( ) ( )
th th
(A) 7 term is 16 (B) 7 term is 18
(C) sum of first 10 terms is (D) sum of first 10 terms is
Solution: (AC)

BT-IIT |Camp| FC Rd| PaudRoad|Wanowrie|Aundh|VimanNagar|Deccan|SataraRd|Pimple Saudagar|PCMC|www.bakliwaltutorialsiit.com 19


BT Tests Bakliwal Tutorials - IIT

Q.7 The value of x satisfying the equation 2 − 8. 2 = −12 is


(A) 1 + (B) log 6 (C) 1 + log (D) 1
Solution: (AD)

Q.8 If 2 −2 +3 = 0
+ + 2 = 0 and
2 + = 0, a ∈
is a system of linear equation, then choose the correct options
(A) The system has no real solution for every a ∈
(B) The system has at least 1 real solution for every a ∈
(C) For a non trivial solution, a = 4
(D) For a non trivial solution, a = 2
Solution: (BD)

6  x
Q.9 If sin  x   0 and cos    0, then  n  Z 
5  5
(A) = ( − 5) (B) = 6 ( − 1)
(C) = 5 − (D) =5 +
Solution: (CD)

BT-IIT |Camp| FC Rd| PaudRoad|Wanowrie|Aundh|VimanNagar|Deccan|SataraRd|Pimple Saudagar|PCMC|www.bakliwaltutorialsiit.com 20


BT Tests Bakliwal Tutorials - IIT

3 3
Q.10 The equation x 3  x   is satisfied by
4 8
(A) = cos (B) = cos (C) = cos (D) = − sin
Solution: (AB)

Q.11 Which of the following are the solution of the equation


2 sin11x  cos 3x  3 sin 3x  0 ?
n  n 7 
(A) x   ,nZ (B) x   ,nZ
7 84 4 48
n  n 
(C) x   ,nZ (D) x   ,nZ
7 63 4 24
Solution: (AB)

BT-IIT |Camp| FC Rd| PaudRoad|Wanowrie|Aundh|VimanNagar|Deccan|SataraRd|Pimple Saudagar|PCMC|www.bakliwaltutorialsiit.com 21


BT Tests Bakliwal Tutorials - IIT

SECTION -III
(One Integer Value Correct Type)
This section contains 5 questions. Each question, when worked out will result in one integer from 0 to 9. (Both
inclusive)

Q.12 If the cubic equation − 7 + = 0 has three distinct (non zero) roots such that one root is twice
the other, then equals to
Solution: (6)

Q.13 Let ( ) = −9 − 6 + and ( ) = −4 + 4 + . If there exists unique pair of real numbers


(x,y) such that ( ) ( ) = 20, then the value of (6 + 10 ) is
Solution: (3)

Q.14 The number of solutions in [0, 2 ] is/are:

BT-IIT |Camp| FC Rd| PaudRoad|Wanowrie|Aundh|VimanNagar|Deccan|SataraRd|Pimple Saudagar|PCMC|www.bakliwaltutorialsiit.com 22


BT Tests Bakliwal Tutorials - IIT
cos sin cos
− sin cos sin =0
− cos − sin cos
Solution: (2)
Q.15 Suppose that for some angles , the equations sin + cos = and cos + sin =
hold simultaneously. The possible value of a is _________.
Solution: (1)

1  sin 2  cos 2
Q.16 If f     then value of 8 (11 ). (34 ) is________.
2 cos 2
Solution: (4)

SECTION –IV (Match the Column)


This section contains 4 questions. Each question has four statements (A, B, C, and D) given in Column I and four
statements (P, Q, R, S & T) in column II, in which ONE OR MORE may be correct.
Q.17
BT-IIT |Camp| FC Rd| PaudRoad|Wanowrie|Aundh|VimanNagar|Deccan|SataraRd|Pimple Saudagar|PCMC|www.bakliwaltutorialsiit.com 23
BT Tests Bakliwal Tutorials - IIT
Column-I Column-II
(A) The value of the determinant (P) 1
+2 +3
+4 + 6 at x = 1

(B) (Q) -6
7 6
If 2 − 13 = 0

Then sum of all roots is

(C) The value of (R) 0


√6 0 3 + √6
√12 √3 3√2
√18 √2 √27

(D) If ( ) = (S) -2
cos cos sin − sin
cos sin sin cos
sin − cos 0

Solution: [(A) – P; (B) – R; (C) – Q; (D) - P]

BT-IIT |Camp| FC Rd| PaudRoad|Wanowrie|Aundh|VimanNagar|Deccan|SataraRd|Pimple Saudagar|PCMC|www.bakliwaltutorialsiit.com 24


BT Tests Bakliwal Tutorials - IIT

Q.18 Consider − ( − 3) + = 0 ; ( ∈ )
Match the correct set of values of m for which

Column I Column II

(A) Both roots are real & distinct (P) (1, 9)

(B) Both roots are not real (Q) =3

(C) Roots are opposite in sign (R) (−∞, 0)

(D) Roots are equal in magnitude but (S) (−∞, 1) ∪ (9, ∞)


opposite in sign
(T) ∈

Solution: [(A) –S ; (B) –P ; (C) –R ; (D) -T ]

BT-IIT |Camp| FC Rd| PaudRoad|Wanowrie|Aundh|VimanNagar|Deccan|SataraRd|Pimple Saudagar|PCMC|www.bakliwaltutorialsiit.com 25


BT Tests Bakliwal Tutorials - IIT
Q.19
Sr No. Column I Sr No. Column II
(A) If a, b, c are in G.P., then (P) A.P.
log 10 , log 10 , log 10 are in
(B) If = = , then a, b, c, d are in (Q) H.P.

(C) If a, b, c are in A.P., a, x, b are in G.P., b, y, c are (R) G.P.


in G.P., then , , are in
(D) If x, y, z are in G.P., = = , then (S) None of these
log , log , log are in

Solution: (A) – Q; (B) – R; (C) – P; (D) - R]

Q.20 Match the equation with their correct number of real solutions

Sr Column I Sr Column II
No. No.
(A) sin = ; ∈ (P) Infinite
(B) cos = + 2 ; ∈ (Q) 0

BT-IIT |Camp| FC Rd| PaudRoad|Wanowrie|Aundh|VimanNagar|Deccan|SataraRd|Pimple Saudagar|PCMC|www.bakliwaltutorialsiit.com 26


BT Tests Bakliwal Tutorials - IIT
(C) = sin ; > 0 (R) 1

(D) 1 (S) 2
2 sin = ; > 0
2

(T) 3

Solution: [(A) – S; (B) – Q; (C) – R; (D) – P]

BT-IIT |Camp| FC Rd| PaudRoad|Wanowrie|Aundh|VimanNagar|Deccan|SataraRd|Pimple Saudagar|PCMC|www.bakliwaltutorialsiit.com 27


Date: 20th October 2019 Bakliwal Tutorials-IIT
Comprehensive 2021 BTest - 5 YOUR STRATEGIC MOVE

Total Time: - 3.00 hrs Sub: - Physics, Chemistry & Math Max. Marks: -300

Rules & regulations for BTEST


SET-A
1. Immediately fill in the particulars on this page of the Test Booklet with Ball Point Pen only.
2. The Answer Sheet (OMR) is provided separately. Please fill in the particulars carefully with Ball
Point Pen.
3. The test is of 3 hours duration.
4. The Test Booklet consists of 75 questions. The maximum marks are 300.
5. There are three parts in the question paper consisting of Physics, Chemistry and Mathematics having
25 questions in each part. Question pattern of question paper is as follows,
New Mains Pattern
Single Correct Answer Type -
This section contains 20 multiple choice questions. (Marking +4, -1)

Numerical Type - (Marking +4, 0)

These questions will have answers lying between 0 to 99.99 and always with xx.xx form meaning the
decimal point will be always after 2 digits. We will provide bubbling for only 4 digits. Hence if the
answer is 99.99 you need to bubble only 9999. A few more cases are as below.
A. Answer 1.54 then Bubble 0154
B. Answer 1.987 will be 0199 due to rounding off of the last digit.
C. Answer 0.1889 will be 0019 due to rounding off.
D. Answer 32.23 will be 3223.
6. No candidate is allowed to carry any textual material, printed or written, bits of papers, pager, mobile
phone, any electronic device, etc. except the BT ID Card inside the examination room/hall.
7. Rough work is to be done on the additional sheets provided
8. On completion of the test, the candidate must immediately hand over the Answer Sheet to the
Invigilator on duty in the Room/Hall. However, the candidates are allowed to take away this Test
Booklet with them.
9. The CODE for this Booklet is A. Make sure that the CODE is marked properly on the answer sheet.
10. Do not fold or make any stray mark on the Answer Sheet.

BT-IIT |Camp| FC Rd| PaudRoad|Wanowrie|Aundh|VimanNagar|Deccan|SataraRd|Pimple Saudagar|PCMC|www.bakliwaltutorialsiit.com 1


BT Tests Bakliwal Tutorials - IIT

PHYSICS
SECTION – I
Single Correct Answer Type
This section contains 20 multiple choice questions. Each question has four choices (A), (B), (C) and (D)
out of which ONLY ONE is correct.

Q.1 A block of mass 'm' is being pulled up a rough and irregular incline of base length , and height 'h'.
The total path length of incline is S. If '' be the friction co-efficient between block and incline, then
work done by friction is:-

(A) - (B) - ℓ (C) − ℎ (D) Zero


Solution: (B)
mgcos   f
dwf  mg(cos d.l)

wf 
 mg(dx)
wf   mgl

PARAGRAPH FOR QUESTION NO. 2 – 3


A suitcase of mass M is placed on a level conveyor belt at a an airport. The coefficient of static friction
between the suitcase and the conveyor belt is and the coefficient of kinetic friction is , with < .
The conveyor belt moves with constant speed u and at time t = 0 the suitcase is placed on the conveyor with
speed v = 0. At time , after moving a distance , the suitcase catches up with the conveyor belt and starts to
move at speed u with the conveyor belt. Gravity acts downward with acceleration g > o. work done depend
on one’s frame of reference, so be sure to answer the following two parts in the frame of reference of the
airport.

BT-IIT |Camp| FC Rd| PaudRoad|Wanowrie|Aundh|VimanNagar|Deccan|SataraRd|Pimple Saudagar|PCMC|www.bakliwaltutorialsiit.com 2


BT Tests Bakliwal Tutorials - IIT

Q.2 How much work does friction do on the suitcase during this period?
(A) (B) − (C) (D)
Solution: (A)
Alternative: Friction is the only horizontal force, since the displacement is horizontal, friction is the
only contribution to the work. The total work has to be the change in kinetic energy (work-energy
theorem), , so this must be the work done by friction. Note that in this case friction does
positive work, although in most circumstances friction slows things down and does negative work.
Note also that the work done by friction is equal to the force times the distance and hence ,
but this was not one of the choices.

Q.3 How much work does the force of friction from the suitcase do on the belt, during this time period?
(A) (B) − (C) (D) −
Solution: (D)
Alternative:
Tricky, tricky. The suitcase is slipping during this time, but the work on the belt is determined by the
force acting on the belt and the displacement of the belt, as measured in the reference frame of the
airport. The belt is moving at constant speed u in this frame and the time interval is . So the
displacement of the belt is , in the x-direction. The force of friction that the suitcase exerts on the
belt is the negative of the force. Of friction that the belt exerts on the suitcase (Newton’s 3rd law), so
it has magnitude , in the negative x-direction. The work is then = = (− ) .
Q.4 The displacement of a body of mass 2 kg varies with time t as = + 2 , where x is in meters and
t is in seconds. The work done by all the forces acting on the body during the time interval [2s, 4s] is:
(A) 36 J (B) 64 J (C) 100 J (D) 120 J
Solution: (B)

Q.5 A car of mass m is accelerating on a level smooth road under the action of a single force F. The
power delivered to the car is constant equal to P. If the velocity of the car at an instant is v, then after
travelling how much distance it becomes double?

BT-IIT |Camp| FC Rd| PaudRoad|Wanowrie|Aundh|VimanNagar|Deccan|SataraRd|Pimple Saudagar|PCMC|www.bakliwaltutorialsiit.com 3


BT Tests Bakliwal Tutorials - IIT

(A) (B) (C) (D)


Solution: (A)
P = Fv = constant
∴ =
∴ =
∴ ∫ = ∫
Solving we get, =
Q.6 System shown in the figure is released from rest when spring is unstretched. Pulley and spring is
massless and friction is absent everywhere. The speed of 5 kg block when 2 kg block leaves the
contact with ground is: (Take force constant of spring k = 40N/m AND = 10 / )

(A)√2 / (B) 2√2 / (C) 2 m/s (D) 4√2 m/s


Solution: (B)

= 2√2 /
Q.7 A block of mass m is taken from A to B under the action of a constant force F. Work done by this
force is

BT-IIT |Camp| FC Rd| PaudRoad|Wanowrie|Aundh|VimanNagar|Deccan|SataraRd|Pimple Saudagar|PCMC|www.bakliwaltutorialsiit.com 4


BT Tests Bakliwal Tutorials - IIT
(A) (B) (C) Zero (D) FR
Solution: (D)
w  FR  force  const.horizontal disp  R 

Q.8 A block of mass ‘m’ is released from rest at point A. The compression in spring, when the speed of
block is maximum

(A) (B) (C) (D)


Solution: (C)

Q.9 A body is displaced from (0, 0) to (1m, 1m) along the path x = y by a force
⃗ = ( ̂ + ̂)N. The work done by this force will be:
(A) (B) (C) (D)
Solution: (B)

F  x 2 j  yj  y 2 j  xi
1 1
 2
F  dx 
 x dx   y dy  5 / 6
0 0
Q.10 A cannon ball of mass m is fired with an initial velocity ⃗ = ̂+ ̂, which makes an angle
= tan / with respect to the horizontal. What is the work done by gravity on the cannon
ball till it reaches the peak (i.e. highest elevation) of its trajectory?
(A) (B) (C) − (D) −
Solution: (C)

Q.11 A particle acted upon by two constant forces 4 ̂ + ̂ − 3 and 3 ̂ + ̂ − is displaced from the point
̂ + 2 ̂ + 3 to the point 5 ̂ + 4 ̂ + . The total work done by the forces is SI unit is:
(A)20 (B) 40 (C) 50 (D) 30

Solution: (B)

BT-IIT |Camp| FC Rd| PaudRoad|Wanowrie|Aundh|VimanNagar|Deccan|SataraRd|Pimple Saudagar|PCMC|www.bakliwaltutorialsiit.com 5


BT Tests Bakliwal Tutorials - IIT
 
w  F.ds
   
 
 F1  F2 . S1  S2 
  7iˆ  2ˆj  4kˆ  4iˆ  2ˆj  2kˆ 
 28  4  8
 40J
Q.12 In figure, a sphere suspended from the ceiling starts from rest from position (1) and ends its upward
swing at position (2). Ignoring friction and air resistance, the correct relation among the following is:

(A) cos = (B) sin =


(C) cos = (D) sin =
Solution: (C)

Q.13 A particle is placed at the origin and a force F = kx is acting on it (where k is a positive constant). If
U (0) = 0, the graph of U (x) versus x will be (where, U is the potential energy function)

Solution: (A)
From =−
( )
∫ = −∫ = −∫ ( )
( )=− as U (0) = 0
Therefore, the correct option is (A)

Q.14 A car of mass m is moving on the concave circular path while another car of mass = m/2 is moving
on the convex path of radius = 2r, where r is the radius of the concave path with same speed. Ratio of
their normal reaction at the lowest and highest points of their paths respectively is: (where u is the
speed of the car)

BT-IIT |Camp| FC Rd| PaudRoad|Wanowrie|Aundh|VimanNagar|Deccan|SataraRd|Pimple Saudagar|PCMC|www.bakliwaltutorialsiit.com 6


BT Tests Bakliwal Tutorials - IIT

(A) (B) 2 (C) (D)

Solution: (D)
Q.15 A particle is rotated in a vertical circle by connecting it to a light rod of length l and keeping the
other end of the rod fixed. The minimum speed of the particle (when the light rod is horizontal) for
which the particle will complete the circle is:
(A) (B) 2 (C) 3 (D) None of these
Solution: (B)

Q.16 Two atoms interact with each other according to the following force F and potential energy V
diagrams. What is their equilibrium separation?

(A) The separation u which is equal to y (B) the separation u which is equal to z
(C) the separation w which is equal to y (D) the separation w which is equal to z
Solution: (B)
For equilibrium F = 0 and from F – x graph it is clear at = 4 = 0
From V – x graph
It is clear that = = 0 at x = z
Q.17 The potential energy of a 1 kg particle free to move along the x-axis is given by:
( )= −
The total mechanical energy of the particle is 2 J. Then, the maximum speed (in m/s) is:
(A) (B) 3√2 (C) (D) 2

Solution: (A)

BT-IIT |Camp| FC Rd| PaudRoad|Wanowrie|Aundh|VimanNagar|Deccan|SataraRd|Pimple Saudagar|PCMC|www.bakliwaltutorialsiit.com 7


BT Tests Bakliwal Tutorials - IIT

Q.18 Eight solid uniform cubes of edge l are stacked together to form a single cube with center O. One
cube is removed from this system. Distance of the centre of mass of remaining 7 cubes from O is

√ √ √
(A) (B) (C) (D) Zero
Solution: (C)

Q.19 If a particle of mass is located at (x, y, z) = (0, a, 0) and a second particle of mass is located
(x, y, z) = (b, c, 0), what is the location of their center of mass?
(A) , ,0 (B) , ,0
(C) , ,0 (D) None of the above
Solution: (B)

BT-IIT |Camp| FC Rd| PaudRoad|Wanowrie|Aundh|VimanNagar|Deccan|SataraRd|Pimple Saudagar|PCMC|www.bakliwaltutorialsiit.com 8


BT Tests Bakliwal Tutorials - IIT

Q.20 A uniform solid right circular cone of base radius R is joined to a uniform solid hemisphere of radius
R and of the same density, as shown. The centre of mass of the composite solid lies at the centre of
base of the cone. The height of the cone is:

(A) 1.5R (B) √3 (C) 3 R (D) 2√3


Solution: (B)

SECTION -II
(Numerical Type)
This section contains 5 questions.

Q.21 A block is released at A and slides on smooth surface in shape of quarter circle. The horizontal part
is rough. If the block comes to rest 10 m away from B and the coefficient of kinetic friction is n/10,
then the value of n is

BT-IIT |Camp| FC Rd| PaudRoad|Wanowrie|Aundh|VimanNagar|Deccan|SataraRd|Pimple Saudagar|PCMC|www.bakliwaltutorialsiit.com 9


BT Tests Bakliwal Tutorials - IIT

Solution: (03.00)

Q.22 Consider the situation shown in figure. The system is released from rest and the block of mass 2 kg is
found to have a speed 2.0 m/s after it has descended through a distance of 1 m. the coefficient of
kinetic friction between the block and the table is N/10. Then the value of N is ( = 10 / )

Solution: (04.00)

Q.23 An electric motor produces a tension of 4500 N in a load lifting cable and rolls it at the rate of 2 m/s.
Evaluate the power of the motor (in Kilo Watt).
Solution: (09.00)
P = F.V = 4500 x 2 = 9 kw

Q.24 A force of 0.5 N is applied on upper block as shown. The work done by lower block on upper block
for a displacement 3 m of the upper block is . Find K

BT-IIT |Camp| FC Rd| PaudRoad|Wanowrie|Aundh|VimanNagar|Deccan|SataraRd|Pimple Saudagar|PCMC|www.bakliwaltutorialsiit.com 10


BT Tests Bakliwal Tutorials - IIT

Solution: (02.00)
F 1
Fmax  mg  1N;a   m / s2
1 2 6
1
For lower block F  2a  N
3
 Friction force acting is 1/3N
w.d. by lower block 
  F.d
on upper block is 
 1
    (3)
 3
 1J
Q.25 A light string of length = 1 m is fixed at point A on a fixed cylinder of radius = ( /2 ) m. The
other end of the string is attached to a small ball of mass 1 kg. The ball is imparted a velocity =
10 / horizontally. The tension in the string when the ball is at the lowest point is (30 x) N. Find
the value of x. ( = 10 / )

Solution: (09.00)

BT-IIT |Camp| FC Rd| PaudRoad|Wanowrie|Aundh|VimanNagar|Deccan|SataraRd|Pimple Saudagar|PCMC|www.bakliwaltutorialsiit.com 11


BT Tests Bakliwal Tutorials - IIT

CHEMISTRY
SECTION – I
Single Correct Answer Type
This section contains 20 multiple choice questions. Each question has four choices (A), (B), (C) and (D)
out of which ONLY ONE is correct.

Q.1 Which of the following curves correctly represents the speed of electron in H-atom as a function of
principal quantum number:

(A) 4 (B) 2 (C) 3 (D) 1


Solution: (D)

BT-IIT |Camp| FC Rd| PaudRoad|Wanowrie|Aundh|VimanNagar|Deccan|SataraRd|Pimple Saudagar|PCMC|www.bakliwaltutorialsiit.com 12


BT Tests Bakliwal Tutorials - IIT
Q.2 Consider an electron in the orbit of a hydrogen atom in the Bohr model. The circumference
of the orbit can be expressed in terms of the de Broglie wavelength of the electron as:
(A) (0.529) (B) √ (C) (13.6) (D)
Solution: (D)

Q.3 A flask containing 250mg of air at 27° is heated till 25% of air by mass is expelled from it. What is
the final temperature of the flask
(A) 127K (B) 127° (C) 254K (D) 254°
Solution: (B)

Q.4 100 of 0.2N HCl solution is added to 100 of 0.2N solution. The molarity of nitrate
ions in the resulting mixture will be
(A) 0.5 M (B) 0.05 M (C) 0.1 M (D) 0.2 M
Solution: (C)
× .
in the mixture = = 0.1

Q.5 reacts with KI in basic medium to form and . When 250 of 0.1 M KI solution is
mixed with 250 of 0.02 M in neutral medium, what is the number of moles formed?
(A) 0.0075 (B) 0.005 (C) 0.01 (D) 0.015
Solution: (A)

BT-IIT |Camp| FC Rd| PaudRoad|Wanowrie|Aundh|VimanNagar|Deccan|SataraRd|Pimple Saudagar|PCMC|www.bakliwaltutorialsiit.com 13


BT Tests Bakliwal Tutorials - IIT

Q.6 If insulin contains 3.4% Sulphur, what will be the minimum molecular mass of insulin?
(A) 941.17 (B) 640.175 (C) 438.172 (D) 245.170
Solution: (A)
For minimum mol. Mass, insulin must contain atleast one Sulphur atom in its one molecule. Because
when 3.4g Sulphur is present then mol. Mass of insulin = 100. 32g Sulphur is present then molecular
×
mass of insulin = = 941.176
.

Q.7 Which of the following statements is incorrect:


(A) 0.2 moles of will oxidise one mole of ferrous ions to ferric ions in acidic medium.
(B) 1.5 moles of will oxidise 1 mole of ferrous oxalate to one mole of ferric ion and carbon
dioxide in acidic medium in acidic medium.
(C) 0.6 moles of will oxidise 1 mole of ferrous oxalate to one mole of ferric ion and carbon
dioxide in acidic medium.
(D) 1 mole of will oxidise 2 moles of ferrous oxalate to ferric ions and carbon dioxide in
acidic medium.
Solution: (B)

BT-IIT |Camp| FC Rd| PaudRoad|Wanowrie|Aundh|VimanNagar|Deccan|SataraRd|Pimple Saudagar|PCMC|www.bakliwaltutorialsiit.com 14


BT Tests Bakliwal Tutorials - IIT

Q.8 Calculate the % of free in oleum (a solution of in ) that is labeled 109% by


mass
(A) 98% (B) 109% (C) 40% (D) 80%
Solution: (C)

Q.9 The normality of 30 volumes solution is


(A) 3.57 (B) 7.53 (C) 5.36 (D) 5.73
Solution: (C)
30 = × 5.6 = . = 5.36

Q.10 1.2 g of carbon is burnt completely in oxygen (limited supply) to produce CO and . This mixture
of gases is treated with solid (to know the amount of CO produced). The liberated iodine
required 120 of 0.1 M hypo solution for complete titration. The % of carbon converted into CO
is:
(A) 60% (B) 100% (C) 50% (D) 30%
Solution: (D)

BT-IIT |Camp| FC Rd| PaudRoad|Wanowrie|Aundh|VimanNagar|Deccan|SataraRd|Pimple Saudagar|PCMC|www.bakliwaltutorialsiit.com 15


BT Tests Bakliwal Tutorials - IIT

Q.11 20 mL of x M HCl is added in 10mL 0.1M NaHCO solution and a further 5mL of 0.2M Na CO
solution is required to neutralize the remaining acid, using methyl orange indicator. The value of x is
(A) 0.167M (B) 0.133M (C) 0.15M (D) 0.2M
Solution: (C)
m.eqt of HCl = m.eqt of ( + ) 20 = (10 × 0.1) + (5 × 0.2 × 2)
∴ = 0.15

Q.12 Match the following: Identify equivalent weight of underlined species from redox reactions
Column I Column II
(A) → + (P) =
(B) → + (Q) =
(C) + + → + (R) =
(D) → + (S) =
The correct answer is
(A) → ; → ; → ; → (B) → ; → ; → ; →
(C) → ; → ; → ; → (D) → ; → ; → ; →
Solution: (D)

BT-IIT |Camp| FC Rd| PaudRoad|Wanowrie|Aundh|VimanNagar|Deccan|SataraRd|Pimple Saudagar|PCMC|www.bakliwaltutorialsiit.com 16


BT Tests Bakliwal Tutorials - IIT

Q.13 For the reaction ( )⇌ ( )+ ( ), the pressure of ( ) depends on


(A) The mass of ( ) (B) The mass of ( )
(C) The masses of both ( ) and ( ) (D) Temperature of the system
Solution: (D)
=
In heterogeneous system equilibrium is unaffected by small amount addition of solid or liquid
Q.14 For the reaction + ⇌2 equilibrium constant = 2. Degrees of dissociation of and
are
(A) , (B) , (C) Both are (D) ,
√ √ √ √ √ √ √
Solution: (C)

Q.15 Consider the reaction equilibrium


2 ( )+ ( ) ⇌2 ( ) , ∆ ° = −198
On the basis of Le chatelier’s principle, the condition favourable for the forward reaction is
(A) Lowering of temperature as well as pressure

BT-IIT |Camp| FC Rd| PaudRoad|Wanowrie|Aundh|VimanNagar|Deccan|SataraRd|Pimple Saudagar|PCMC|www.bakliwaltutorialsiit.com 17


BT Tests Bakliwal Tutorials - IIT
(B) Increasing temperature as well as pressure
(C) Lowering the temperature and increasing the pressure
(D) Any value of temperature and pressure
Solution: (C)
As ∆ < and ∆ < hence reaction is favourable at high pressure and low temperature.
Q.16 When a mixture of and in the volume ratio of 1 : 5 is allowed to react at 700 K and 10 atm
pressure, 0.4 mole fraction of is formed at equilibrium. The for the reaction
( )+3 ( ) ⇌ 2 ( )
(A) 2.6 × 10 (B) 2.6 × 10
(C) 2.6 × 10 (D) 5.1 × 10
Solution: (A)
Conceptual
Q.17 In the system, ( )+ ( )+ℎ ⇌ ( )+2 ( )
Equilibrium is established. More water vapour is added to disturb the equilibrium. If the pressure of
water vapour at new equilibrium is double of that at initial equilibrium, the factor to which pressure
of HCl is changed is
(A) 2 times (B) √2 times (C) (D) 4 times

Solution: (B)

Conceptual
Q.18 (molecular mass = M) dissociates into and as: ( )⇌ ( )+ ( ). If the
total pressure of the system at equilibrium is P and the density is ‘d’ at temperature, T K. The degree
of dissociation of may be represented as
(A) (B) −1 (C) −1 (D)
Solution: (B)

BT-IIT |Camp| FC Rd| PaudRoad|Wanowrie|Aundh|VimanNagar|Deccan|SataraRd|Pimple Saudagar|PCMC|www.bakliwaltutorialsiit.com 18


BT Tests Bakliwal Tutorials - IIT

Q.19 The preparation of ammonia by Haber’s process is an exothermic reaction. If the preparation follows
the following temperature pressure relationship for its % yield. Then the temperature , and ,
the correct option is:

(A) > > (B) > >


(C) = = (D) Nothing could be predicted
Solution: (B)

Q.20 Which of the following statements is correct?


(A) If be the equilibrium constant for ⇌ and be the equilibrium constant for ⇌ then
equilibrium constant for the reaction

BT-IIT |Camp| FC Rd| PaudRoad|Wanowrie|Aundh|VimanNagar|Deccan|SataraRd|Pimple Saudagar|PCMC|www.bakliwaltutorialsiit.com 19


BT Tests Bakliwal Tutorials - IIT
+ ⇌ +
(B) If be the equilibrium constant for ⇌ and be the equilibrium constant for ⇌ then
equilibrium constant for the reaction
+ ⇌ + is
(C) If be the equilibrium constant for + ⇌ + the equilibrium constant for
+ ⇌ + is ( ) / , can be fraction also
(D) The equilibrium constant unit for the gaseous reaction + ⇌ is
Solution: (A)
i.e. + ⇌ +

SECTION -II
(Numerical Type)
This section contains 5 questions.

Q.21 Certain amount of is taken in a closed container to establish the equilibrium


⇌ + , 25% of is found to be dissociated at equilibrium. If total pressure at
equilibrium is 30 atm then find for the reaction
Solution: (2.00)

Q.22 100 of each of 1 and 11.2 V solutions are mixed, then the normality of resulting
solution will be
Solution: (25.5)
.
11.2 = . . 2
+ = ( + )
1 × 100 + 2 × 100 = × 200
= 1.5

Q.23 In how many acids among the following, the basicity is greater than 1?
H PO , H PO , H PO , H BO
Solution: (2.00)
H PO Basicity = 3
H PO Basicity = 2
H PO Basicity = 1
H BO Basicity = 1

Q.24 3.2 g of oxygen and 0.2 g of hydrogen are placed in 1.12 litre flask at 0° . The total pressure in atm
of the gas mixture will be
Solution: (4.00)

BT-IIT |Camp| FC Rd| PaudRoad|Wanowrie|Aundh|VimanNagar|Deccan|SataraRd|Pimple Saudagar|PCMC|www.bakliwaltutorialsiit.com 20


BT Tests Bakliwal Tutorials - IIT
. ×
=( + ) = (0.1 + 0.1)
.

Q.25 Phosphorous pentachloride when heated in a sealed tube at 700 K, it undergoes decomposition as,
( )⇌ ( )+ ( ); = 38
Vapour density of the mixture is 74.25. Find the percentage dissociation of PCl5
Solution: (40.40)
MATH
SECTION – I
Single Correct Answer Type
This section contains 20 multiple choice questions. Each question has four choices (A), (B), (C) and (D)
out of which ONLY ONE is correct.

Q.1 The image of the point A (1, 2) by the line mirror y = x is the point B and the image of B by the line
mirror y = 0 is the point ( , )
(A) = 1 , = −2 (B) = 0 , = 0
(C) = 2 , = −1 (D) = 1 , = −1
Solution: (C)

Q.2 Let ABC be a triangle right angled at C. The value of


log b c a  logc  b a
 b  c  1, c  b  1 equals (where a, b, c denotes the sides of ∆ opposite
log b c a.log c b a
angle A, B, C respectively):
(A) 1 (B) 2 (C) 3 (D)
Solution: (B)

Q.3 The orthocenter of the triangle ABC is ‘B’ and the circumcenter is ‘S’ (a, b). If A is the origin, then
the co-ordinates of C are:
(A) (2 , 2 ) (B) , (C) √ + , 0 (D) None
Solution: (A)

BT-IIT |Camp| FC Rd| PaudRoad|Wanowrie|Aundh|VimanNagar|Deccan|SataraRd|Pimple Saudagar|PCMC|www.bakliwaltutorialsiit.com 21


BT Tests Bakliwal Tutorials - IIT
Q.4 The set of values of satisfying the inequation 2 sin − 5 sin + 2 > 0, where 0 < <2 ,
is:
(A) 0, ∪ ,2 (B) 0, ∪ ,2
(C) 0, ∪ ,2 (D) None of these
Solution: (A)

Q.5 The distance of the point (2, 3) from the line 2 − 3 + 9 = 0 measured along a line − +1=0
is:
(A) 5√3 (B) 4√2 (C) 3√2 (D) 2√2
Solution: (B)

Q.6 The equation of the internal bisector of ∠ of ∆ with vertices A(5, 2), B(2, 3) and C(6, 5) is
(A) 2 + + 12 = 0 (B) + 2 − 12 = 0
(C) 2 + − 12 = 0 (D) 2 − − 12 = 0
Solution: (C)

Q.7 The line joining two points A(2, 0) and B(3, 1) is rotated about A in the anticlockwise direction
through an angle of 15°. The equation of the line in the new position is:
(A) − √3 − 2 = 0 (B) − 2 − 2 = 0
(C) √3 − − 2√3 = 0 (D) √2 − − 2√2 = 0
Solution: (C)

Q.8 The variable straight line 3 ( + 1) + 4 ( − 1) − 3 ( − 1) = 0 for different values of ‘a’ passes
through a fixed point whose coordinates are:
(A) (0, 1) (B) (1, 2) (C) 0, (D) (2, 1)
Solution: (C)

BT-IIT |Camp| FC Rd| PaudRoad|Wanowrie|Aundh|VimanNagar|Deccan|SataraRd|Pimple Saudagar|PCMC|www.bakliwaltutorialsiit.com 22


BT Tests Bakliwal Tutorials - IIT

Q.9 A variable straight line passes through a fixed point (a, b) intersecting the co-ordinates axes at A and
B. If O is the origin then the locus of the centroid of the triangle OAB is:
(A) + −3 =0 (B) + −2 = 0
(C) + −3 = 0 (D) None of these
Solution: (A)

Q.10 If + = 0 ; + = 0 ; + = 0 then real value of a for which system of equations will


have infinite number of solutions, is :
(A) -1 (B) 1 (C) 0 (D) 2
Solution: (A)

Q.11 A circle has a radius of log ( ) and a circumference of log ( ). The value of log is equal to:
(A) (B) (C) (D) 2
Solution: (C)

Q.12 If and are the roots of the equation − + 11 = 0 then the value of
BT-IIT |Camp| FC Rd| PaudRoad|Wanowrie|Aundh|VimanNagar|Deccan|SataraRd|Pimple Saudagar|PCMC|www.bakliwaltutorialsiit.com 23
BT Tests Bakliwal Tutorials - IIT
3 −3 +2 − 2 + 11 is:
(A) 33 (B) -33 (C) 22 (D) -22
Solution: (D)

Q.13 The line through point (m, -9) and (7, m) has slope m. The y-intercept of this line, is:
(A) -18 (B) -6 (C) 6 (D) 18
Solution: (A)

Q.14 Let a and b be two distinct roots of the equation + 3 − 1 = 0. The equation which has (ab) as
its root is equal to:
(A) −3 −1= 0 (B) −3 +1=0
(C) + −3 +1 = 0 (D) + +3 −1 = 0
Solution: (A)

Q.15 In what ratio the line − + 2 = 0 divides the line joining the points (3, -1) and (8, 9)?
(A) 1 : 2 (B) 2 : 1 (C) 2 : 3 (D) 3 : 4
Solution: (C)
  1  3  2  2
ratio= 
9 8  2 3

Q.16 If the roots of − + = 0 are sin and cos , then equals:


(A) (B) 1 + 2 (C) 1 + (D) 1 +
BT-IIT |Camp| FC Rd| PaudRoad|Wanowrie|Aundh|VimanNagar|Deccan|SataraRd|Pimple Saudagar|PCMC|www.bakliwaltutorialsiit.com 24
BT Tests Bakliwal Tutorials - IIT
Solution: (B)

Q.17 Angle between the lines represented by the equation +2 + = 0 is


(A) (B) 2 (C) (D) None of these
Solution: (A)

Q.18 Which value of listed below leads to:


2 > 1 3 < 1?
(A) 70° (B) 140° (C) 210° (D) 280°
Solution: (B)

19. The value of log 2  cos 2       cos 2       cos 2 cos   :


(A) depends on  and  both (B) depends on but not on
(C) depends on but not on (D) independent of both and
Solution: (D)

20. If , and are the roots of the equation + + = 0, then the value of the determinant
  
   
  
(A) (B) (C) −2 (D) None of these
Solution: (D)

BT-IIT |Camp| FC Rd| PaudRoad|Wanowrie|Aundh|VimanNagar|Deccan|SataraRd|Pimple Saudagar|PCMC|www.bakliwaltutorialsiit.com 25


BT Tests Bakliwal Tutorials - IIT

SECTION -II
(Numerical Type)
This section contains 5 questions.

Q.21 Find the area of the triangle formed by the mid points of sides of the triangle whose vertices are
(2, 1), (-2, 3), (4, -3)
Solution: (01.50)

+3 −1 +3
Q.22 If + + + + ≡ +1 2− − 3 then t is:
−3 +4 3
Solution: (21.00)
Put = 0 and then evaluate.
4 2 −
Q.23 Let = and suppose that det (A) = 2 then the det (B) equals, where = 4 2 −
4 2 −
( )
Then value of
Solution: (00.25)

24. Let 0  1  2  3  ... denote the positive solution of the equation


3 + 3 cos = 2 sin . If + = , where is an integer then equals:
Solution: (6.00)

BT-IIT |Camp| FC Rd| PaudRoad|Wanowrie|Aundh|VimanNagar|Deccan|SataraRd|Pimple Saudagar|PCMC|www.bakliwaltutorialsiit.com 26


BT Tests Bakliwal Tutorials - IIT

25. In any triangle, if (sin + sin + sin )(sin + sin − sin ) = 3 sin sin , Find the angle C
(in degree).
Solution: (60.00)

BT-IIT |Camp| FC Rd| PaudRoad|Wanowrie|Aundh|VimanNagar|Deccan|SataraRd|Pimple Saudagar|PCMC|www.bakliwaltutorialsiit.com 27


Date: 17th November 2019 Bakliwal Tutorials-IIT
Comprehensive 2021 BTest - 6 YOUR STRATEGIC MOVE

Total Time: - 3.00 hrs Sub: - Physics, Chemistry & Math Max. Marks: -210

SET-A
Rules & regulations for BTEST

1. Immediately fill in the particulars on this page of the Test Booklet with Ball Point Pen only.
2. The Answer Sheet (OMR) is provided separately. Please fill in the particulars carefully with Ball Point
Pen.
3. The test is of 3 hours duration.
4. The Test Booklet consists of 60 questions. The maximum marks are 210.
5. There are three parts in the question paper consisting of Physics, Chemistry and Mathematics having 20
questions in each part. Question pattern of question paper is as follows,
Advance Pattern
Multiple Correct Answer Type –
This section contains 10 multiple choice questions. (Marking +3, -1)

Numeric Type -
This section contains 8 multiple choice questions. (Marking +4, 0)

Matrix Answer Type –


This section contains 2 questions. (Marking +4, 0) (Partial Marking)
Each question has four statements (A, B C and D) given in Column I and four or five statements (P, Q, R,
S and T) in column II, in which ONE OR MORE may be correct.

These questions will have answers lying between 0 to 99.99 and always with xx.xx form meaning the
decimal point will be always after 2 digits. We will provide bubbling for only 4 digits. Hence if the
answer is 99.99 you need to bubble only 9999. A few more cases are as below.
A. Answer 1.54 then Bubble 0154
B. Answer 1.987 will be 0199 due to rounding off of the last digit.
C. Answer 0.1889 will be 0019 due to rounding off.
D. Answer 32.23 will be 3223.
6. No candidate is allowed to carry any textual material, printed or written, bits of papers, pager, mobile
phone, any electronic device, etc. except the BT ID Card inside the examination room/hall.
7. Rough work is to be done on the additional sheets provided
8. On completion of the test, the candidate must immediately hand over the Answer Sheet to the Invigilator
on duty in the Room/Hall. However, the candidates are allowed to take away this Test Booklet with them.
9. The CODE for this Booklet is A. Make sure that the CODE is marked properly on the answer sheet.
10. Do not fold or make any stray mark on the Answer Sheet.

BT-IIT |Camp| FC Rd| PaudRoad|Wanowrie|Aundh|VimanNagar|Deccan|SataraRd|Pimple Saudagar|PCMC|www.bakliwaltutorialsiit.com 1


BT Tests Bakliwal Tutorials - IIT

PHYSICS
SECTION – I
(Multiple Correct Answer(s) Type)
This section contains 10 multiple choice questions. Each question has four choices (A), (B), (C) and (D)
Out of Which ONE OR MORE may be correct.

Q.1 A ball of mass 1 kg is thrown up with an initial speed of 4 m/s. A second ball of mass 2 kg is
released from rest from some height as shown in Figure. Choose the correct statement(s).

(A) The centre of mass of the two balls comes down with acceleration /3
(B) The centre of mass first moves up and then comes down
(C) The acceleration of the centre of mass is g downwards
(D) The centre of mass of the two balls remains stationary.
Solution: (BC)
The initial velocity of CM is upward. The acceleration of the CM is ‘g’ downward.

Q.2 In an elastic collision between two particles


(A) The total kinetic energy of the system is always conserved
(B) The kinetic energy of the system before collision is equal to the kinetic energy of the system
after collision
(C) The linear momentum of the system is conserved
(D) The mechanical energy of the system before collision is equal to the mechanical energy of
the system after collision.
Solution: (BCD)
In an elastic collision, linear momentum of the system is always conserved. i.e., in all the three
stages of collision. But the kinetic energy of the system is not always conserved. It is conserved
only before and after the collision. During collision it gets converted into potential energy.

Q.3 A body of mass 2 kg moving with a velocity 3 m/s collides with a body of mass 1 kg moving
with a velocity of 4 m/s in opposite direction. If the collision is head on and completely inelastic,
then
(A) Both particles move together with velocity (2/3) m/s
(B) The momentum of system is 2 kg m/s throughout
(C) The momentum of system is 10 kg m/s
(D) The loss of KE of system is (49/3) J
Solution: (ABD)

BT-IIT |Camp| FC Rd| PaudRoad|Wanowrie|Aundh|VimanNagar|Deccan|SataraRd|Pimple Saudagar|PCMC|www.bakliwaltutorialsiit.com 2


BT Tests Bakliwal Tutorials - IIT

Q.4 A block of mass m moving with a velocity collides with a stationary block of mass M to
which a spring of stiffness k is attached, as shown in Figure. Choose the correct alternative(s).

(A) The velocity of the centre of mass is


(B) The initial kinetic energy of the system in the centre of mass frame is

(C) The maximum compression in the spring is


(D) When the spring is in the state of maximum compression, the kinetic energy in the centre of
mass frame is zero.
Solution: (CD)

Q.5 A ball of mass 1 kg is dropped from a height of 3.2 m on fixed smooth inclined plane. The
coefficient of restitution for the collision is = 1/2. The ball’s velocity become horizontal after
the collision.

BT-IIT |Camp| FC Rd| PaudRoad|Wanowrie|Aundh|VimanNagar|Deccan|SataraRd|Pimple Saudagar|PCMC|www.bakliwaltutorialsiit.com 3


BT Tests Bakliwal Tutorials - IIT

(A) The angle = tan



(B) The speed of the ball after the collision = 4√2
(C) The total loss in kinetic energy during the collision is 8 J
(D) The ball hits the inclined plane again while travelling vertically downward.
Solution: (AC)

Q.6 A strip of wood of mass M and length is placed on a smooth horizontal surface. An insect of
mass m starts at one end of the strip and walks to the other end in time t, moving with a constant
speed:
(A) The speed of the insect as seen from the ground is <
(B) The speed of the strip as seen from the ground is
(C) The speed of the strip as seen from the ground is

(D) The total kinetic energy of the system is ( + )


Solution: (AC)

BT-IIT |Camp| FC Rd| PaudRoad|Wanowrie|Aundh|VimanNagar|Deccan|SataraRd|Pimple Saudagar|PCMC|www.bakliwaltutorialsiit.com 4


BT Tests Bakliwal Tutorials - IIT

Q.7 Two identical buggies move one after other due to inetia (without friction) with the same
velocity . A man of mass m rides the rear buggy. At a certain moment the man jumps into the
front buggy with a velocity u relative to his buggy (after the state of jump). If mass of each
buggy is equal to M and velocity of buggies after jumping of man are and . Then:
(A) = + (B) = −
(C) = +( )
(D) = −( )
Solution: (BC)

Q.8 A pendulum bob of mass m connected to the end of an ideal string of length is released from
rest from horizontal position as shown in the figure. At the lowest point the bob makes an elastic
BT-IIT |Camp| FC Rd| PaudRoad|Wanowrie|Aundh|VimanNagar|Deccan|SataraRd|Pimple Saudagar|PCMC|www.bakliwaltutorialsiit.com 5
BT Tests Bakliwal Tutorials - IIT
collision with a stationary block of mass 5m, which is kept on a frictionless surface. Mark out
the correct statement(s) for the instant just after the impact.

(A) Tension in the string is


(B) Tension in the string is 3 mg
(C) The velocity of the block is
(D) The maximum height attained by the pendulum bob after impact is (measured from the
lowest position)
Solution: (ACD)

Q.9 A ball of mass m is projected from a point P on the ground, as shown in the figure. It was
supposed to have a horizontal range R. However, it hits a fixed vertical wall elastically at a
distance from P. Which of the following are correct?

(A) The ball will return to the point P directly if = /2


(B) The ball will fall between point P and the wall if < /2
(C) The ball will fall between point P and the wall if > /2
(D) If < /2, the ball spends more time in air after collision than before it
Solution: (ACD)
BT-IIT |Camp| FC Rd| PaudRoad|Wanowrie|Aundh|VimanNagar|Deccan|SataraRd|Pimple Saudagar|PCMC|www.bakliwaltutorialsiit.com 6
BT Tests Bakliwal Tutorials - IIT

Q.10 A small particle of mass = 2 is projected with speed = 30 / at an angle = 37


with horizontal in x-y plane. Then which of the following options are correct? (take =
10 / )
(A) Magnitude of torque of gravitational force about point of projection after 2 sec is 960 −
.
(B) Magnitude of torque of gravitational force about highest point of motion after 2 sec is 96 N-
M
(C) Magnitude of torque of gravitational force about x axis and y axis is always zero during the
motion.
(D) Magnitudes of torque of gravitational force about highest point of motion first decreases then
increases.
Solution: (ABCD)

SECTION -II
(Numerical Type)
This section contains 8 Numeric Type questions.

Q.11 N beads identical beads are resting on a smooth horizontal wire which is circular at the end with
radius r = 0.5 m as shown in the figure. Find the minimum velocity which should be imparted to

BT-IIT |Camp| FC Rd| PaudRoad|Wanowrie|Aundh|VimanNagar|Deccan|SataraRd|Pimple Saudagar|PCMC|www.bakliwaltutorialsiit.com 7


BT Tests Bakliwal Tutorials - IIT
the first bead such that bead will fall in the tank after completing full circle in vertical pane
as shown in the figure. Take all the collisions between the beads elastic (e = 1)

Solution: (5)

Q.12 A boy of mass 60 kg is standing over a platform of mass 40 kg placed over a smooth horizontal
surface. He throws a stone of mass 1 kg with velocity v = 10 m/s at an angle of 45° with respect
to the ground. Find the displacement of the platform (with boy) on the horizontal surface (in cm)
when the stone lands on the ground. Take = 10 /
Solution: (10)

Q.13 A particle of mass 2m is projected at an angle of 45° with horizontal with a velocity of 20√2
m/s. After 1 s explosion takes place and the particle is broken into two equal pieces. As a result
of explosion one part comes to rest. Find the maximum height attained by the other part. Take
= 10 /
Solution: (35)

BT-IIT |Camp| FC Rd| PaudRoad|Wanowrie|Aundh|VimanNagar|Deccan|SataraRd|Pimple Saudagar|PCMC|www.bakliwaltutorialsiit.com 8


BT Tests Bakliwal Tutorials - IIT

Q.14 A ball of mass 1 kg is attached to an inextensible string. The ball is released from the position
shown in figure. Find the impulse (in N-s) imparted by the string to the ball immediately after the
string becomes taut. ( = 10 / )

Solution: (5)

Q.15 The friction coefficient between the horizontal surface and each of the block shown in the figure
is 0.2. The collision between the blocks is perfectly elastic. Find the separation (in cm) between
them when they come to rest [ = 10 / ]

Solution: (5)

BT-IIT |Camp| FC Rd| PaudRoad|Wanowrie|Aundh|VimanNagar|Deccan|SataraRd|Pimple Saudagar|PCMC|www.bakliwaltutorialsiit.com 9


BT Tests Bakliwal Tutorials - IIT

Q.16 A ball is projected from the point O with velocity 20 m/s at an angle of 60° with horizontal as
shown in figure. At highest point of its trajectory it strikes a smooth plane of inclination 30° at
point A. The collision is perfectly inelastic. The maximum height from the ground attained by
the ball is meter. Find the value of k? ( = 10 / )

Solution: (4)

Q.17 A is a fixed point at a height H = 2m above a perfectly inelastic smooth horizontal plane. A light
inextensible string of length = 3 has one end attached to A and other to a heavy particle. The
particle is held at the level of A with string just taut and released from rest. Find the height of the
particle above the plane when it is next instantaneously at rest.

Solution: (0.39)

BT-IIT |Camp| FC Rd| PaudRoad|Wanowrie|Aundh|VimanNagar|Deccan|SataraRd|Pimple Saudagar|PCMC|www.bakliwaltutorialsiit.com 10


BT Tests Bakliwal Tutorials - IIT

Q.18 On the rod on B point a force ⃗ = 3 ̂ + 10 ̂ + 5 is applied then the ratio of torque about point
A bout y-axis is √ then the value of n is?

Solution: (5)

SECTION –III
(Match the Column)
This section contains 2 questions. Each question has four statements (A, B, C, and D) given in Column I and four
statements (P, Q, R, S & T) in column II, in which ONE OR MORE may be correct.

BT-IIT |Camp| FC Rd| PaudRoad|Wanowrie|Aundh|VimanNagar|Deccan|SataraRd|Pimple Saudagar|PCMC|www.bakliwaltutorialsiit.com 11


BT Tests Bakliwal Tutorials - IIT
Q.19 Two identical balls A and B are kept on a smooth table as shown. B collides with A with speed
v. For different conditions mentioned in Column I, match with speed of A after collision given in
Column II

Column I Column II
(A) Elastic collision (P) 3
4
(B) Perfectly inelastic collision (Q) 5
8
(C) Inelastic collision with = (R)
(D) Inelastic collision with = (S)
2

Solution: [(A) – R; (B) – S; (C) - P; (D) - Q ]

Q.20 A body initially moving towards right explodes into two pieces, 1 and 2. Direction of motion of
the pieces as shown in Column I and possible mass ratios are shown in Column II

Column I Column II
(A) (P) >

V1 is vertical
(B) (Q) =

V1 is horizontal
(C) (R) <

V1 & V2 is both are horizontal


BT-IIT |Camp| FC Rd| PaudRoad|Wanowrie|Aundh|VimanNagar|Deccan|SataraRd|Pimple Saudagar|PCMC|www.bakliwaltutorialsiit.com 12
BT Tests Bakliwal Tutorials - IIT
(D) (S) Impossible for any masses

Solution: [(A) – S; (B) – S; (C) – P; (D) - PQR]

CHEMISTRY
SECTION – I
(Multiple Correct Answer(s) Type)
This section contains 10 multiple choice questions. Each question has four choices (A), (B), (C) and (D)
Out of Which ONE OR MORE may be correct.

Q.1 In 0.2 M aqueous solution of Ethylene diamine ( )∶


Given: = 8 × 10 & = 2.7 × 10 [log2 = 0.3]
(A) [ ] = 4 × 10 (B) = 9.8
(C) [ ] = 2.7 × 10 (D) = 11.6
Solution : (ACD)

Q.2 Which of the following statements are true for indicators?


(A) Indicators have different colours in dissociated and undissociated forms
(B) Indicators do not significantly affect the of the solution to which they are added
(C) For the titration of a strong acid and weak base phenolphthalein indicator (pH range 8.2 to
10) can be used.
(D) For the titration of a strong acid and strong base Methyl red indicator (pH range 4.8 to 6) can
be used.
Solution: (ABD)

BT-IIT |Camp| FC Rd| PaudRoad|Wanowrie|Aundh|VimanNagar|Deccan|SataraRd|Pimple Saudagar|PCMC|www.bakliwaltutorialsiit.com 13


BT Tests Bakliwal Tutorials - IIT

Q.3 is dissolved in 500 ml water to have = 4.5, choose the correct options:
Given : = 4.74
Log 1.82 = 0.26
Molar mass = 53.5
(A) Concentration of is 1.82 M
(B) Moles of are 0.91
(C) Amount of present in solution is 48.7 gm
(D) Solution is Acidic in nature.
Solution: (ABCD)

Q.4 A solution is a mixture of 0.01 M NaCl, 0.01 M NaBr and 0.01 M NaI, is gradually
added to the solution.
[ ] = 10 , [ ] = 10 , [ ] = 10
Choose the correct statements from the following:
(A) AgI will precipitate first followed by AgBr and AgCl will be last salt to precipitate
(B) When AgBr starts to precipitate the concentration of [ ] = 10
(C) When AgCl starts to precipitate the concentration of [ ] = 10
(D) Concentration of remaining in the solution when AgBr starts to precipitate is 10
Solution: (ABD)

BT-IIT |Camp| FC Rd| PaudRoad|Wanowrie|Aundh|VimanNagar|Deccan|SataraRd|Pimple Saudagar|PCMC|www.bakliwaltutorialsiit.com 14


BT Tests Bakliwal Tutorials - IIT

Q.5 Which of the following solutions would have same :


(A) 50 ml of 0.1 M HCl + 50 ml of 0.2 M
(B) 100 ml of 0.3 M HCl + 100 ml of 0.6 M
(C) 50 ml of 0.2 M HCl + 100 ml of 0.4 M
(D) 75 ml of 0.1 M HCl + 25 ml of 0.1 M
Solution: (AB)

Q.6 Dichromate ion in acidic medium oxidizes stannous ion as


+ + ⟶ + +
Choose the correct options from following :-
(A) The value of ∶ is 1 ∶ 3
(B) The value of + + is 18
(C) The value of a : b is 3 : 2
(D) The value of − is 7
Solution:( BC)

Q.7 An amount of solid is placed in a flask already containing ammonia gas at a certain
temperature at 0.20 atm pressure. Ammonium hydrogen sulfide decomposes to yield and
gases in the flask. When the decomposition reaction reaches Equilibrium, the total pressure
in the flask rises to 0.44 atm. Choose the correct options.
(A) Partial pressure of ( ) at equilibrium is 0.12 atm
(B) Partial pressure of ( ) at equilibrium is 0.12
(C) Total Pressure ( )
Equilibrium is 0.32
(D) The equilibrium constant for decomposition at this temperature is 0.038.
Solution :(ACD)

BT-IIT |Camp| FC Rd| PaudRoad|Wanowrie|Aundh|VimanNagar|Deccan|SataraRd|Pimple Saudagar|PCMC|www.bakliwaltutorialsiit.com 15


BT Tests Bakliwal Tutorials - IIT

Q.8 The rate of diffusion of 2 gases A and B are in the ratio 16 : 3. If the ratio of their masses present
in the mixture is 2 : 3. Then choose the correct options :
(A) Ratio of molar mass, =
(B) Ratio of molar mass, =8
(C) The ratio of their moles present inside the container : =8∶3
(D) The ratio of their moles present inside the container : =2∶3
Solution: (AC)

Q.9 For 10 Solution. Choose the correct statements:


Given = 5 × 10 , of aq. = 2 × 10 at 25° {Log 5 = 0.7,
2 = 0.3)
(A) of solution is 1
(B) Degree of hydrolysis 0.5
(C) of solution is 9.3
(D) On decreasing the concentration of salt the value of degree of hydrolysis
increases.
Solution : (ABC)

BT-IIT |Camp| FC Rd| PaudRoad|Wanowrie|Aundh|VimanNagar|Deccan|SataraRd|Pimple Saudagar|PCMC|www.bakliwaltutorialsiit.com 16


BT Tests Bakliwal Tutorials - IIT

Q.10 Choose the correct statements from the following:


(A) and ions have the same magnetic moments.
(B) < < – Acidic order
(C) > > – Electron Affinity
(D) < < < < < – Ionization energy
Solution: (A,B,C,D)

SECTION -II
(Numerical Type)
This section contains 8 Numeric Type questions.

Q.11 The concentration of in a given NiS solution is 2 × 10 moles/L. The minimum


ions necessary to cause precipitation of NiS is × 10 . Find the value of x
Given = 1.4 × 10
Solution: (7)

Q.12 x ml of 0.3 M is mixed with 30 ml of 0.2 M solution of to give buffer soluton of


8.25. Calculate value of x.
[ = 4.75]
Solution: (2)

BT-IIT |Camp| FC Rd| PaudRoad|Wanowrie|Aundh|VimanNagar|Deccan|SataraRd|Pimple Saudagar|PCMC|www.bakliwaltutorialsiit.com 17


BT Tests Bakliwal Tutorials - IIT

Q.13 Dissociation constant of a weak acid HA is 10 . Equilibrium constant of this weak acid with a
strong base is 1 × 10 . Calculate the value of x?
Solution: (9)

Q.14 Find the number of species having bond angle less than 109°28 .
, , , , ,( ) ,
Solution: (4)

Q.15 For the given equation: ( ) + 3 ( ) ⇌ 4 ( ) , initial concentration of A is equal to that of B.


The equilibrium concentration of A and C are equal. of the reaction is y. Calculate the value
of y?
Solution: (8)

BT-IIT |Camp| FC Rd| PaudRoad|Wanowrie|Aundh|VimanNagar|Deccan|SataraRd|Pimple Saudagar|PCMC|www.bakliwaltutorialsiit.com 18


BT Tests Bakliwal Tutorials - IIT

Q.16 In the reaction, ( ) ⇌2 ( ) if D and d are the vapour densities at initial stage and at
equilibrium then the value of at point A in the following graph is x. Calculate the value of
?

Solution: (2)

Q.17 Two weak base BOH and XOH have same pH, when their concentration ratio is 3 : 1. If of
BOH = 5, then of XOH is × 10 . Find n.
Solution: (3)

Q.18 Number of peaks in 4 vs r curve in 4d orbital will be x. calculate value of x


Solution: (2)

BT-IIT |Camp| FC Rd| PaudRoad|Wanowrie|Aundh|VimanNagar|Deccan|SataraRd|Pimple Saudagar|PCMC|www.bakliwaltutorialsiit.com 19


BT Tests Bakliwal Tutorials - IIT

SECTION –III
(Match the Column)
This section contains 2 questions. Each question has four statements (A, B, C, and D) given in Column I and four
statements (P, Q, R, S & T) in column II, in which ONE OR MORE may be correct.

Q.19 Match the Column I with Column II


Column I Column II
(A) 0.1 M Ph ] solution (P) = [ − − ]
(B) 0.1 M ( = 4.74) + (Q) = [ + + log ]
0.1
(C) 10 solution (R) | |
= + log | |
(D) 0.1 + (S) <7
0.1 ( = 4.75)
(T) >7

Solution: ([(A) – P, S; (B) – R, S; (C) – Q, T; (D) – T]

Q.20 Match the column


Column I Column II
(A) +3 ( ) ⇌2 ;∆ =− (P) Forward shift by rise in pressure
(B) 2 ( )
+ ( ) ⇌2 ( )
; ∆ = − (Q) Unaffected by change in pressure
(C) ( )
+ ( ) ⇌ 2 ( ) ; ∆ = + (R) Forward shift by rise in temperature
(D) ( )
+ ( )
⇌ ( )
; ∆ = + (S) Forward shift by lowering the
temperature

Solution: [(A) – P,S; (B) – P,S; (C) – Q, R; (D) – P,R]

MATH
SECTION – I
(Multiple Correct Answer(s) Type)
This section contains 10 multiple choice questions. Each question has four choices (A), (B), (C) and (D)
Out of Which ONE OR MORE may be correct.

BT-IIT |Camp| FC Rd| PaudRoad|Wanowrie|Aundh|VimanNagar|Deccan|SataraRd|Pimple Saudagar|PCMC|www.bakliwaltutorialsiit.com 20


BT Tests Bakliwal Tutorials - IIT
Q.1 The centre of circle passing through the points  0, 0  , 1,0  and touching the circle x2  y 2  9
is
(a)  3 / 2,1/ 2  (b) 1/ 3,3 / 2  (c) 1/ 2, 2  
(d) 1/ 2,  2  
Solution: (CD)

Q.2 Two circles, each of radius 5 units, touch each other at (1, 2). If the equation of their common
tangent is 4 x  3 y  10 , then the equation of the circles are
(A) x2  y2  10 x  10 y  25  0 (B) x2  y 2 10 x  10 y  25  0
(C) x2  y 2  6 x  2 y  15  0 (D) x2  y 2  6 x  2 y  15  0
Solution: (BC)
normal 3x-4y+5=0 passing through center of the circle

Q.3 The equation of a circle of equal radius, touching both the circles x2  y 2  a 2 and
2
 x  2a   y 2  a 2 is given by
(A) x 2  y 2  2ax  2 3ay  3a 2  0 (B) x 2  y 2  2ax  2 3ay  3a 2  0
(C) x 2  y 2  2ax  2 3ay  3a 2  0 (D) none of these
Solution: (AB)

BT-IIT |Camp| FC Rd| PaudRoad|Wanowrie|Aundh|VimanNagar|Deccan|SataraRd|Pimple Saudagar|PCMC|www.bakliwaltutorialsiit.com 21


BT Tests Bakliwal Tutorials - IIT

Q.4 If a < 0, then the roots of the equation x 2  2 a x  a  3a 2  0 is _________


(A) a 1  6  (B) a 1  2  
(C) a 1  6  
(D) a 1  2 
Solution: (BC)

3 5
Q.5 If cos   and cos   , then
5 13
33 56     1 63
(a) cos      (b) sin      (c) sin 2   (d) cos     
65 65  2  65 65
Solution: (BCD)

2
Q.6 If x  0, then 2 x   2  x can be

(A) 2  3x (B) 3x  2 (C) 2  x (D) 2  x


Solution: (A)

BT-IIT |Camp| FC Rd| PaudRoad|Wanowrie|Aundh|VimanNagar|Deccan|SataraRd|Pimple Saudagar|PCMC|www.bakliwaltutorialsiit.com 22


BT Tests Bakliwal Tutorials - IIT

Q.7 The value of ∈ for which the system of equations


+ + 3 = 0, + 2 + 2 = 0, 2 + 3 + 4 = 0
Admits of nontrivial solution is
(A) 2 (B) 5/2 (C) 3 (D) 5/4
Solution: (AD)

Q.8 The lines and denoted by 3 + 10 + 8 + 14 + 22 + 15 = 0 intersects at the point


P and have gradients and respectively. The acute angle between them is . Which of
following relations hold good:
(A) + = (B) =
(C) = sin (D) Sum of the abscissa and ordinate of point P is -

1.
Solution: (BCD)

Q.9 Let (1,1) and (3,3) be two fixed points and P be a variable point such that area of ∆
remain constant equal to 1 for all position of P, then locus of P is given by:
(A) 2 = 2 + 1 (B) 2 = 2 − 1 (C) = + 1 (D) = − 1
Solution: (CD)

BT-IIT |Camp| FC Rd| PaudRoad|Wanowrie|Aundh|VimanNagar|Deccan|SataraRd|Pimple Saudagar|PCMC|www.bakliwaltutorialsiit.com 23


BT Tests Bakliwal Tutorials - IIT

Q.10 If the chord = + 1 of the circle + = 1 subtends an angle of measure 45 at the


major segment of the circle then the values(s) of m is?
(A) 2 (B) 1 (C) -1 (D) None of these
Solution: (BC)

SECTION -II
(Numerical Type)
This section contains 8 Numeric Type questions.

3
Q.11 The maximum value of the expression is
9cos   8sin  cos   3sin 2 
2

Solution: (03.00)

Q.12 The locus of the mid-point of the chord of the circle x 2  y 2  2 x  2 y  2  0 which makes an
angle of 1200 at the centre is circle having center is (a,b) then ab is
Solution: (01.00)

BT-IIT |Camp| FC Rd| PaudRoad|Wanowrie|Aundh|VimanNagar|Deccan|SataraRd|Pimple Saudagar|PCMC|www.bakliwaltutorialsiit.com 24


BT Tests Bakliwal Tutorials - IIT

Q.13 The coordinates of the point on the circle x2  y 2  2 x  4 y  11  0 farthest from the origin is
a
(a,b) then is
b
Solution: (00.50)

Q.14 If the tangents are drawn from any point on the line x  y  3 to the circle x2  y 2  9 , then the
chord of contact passes through the point (a, b). then a+b is
Solution: (06.00)

Q.15 The equation  x  y  6  xy  3 x  y  3   0 represents the sides of a triangle then the radius of
circle passing through vertex of triangle .
Solution: (01.41)

BT-IIT |Camp| FC Rd| PaudRoad|Wanowrie|Aundh|VimanNagar|Deccan|SataraRd|Pimple Saudagar|PCMC|www.bakliwaltutorialsiit.com 25


BT Tests Bakliwal Tutorials - IIT

Q.16 The abscissas of two points A and B are the roots of the equation x 2  4 x  3  0 and their
ordinate are the roots of the equation x 2  2 x  8  0 . The radius of the circle with AB as
diameter is
Solution: (04.00)

Q.17 The number of values of in the interval − , such that ≠ for = 0, ±1, ±2 and
tan = cot 5 as well as sin 2 = cos 4 is
Solution: (03.00)

Q.18 For all real values of a and b, lines (2 + ) + ( + 3 ) + ( − 3 ) = 0 and +2 +


6 = 0 are concurrent. Then | | is equal to _______.
Solution: (02.00)

BT-IIT |Camp| FC Rd| PaudRoad|Wanowrie|Aundh|VimanNagar|Deccan|SataraRd|Pimple Saudagar|PCMC|www.bakliwaltutorialsiit.com 26


BT Tests Bakliwal Tutorials - IIT

SECTION –III
(Match the Column)
This section contains 2 questions. Each question has four statements (A, B, C, and D) given in Column I and four
statements (P, Q, R, S & T) in column II, in which ONE OR MORE may be correct.

Q.19
Sr. No. Column I Sr. No. Column II
(A) If , , are in A.P., then lines + + = 0 are (P) (−4, −7)
concurrent at
(B) A point on the lines + = 4 which lies at a unit (Q) (−7,11)
distance from the line 4 + 3 = 10 is
(C) Orthocentre of triangle made by lines + = 1, (R) 1, −2
− + 3 = 0, 2 + = 7 is
(D) Two vertices of a triangle are (5, −1) and (−2,3). If (S) (−1,2)
orthocentre is the origin, then coordinates of the third
vertex are
Solution: [(A  R),(B  Q),(C  S),(D  P)]

BT-IIT |Camp| FC Rd| PaudRoad|Wanowrie|Aundh|VimanNagar|Deccan|SataraRd|Pimple Saudagar|PCMC|www.bakliwaltutorialsiit.com 27


BT Tests Bakliwal Tutorials - IIT

Q.20
Sr. Column I Sr. No. Column II
No.
(A) Two intersecting circles (P) Have a common tangent
(B) Two circles touching each other (Q) Have a common normal
(C) Two non-concentric circles, one strictly (R) Do not have a common normal
inside the other
(D) Two concentric circles of different radii (S) Do not have a radical axis

Solution: [(A  PQ),(B  PQ),(C  Q),(D  QS)]


Conceptual.

BT-IIT |Camp| FC Rd| PaudRoad|Wanowrie|Aundh|VimanNagar|Deccan|SataraRd|Pimple Saudagar|PCMC|www.bakliwaltutorialsiit.com 28


Date: 15th December 2019 Bakliwal Tutorials-IIT
Comprehensive 2021 BTest - 7 YOUR STRATEGIC MOVE

Total Time: - 3.00 hrs Sub: - Physics, Chemistry & Math Max. Marks: -300

SET-A
Rules & regulations for BTEST

1. Immediately fill in the particulars on this page of the Test Booklet with Ball Point Pen only.
2. The Answer Sheet (OMR) is provided separately. Please fill in the particulars carefully with Ball Point
Pen.
3. The test is of 3 hours duration.
4. The Test Booklet consists of 75 questions. The maximum marks are 300.
5. There are three parts in the question paper consisting of Physics, Chemistry and Mathematics having 25
questions in each part. Question pattern of question paper is as follows,
New Mains Pattern
Single Correct Answer Type -
This section contains 20 multiple choice questions. (Marking +4, -1)

Numerical Type - (Marking +4, 0)

These questions will have answers lying between 0 to 99.99 and always with xx.xx form meaning the
decimal point will be always after 2 digits. We will provide bubbling for only 4 digits. Hence if the
answer is 99.99 you need to bubble only 9999. A few more cases are as below.
A. Answer 1.54 then Bubble 0154
B. Answer 1.987 will be 0199 due to rounding off of the last digit.
C. Answer 0.1889 will be 0019 due to rounding off.
D. Answer 32.23 will be 3223.
6. No candidate is allowed to carry any textual material, printed or written, bits of papers, pager, mobile
phone, any electronic device, etc. except the BT ID Card inside the examination room/hall.
7. Rough work is to be done on the additional sheets provided
8. On completion of the test, the candidate must immediately hand over the Answer Sheet to the Invigilator
on duty in the Room/Hall. However, the candidates are allowed to take away this Test Booklet with them.
9. The CODE for this Booklet is A. Make sure that the CODE is marked properly on the answer sheet.
10. Do not fold or make any stray mark on the Answer Sheet.

BT-IIT |Camp| FC Rd| PaudRoad|Wanowrie|Aundh|VimanNagar|Deccan|SataraRd|Pimple Saudagar|PCMC|www.bakliwaltutorialsiit.com 1


BT Tests Bakliwal Tutorials - IIT
PHYSICS
SECTION – I
Single Correct Answer Type
This section contains 20 multiple choice questions. Each question has four choices (A), (B), (C) and
(D) out of which ONLY ONE is correct.

Q.1 A force ⃗ is applied at a height ℎ = above the center of a hollow sphere. Then for no slipping
on rough surface; the direction of friction on sphere is in
(A) forward direction (B) backward direction
(C) not defined (D) can be forward or backward
Solution : (B)

Q.2 A uniform disc of mass 600 kg and radius 2 metre is rotating at the rate of 600 r.p.m. The torque
required to rotate the disc in the opposite direction with same speed in a time of 100 second will
be -
(A) 480π Nm (B) 240π Nm (C) 120π Nm (D) 600π Nm
Solution : (A)

Q.3 A hollow sphere rests on a rough horizontal plane. It is given a horizontal velocity at its centre.
After some time, it starts executing pure rolling. The loss in energy during the process is -
(A) 10 % (B) 40 % (C) 33 (1/3) % (D) 66 (2/3) %
Solution : (B)

BT-IIT |Camp| FC Rd| PaudRoad|Wanowrie|Aundh|VimanNagar|Deccan|SataraRd|Pimple Saudagar|PCMC|www.bakliwaltutorialsiit.com 2


BT Tests Bakliwal Tutorials - IIT
Q.4 A disc-like reel with massless thread unrolls itself while falling vertically downwards. The end
of the thread is fixed to roof. The acceleration of the centre of reel is
(A) zero (B) g (C) 2g/3 (D) g/2
Solution : (C)

Q.5 A solid sphere is placed on a horizontal plane. A horizontal impulse J is applied at a distance ‘h’
above the central line as shown in the figure. Soon after giving the impulse the sphere starts
rolling. The ratio h/R would be –

(A) 1/2 (B) 2/5 (C) 1/4 (D) 1/5


Solution : (B)

Q.6 In the arrangement shown in the figure, the mass of uniform solid cylinder of radius R is equal to
‘m’ and the masses of two bodies are equal to m1 and m2. The thread slipping and the friction in
the axle of the cylinder are supposed to be absent. Then the ratio of tensions T1/T2 of the vertical
sections of the thread in the process of motion is;

( ) ( ) ( ) ( )
(A) ( )
(B) ( )
(C) ( )
(D) ( )
BT-IIT |Camp| FC Rd| PaudRoad|Wanowrie|Aundh|VimanNagar|Deccan|SataraRd|Pimple Saudagar|PCMC|www.bakliwaltutorialsiit.com 3
BT Tests Bakliwal Tutorials - IIT
Solution : (A)

Q.7 A square plate of side l has mass per unit area . Its moment of inertial about an axis passing
through the centre and perpendicular to its plane is:
(A) (B) (C) (D)
Solution : (D)

Q.8 When a solid sphere rolls without slipping, the ratio of its kinetic energy of translation to its total
kinetic energy is:
(A) 1 : 7 (B) 1 : 2 (C) 1 : 1 (D) 5 : 7
Solution : (D)

Q.9 A circular wooden hoop of mass ‘m’ and radius R rests on a horizontal frictionless surface. A
bullet, also of mass ‘m’ and moving with a velocity ‘v’, strikes the hoop and get embedded in it.
The thickness of wooden hoop is much smaller than R. The angular velocity with which the
system rotates after bullet strikes the hoop is –

(A) v/3R (B) v/2R (C) v/R (D) 2v/R


Solution : (A)

BT-IIT |Camp| FC Rd| PaudRoad|Wanowrie|Aundh|VimanNagar|Deccan|SataraRd|Pimple Saudagar|PCMC|www.bakliwaltutorialsiit.com 4


BT Tests Bakliwal Tutorials - IIT

Q.10 One end of a uniform rod of length l and mass ‘m’ is hinged at A. It is released from rest from
horizontal position AB as shown in the figure. The force exerted by the rod on the hinge when it
becomes vertical is –

(A) (3/2) mg (B) (5/2) mg (C) 3 mg (D) 5 mg


Solution : (B)

Q.11 A spherical ball of mass 5 kg is resting on a plane with angle of inclination 30° with respect to
the horizontal as shown in the figure. The ball is held in place by a rope attached horizontally to
the top of the ball and to the slope. The frictional force acting on the ball is –

(A) N (B) N (C) N (D) none of these


√ √ √
Solution : (A)

BT-IIT |Camp| FC Rd| PaudRoad|Wanowrie|Aundh|VimanNagar|Deccan|SataraRd|Pimple Saudagar|PCMC|www.bakliwaltutorialsiit.com 5


BT Tests Bakliwal Tutorials - IIT

Q.12 A round uniform body of radius R, mass M and moment of inertia l rolls down (without slipping)
an inclined plane making an angle with the horizontal. Then its acceleration is:
(A) (B) (C) (D)
Solution : (B)

Q.13 A thin uniform rod AB of mass m = 1 kg moves translationally with acceleration a = 2 m/s
due to two anti parallel forces F and F . The distance between the points at which these forces
are applied is equal to a = 20 cm. Besides, it is known that F = 5 N. Then the length of the rod
will be ;

(A) 1.0 m (B) 1.5 m (C) 2.0 m (D) 2.5 m


Solution : (A)

BT-IIT |Camp| FC Rd| PaudRoad|Wanowrie|Aundh|VimanNagar|Deccan|SataraRd|Pimple Saudagar|PCMC|www.bakliwaltutorialsiit.com 6


BT Tests Bakliwal Tutorials - IIT

Q.14 Two point masses of 0.3 kg and 0. 7 kg are fixed at the ends of a rod of length 1.4 m and of
negligible mass. The rod is set rotating about an axis , perpendicular to its length with a uniform
angular speed. The point on the rod through which the axis should pass in order that the work
required for rotation of the rod is minimum, is located at a distance of –
(A) 0.42 m from mass of 0.3 kg (B) 0.70 m from mass of 0.7 kg
(C) 0.98 m from mass of 0.3 kg (D) 0.98 m from mass of0.7 kg
Solution : (C)

Q.15 A spool of wire rests on a horizontal rough surface. As the wire is pulled, the spool does not slip
at the point of contact P. On separate trials, each one of the four forces is applied as shown. For
which force will the spool not move?

(A) F (B) F (C) F (D) F


Solution : (B)

Q.16 Three different liquids are filled in a U-tube as shown in figure. Their densities are , and
respectively. From the figure we may conclude that:

BT-IIT |Camp| FC Rd| PaudRoad|Wanowrie|Aundh|VimanNagar|Deccan|SataraRd|Pimple Saudagar|PCMC|www.bakliwaltutorialsiit.com 7


BT Tests Bakliwal Tutorials - IIT

(A) =2× (B) > (C) = 2( − ) (D) =


Solution : (C)
Equate pressure in the lower fluid
Q.17 The three vessels shown in figure have same base area. Equal volumes of a liquid are poured in
the three vessels. The force by liquid on the base will be

(A) maximum in vessel A (B) maximum in vessel B


(C) maximum in vessel C (D) equal in all the vessels
Solution : (C)
Height will be more in C
Q.18 A piece of wood is floating in water kept in a bottle. The bottle is connected to an air pump.
Neglect the compressibility of water. When more air is pushed into the bottle from the pump, the
piece of wood will float with;
(A) larger part in the water (B) lesserpartin the water
(C) same part in the water (D) it will sink
Solution : (C)

Q.19 A cubical block of wood of specific gravity 0.5 and a chunk of concrete of specific gravity 2.5
are fastened together. The ratio of the mass of wood to the mass of concrete which makes the
combination to float with its entire volume submerged under water is;
(A) 1/5 (B) 1/3 (C) 3/5 (D) 2/5
Solution : (C)
Let total volume be 1 units
Let volume of wood = v
Then, Volume of concrete = (1-v)

For floatation,
Weight of block = buoyant force
0.5vg + 2.5(1-v)g = 1*1*g
then, v = 0.75

Mass of wood = 0.5*v = 0.375


Mass of concrete = 2.5(1-v) = 0.625

Ratio = 3/5

BT-IIT |Camp| FC Rd| PaudRoad|Wanowrie|Aundh|VimanNagar|Deccan|SataraRd|Pimple Saudagar|PCMC|www.bakliwaltutorialsiit.com 8


BT Tests Bakliwal Tutorials - IIT
Q.20 To construct a barometer, a tube of length 1 m is filled completely with mercury and is
inverted in a mercury cup. The barometer reading on a particular day is 76 cm. Suppose a 1 m
tube is filled with mercury up to 76 cm and then closed by a cork. It is inverted in a mercury cup
and the cork is removed. The height of mercury column in the tube over the surface in the cup
will be;
(A) zero (B) 76 cm (C) > 76 cm (D) < 76 cm
Solution : (D)

SECTION -II
(Numerical Type)
This section contains 5 questions.

Q.21 In the diagram shown, a disc of mass m = 500 g and radius R = 5 cm is hinged from its center to
the lower end of the rod of mass m and length L = 20 cm. The disc is free to rotate about the
lower end of the rod. The rod is free to rotate about its upper end which is hinged. The rod and
the disc are at rest. A particle of mass m approaches with speed u and collides elastically with the
rod at a distance x = 10 cm from the upper end. Find the angular speed of the disc in radians per
second just after the impact.

Solution : (00.00)

Q.22 The uniform semicircular bar of mass 1 kg and radius 20 cm is hinged freey about a horizontal
axis through A. if the bar is released from rest in the position shown, where AB is horizontal,
determine the initial angular acceleration of the bar in SI units. Take g = 9.8 m/s2.

Solution: (24.50)

BT-IIT |Camp| FC Rd| PaudRoad|Wanowrie|Aundh|VimanNagar|Deccan|SataraRd|Pimple Saudagar|PCMC|www.bakliwaltutorialsiit.com 9


BT Tests Bakliwal Tutorials - IIT
Q.23 A string is wound around a cylinder of mass 2 kg and radius 10 cm. The string is pulled
vertically upward to prevent the centre of mass from falling, as the string unwinds. Find the
pulling force in SI units.
Solution : (20.00)

Q.24 A heavy plank of mass 55 kg is placed over two cylindrical rollers of radii 10 cm and 5 cm. Mass
of rollers is 40 kg and 20 kg respectively. Plank is pulled rightwards by applying a force of 25 N.
Find the frictional force applied by the smaller roller on the plank if the plank remains horizontal
and slipping does not take place anywhere. (find the answer upto first decimal place)

Solution : (02.40)

Q.25 A vessel contains oil (density = 0.8 g/cm ) over mercury (density = 13.6 g/cm ) . A
homogeneous sphere floats with half its volume immersed in mercury and the other half in oil.
The density of the material of the sphere in g/cm is;
Solution : (07.20)

CHEMISTRY
SECTION – I
Single Correct Answer Type
This section contains 20 multiple choice questions. Each question has four choices (A), (B), (C) and
(D) out of which ONLY ONE is correct.

Q.1 Choose incorrect Statement from following:


(A) Li < Na < K < Rb - ionic mobility
(B) Na O < K O < Rb O < Cs O - stability of peroxide
(C) Li O > Na O > K O > Rb O - basic strength
(D) LiHCO < NaHCO < KHCO < RbHCO - stability of bicarbonate
Solution : (C)
Conceptual.

BT-IIT |Camp| FC Rd| PaudRoad|Wanowrie|Aundh|VimanNagar|Deccan|SataraRd|Pimple Saudagar|PCMC|www.bakliwaltutorialsiit.com 10


BT Tests Bakliwal Tutorials - IIT
Q.2. CH NH (0.1 mole, K = 5 × 10 ) is added to 0.08 mole of HCl and the solution is diluted
to one litre, resulting hydrogen ion concentration is
(A) 1.6 × 10 (B) 8 × 10 (C) 5 × 10 (D) 8 × 10

Solution : (B)

Q.3. The pH of 0.1 M solution of the following salts increases in the order.
(A) NaCl < H Cl < < (B) HCl < NH Cl < <
(C) NaCN < H Cl < NaCl < HCl (D) HCl < < < NH Cl

Solution : (B)

Q.4. Which of the following statement is incorrect?


(A) Li is the strongest reducing agent among all the alkali elements.
(B) Solutions of alkali metals in liquid ammonia are a good reducing agents and blue in colour
due to solvated electrons.
(C) In preparation of caustic soda by the electrolysis of aqueous NaCl, Cl is obtained at cathode
(D) Li and Mg both form Nitrides.

Solution : (C)

Q.5. A gas contained in a cylinder fitted with a frictionless piston expands against a constant pressure
1 atmosphere from a volume of 4 litre to a volume of 14 litre. In doing so, it absorb 800 J
thermal energy from surroundings. Determine ∆ for the process.
(A) 213.7 J (B) - 213.7 J (C) 557.3 J (D) -557.3 J

Solution : (B)
W = − P × ∆V = −1 × 10 = −10 lit − atm
W = 101.37 × −10 = −1013.7 J
∆U = Q + W = 800 − 1013.7 = −213.7 J

BT-IIT |Camp| FC Rd| PaudRoad|Wanowrie|Aundh|VimanNagar|Deccan|SataraRd|Pimple Saudagar|PCMC|www.bakliwaltutorialsiit.com 11


BT Tests Bakliwal Tutorials - IIT

Q.6. If a gas absorbs 200 J of heat and expands by 500 against a constant pressure of 2 ×
10 , then the change in internal energy is
(A) -300 J (B) -100 J (C) +100 J (D) +300 J

Solution : (C)
∆ = + = − ∆
= 200 − 2 × 10 × 500 × 10
= 100

Q.7. An ideal gas is taken around the cycle ABCA as shown in P-V diagram. The net work done
during the cycle is equal to

(A) -12P V (B) −5P V (C) −6P V (D) −10P V

Solution : (B)
Work done in the cyclic process = Area bounded (ABCA) = −5P V

Q.8. Out of molar entropy (I), Specific Volume (II), heat capacity (III), internal energy (IV), extensive
properties are :
(A) I, II (B) II, III, IV (C) III, IV (D) All

Solution : (C)

Q.9. Calculate final temperature of a monoatomic ideal gas that is compressed reversibly and
adiabatically from 16 L to 2 L at 300 K :
(A) 600 k (B) 1200 k (C) 1800 k (D) 2400 k

Solution : (B)

BT-IIT |Camp| FC Rd| PaudRoad|Wanowrie|Aundh|VimanNagar|Deccan|SataraRd|Pimple Saudagar|PCMC|www.bakliwaltutorialsiit.com 12


BT Tests Bakliwal Tutorials - IIT

Q.10. The solubility of Ba (AsO ) (formula mass = 690) is 6.9 × 10 g/100 mL. What is the K ?
(A) 1.08 × 10 (B) 1.08 × 10 (C) 1.0 × 10 (D) 6.0 × 10

Solution : (B)

Q.11. In the following which has highest boiling point


(A) HI (B) HF (C) HBr (D) HCl

Solution : (B)
in HF due to hydrogen bonding boiling point increases

Q.12. Maximum number of lone pairs on central atom are present in :


(A) BF (B) IO (C) XeF (D) IF

Solution : (C)

Q.13. Energy is released in which of the following process?


(A) Na ( ) → Na( ) + e (B) O( ) + e → O( )
(C) N( ) + e → N( ) (D) O( ) + e → O( )

Solution : (D)

Q.14. In the energy of an electron in the first energy level of H atom is -13.6V. The possible value of
the first excited state for electron in He is
(A) −54.4 (B) -13.6 (C) -3.4 (D) -1.4

BT-IIT |Camp| FC Rd| PaudRoad|Wanowrie|Aundh|VimanNagar|Deccan|SataraRd|Pimple Saudagar|PCMC|www.bakliwaltutorialsiit.com 13


BT Tests Bakliwal Tutorials - IIT
Solution : (B)
For He : E = −13.6 z
= - 54.4
.
∴ E = = −13.6

Q.15. The difference between the heats of reaction at constant pressure and a constant volume for the
reaction 2C H ( ) + 15O ( ) → 12CO ( ) + 6H O( ) at 25°C in kJ is
(A) -7.43 (B) +3.72 (C) -3.72 (D) +7.43

Solution : (A)

Q.16. 0.4 M solution completely reacts with 0.05 M solution under acidic conditions.
the volume of used is 50 . What volume of was used in ?
(A) 1.25 (B) 2.50 (C) 5.75 (D) 7.50

Solution : (A)

Q.17. Among the following, what is the total number of compounds having +3 oxidation state of the
underlined elements.
I Cl , K P O , Ca ClO
OF , NO , Ba X O , Ni(CO)
(A) 2 (B) 3 (C) 4 (D) 5
Solution : (B)

Q.18. Choose the correct statement from following :


(A) In preparation of Na CO from − Blanc process, CaCl , CO and NH gas are recovered.
(B) Acidified solution of sodiumthiosulphate is unstable and the reaction is disproportionate.
(C) Li NO & (NO ) on decomposition give N and O
(D) Li CO on heating gives Li O and CO as major products.
BT-IIT |Camp| FC Rd| PaudRoad|Wanowrie|Aundh|VimanNagar|Deccan|SataraRd|Pimple Saudagar|PCMC|www.bakliwaltutorialsiit.com 14
BT Tests Bakliwal Tutorials - IIT

Solution : (B)

Q.19. At 27℃ , one mole of an ideal gas is compressed isothermally and reversibly from a pressure of
5 atm to 10 atm the values of is (R = 2 Cal/mole K)
(A) 414.54 (B) - 414.54 (C) 812.24 (D) - 812.24

Solution : (B)

Q.20. A B( ) is introduced in a vessel at 1000 K. If partial pressure of A B( ) is l atm initially and K


for reaction A B( ) ⇌ 2A( ) + B( ) is 81 × 10 then percentage of dissociation of A B will be :
(A) 2.7 (B) 6.3 (C) 7.2 (D) 4.5

Solution : (A)

SECTION -II
(Numerical Type)
This section contains 5 questions.

Q.21. One mole of an ideal gas undergoes a change of state (2.0 atm, 3.0 L) to (2.0 atm, 7.0 L) with a
change in internal energy (∆U) = 30 L − atm. Calculate the change in enthalpy (∆H) of the
process in L-atm.

Solution : (38.00)
At Constant pressure, ∆H = ∆U + P ∙ ∆V
= 30 + 2 × 4
= 38 atm − L

BT-IIT |Camp| FC Rd| PaudRoad|Wanowrie|Aundh|VimanNagar|Deccan|SataraRd|Pimple Saudagar|PCMC|www.bakliwaltutorialsiit.com 15


BT Tests Bakliwal Tutorials - IIT

Q.22. CuSO ∙ 5H O(s) ⇌ CuSO ∙ 3H O(s) + 2H O(g); K = 4 × 10 atm . If the vapour pressure
of water is 38 torr then percentage of relative humidity is : (Assume all data at constant
temperature)

Solution: (40.00)
P = 2 × 10 atm
× ×
R. H. = × 100
= 40 %

Q.23. To a 10 mL of 10 N H SO solution water has been added to make the total volume of one
litre. Its pOH would be :

Solution : (09.00)

Q.24. At 273 K temperature and 9 atm pressure, the compressibility for a gas is 0.9. Calculate The
volume of 1 milli-mole of gas at this temperature and pressure in ?

Solution: (02.24 )
. × . ×
Z= = =
= 2.24 litre/mol
∴ Volume of 1 milli − mole of gas = 2.24 mL

Q.25. What will be the pH of an aqueous solution of 1.0 M ammonium formate?


Given : pK (HCOOH) = 3.8 and pK ( OH) = 4.8
Solution : (06.50)
pH = (pK + pK − pK ) = 6.5

MATH
SECTION – I
Single Correct Answer Type
This section contains 20 multiple choice questions. Each question has four choices (A), (B), (C) and
(D) out of which ONLY ONE is correct.

Q.1 If the second, third and fourth terms in the expansion of ( + ) are 135, 30 and 10/3 respectively, then
(A) = 7 (B) = 5 (C) = 6 (D) None of these
Solution: (B)

BT-IIT |Camp| FC Rd| PaudRoad|Wanowrie|Aundh|VimanNagar|Deccan|SataraRd|Pimple Saudagar|PCMC|www.bakliwaltutorialsiit.com 16


BT Tests Bakliwal Tutorials - IIT

Q.2 A person writes letter to 6 friends and addresses the corresponding envelopes. In how many ways can the
letters be placed in the envelopes so that at least 4 of them are in wrong envelopes?
(A) 72 (B) 719 (C) 664 (D) None of these
Solution: (C)

Q.3 The number of permutations of all the letters of the word ‘PERMUTATION’ such that any two
consecutive letters in the arrangement are neither both vowels nor both identical is
(A) 63 × 6 × 5 (B) 57 × 5 × 5 (C) 33 × 6 × 5 (D) 7 × 7 × 5
Solution: (B)

Q.4 Number of ways in which AAABBB can be placed in the squares of the figure as shown so that no row
remains empty, is

BT-IIT |Camp| FC Rd| PaudRoad|Wanowrie|Aundh|VimanNagar|Deccan|SataraRd|Pimple Saudagar|PCMC|www.bakliwaltutorialsiit.com 17


BT Tests Bakliwal Tutorials - IIT

(A) 2430 (B) 2160 (C) 1620 (D) None of these


Solution: (C)

Q.5 Number of different ways in which 8 different books can be distributed among 3 students, if each student
receives at least 2 books is
(A) 2940 (B) 2980 (C) 3042 (D) None of these
Solution: (A)

Q.6 Find the total number of proper divisors divisible by 10 of the number 35700.
(A) 72 (B) 31 (C) 70 (D) 32
Solution: (B)
BT-IIT |Camp| FC Rd| PaudRoad|Wanowrie|Aundh|VimanNagar|Deccan|SataraRd|Pimple Saudagar|PCMC|www.bakliwaltutorialsiit.com 18
BT Tests Bakliwal Tutorials - IIT
The number of proper divisors divisible by 10 is equal to number of selections from (5, 5), (2, 2), (3),
(7), (17) consisting of at least one 5 and at least one 2 and 35700 is to be excluded and is given by
2 × 2 × 2 × 2 × 2 − 1 = 31

Q.7 Number of ways in which two Americans, two British, one Chinese, one Dutch and one Egyptian can sit
on a round table so that persons of the same nationality are separated is
(A) 48 (B) 240 (C) 336 (D) None of these
Solution: (C)

Q.8 In how many ways 50 different things can be distributed among 5 persons so that, three of them get 12
things each and two of them get 7 things each?
! ! !
(A) ! ! ! ! (B) ( !) ( !) ! ! 5! (C) ( !) ( !) ! ! (D) None of these
Solution: (B)

Q.9 The number of four-digit number strictly greater than 4321 that can be formed using the digits 0,1,2,3,4,5
(repetition of digits is allowed) is:
(A) 288 (B) 360 (C) 306 (D) 310
Solution: (D)

BT-IIT |Camp| FC Rd| PaudRoad|Wanowrie|Aundh|VimanNagar|Deccan|SataraRd|Pimple Saudagar|PCMC|www.bakliwaltutorialsiit.com 19


BT Tests Bakliwal Tutorials - IIT

Q.10 In how many ways can the letter of the word INDEPENDENT be arranged such that the relative position
of consonants remain unchanged
.
(A) × (B) 1680 (C) (D) 11088
.
Solution: (B)

Q.11 If the roots of the quadratic equation + + = 0 are tan 30° and tan 15° respectively, then the
value of 2 + q – p is
BT-IIT |Camp| FC Rd| PaudRoad|Wanowrie|Aundh|VimanNagar|Deccan|SataraRd|Pimple Saudagar|PCMC|www.bakliwaltutorialsiit.com 20
BT Tests Bakliwal Tutorials - IIT
(A) 3 (B) 0 (C) 1 (D) 2
Solution: (A)


Q.12 The sum of the following series 1 + 6 + + + + ⋯ up to 15
terms is
(A) 7510 (B) 7820 (C) 7830 (D) 7520
Solution: (B)

Q.13 The tangent and the normal lines at the point √3, 1 to the circle + = 4 and the x-axis form a
triangle. The area of this triangle (in square units) is
(A) (B) (C) (D)
√ √ √
Solution: (C)

BT-IIT |Camp| FC Rd| PaudRoad|Wanowrie|Aundh|VimanNagar|Deccan|SataraRd|Pimple Saudagar|PCMC|www.bakliwaltutorialsiit.com 21


BT Tests Bakliwal Tutorials - IIT

Q.14 If the circles + − 16 − 20 + 164 = and ( − 4) + ( − 7) = 36 intersect at two distinct


points, then
(A) 0 < < 1 (B) > 11 (C) 1 < < 11 (D) = 11
Solution: (C)

Q.15 Product of real roots of


5 + 2√6 + 5 − 2√6 = 10
(A) 8 (B) 4 (C) -8 (D) -4
BT-IIT |Camp| FC Rd| PaudRoad|Wanowrie|Aundh|VimanNagar|Deccan|SataraRd|Pimple Saudagar|PCMC|www.bakliwaltutorialsiit.com 22
BT Tests Bakliwal Tutorials - IIT
Solution: (A)

Q.16 The exponent of 2 in


= 20 × 19 × 18 × … × 11. is
(A) 10 (B) 15 (C) 20 (D) 12
Solution; (A)

Q.17 The number of integral solutions for the equation + + + = 20, where x, y, z, t are all ≥ −1, is
(A) (B) (C) (D)
Solution: (D)

BT-IIT |Camp| FC Rd| PaudRoad|Wanowrie|Aundh|VimanNagar|Deccan|SataraRd|Pimple Saudagar|PCMC|www.bakliwaltutorialsiit.com 23


BT Tests Bakliwal Tutorials - IIT

Q.18 7 boys be seated at a round table. X is the number of ways in which two particular boys are next to each
other, Y is number of ways in which they are separated. Then + is
(A) 500 (B) 600 (C) 700 (D) 720
Solution: (D)

Q.19 The maximum value of 5 cos + 3 cos + + 3 is


(A) 5 (B) 11 (C) 10 (D) -1
Solution: (C)

Q.20 The equation of circle which intersects circles


+ + + 2 + 3 = 0, + + 2 + 4 + 5 = 0 and + − 7 − 8 − 9 = 0 at right
angle, will be
(A) + −4 −4 −3= 0 (B) 3( + ) + 4 − 4 − 3 = 0
(C) + +4 +4 −3 = 0 (D) 3( + ) + 4( + ) − 3 = 0
Solution: (D)

BT-IIT |Camp| FC Rd| PaudRoad|Wanowrie|Aundh|VimanNagar|Deccan|SataraRd|Pimple Saudagar|PCMC|www.bakliwaltutorialsiit.com 24


BT Tests Bakliwal Tutorials - IIT

SECTION -II
(Numerical Type)
This section contains 5 questions.

Q.21 If the middle term in the expansion of + is 924 , then = ……….., (where n is even)
Solution: (12.00)

Q.22 From 25 tickets numbered from 1 to 25, number of ways in which 3 tickets can be chosen such that the
numbers on them are in A.P. with even common difference, is:
Solution: (66.00)

Q.23 If the one of the diameters of the circle, given by the equation, + − 4 + 6 − 12 = 0, is a chord
of a circles S, whose centre is at (−3, 2), then the radius of S is
Solution: (8.64 to 8.68)

BT-IIT |Camp| FC Rd| PaudRoad|Wanowrie|Aundh|VimanNagar|Deccan|SataraRd|Pimple Saudagar|PCMC|www.bakliwaltutorialsiit.com 25


BT Tests Bakliwal Tutorials - IIT

Q.24 (1 − cot 1°)(1 − cot 2°)(1 − cot 3°) … … (1 − cot 44°) = 2 then find n.
Solution: (22.00)

Q.25 From the point (4, -4) tangent lines are drawn to the circle + − 6 + 2 + 5 = 0. Calculate the
length of the chord joining the points of contact.
Solution: (03.14 to 03.18)

BT-IIT |Camp| FC Rd| PaudRoad|Wanowrie|Aundh|VimanNagar|Deccan|SataraRd|Pimple Saudagar|PCMC|www.bakliwaltutorialsiit.com 26


BT Tests Bakliwal Tutorials - IIT

BT-IIT |Camp| FC Rd| PaudRoad|Wanowrie|Aundh|VimanNagar|Deccan|SataraRd|Pimple Saudagar|PCMC|www.bakliwaltutorialsiit.com 27


Date: 12th January 2020 Bakliwal Tutorials-IIT
Comprehensive 2021 BTest - 8 YOUR STRATEGIC MOVE

Total Time: - 3.00 hrs Sub: - Physics, Chemistry & Math Max. Marks: -204

SET-A
Rules & regulations for BTEST

1. Immediately fill in the particulars on this page of the Test Booklet with Ball Point Pen only.
2. The Answer Sheet (OMR) is provided separately. Please fill in the particulars carefully with Ball Point
Pen.
3. The test is of 3 hours duration.
4. The Test Booklet consists of 60 questions. The maximum marks are 240.
5. There are three parts in the question paper consisting of Physics, Chemistry and Mathematics having 20
questions in each part. Question pattern of question paper is as follows,
Advance Pattern
Single Correct Answer Type -
This section contains 8 multiple choice questions. (Marking +3, -1)

Multiple Correct Answer Type –


This section contains 8 multiple choice questions. (Marking +4, -2)

Numeric Type -
This section contains 4 multiple choice questions. (Marking +3, 0)

These questions will have answers lying between 0 to 99.99 and always with xx.xx form meaning the
decimal point will be always after 2 digits. We will provide bubbling for only 4 digits. Hence if the answer
is 99.99 you need to bubble only 9999. A few more cases are as below.
A. Answer 1.54 then Bubble 0154
B. Answer 1.987 will be 0199 due to rounding off of the last digit.
C. Answer 0.1889 will be 0019 due to rounding off.
D. Answer 32.23 will be 3223.
6. No candidate is allowed to carry any textual material, printed or written, bits of papers, pager, mobile
phone, any electronic device, etc. except the BT ID Card inside the examination room/hall.
7. Rough work is to be done on the additional sheets provided
8. On completion of the test, the candidate must immediately hand over the Answer Sheet to the Invigilator on
duty in the Room/Hall. However, the candidates are allowed to take away this Test Booklet with them.
9. The CODE for this Booklet is A. Make sure that the CODE is marked properly on the answer sheet.
10. Do not fold or make any stray mark on the Answer Sheet.

BT-IIT |Camp| FC Rd| PaudRoad|Wanowrie|Aundh|VimanNagar|Deccan|SataraRd|Pimple Saudagar|PCMC|www.bakliwaltutorialsiit.com 1


BT Tests Bakliwal Tutorials - IIT
PHYSICS
SECTION – I
Single Correct Answer Type
This section contains 8 multiple choice questions. Each question has four choices (A), (B), (C) and (D) out of
which ONLY ONE is correct.

3
Q.1 Air is blown through a pipe AB at a rate of 1600 cm /sec . The cross-section area of the broad portion of
2 2
pipe is 4 cm and that of narrow portion is 2 cm . The difference in water length is
 air  1.4kg / m 3 ,  water  103 kg / m3  (Take g= 10 m/s-2)

(A) 1.60 mm (B) 3.02 mm (C) 3.36 mm (D) 4.20 mm


Solution: (C)

Q.2 A liquid is filled in a spherical container of radius R till a height h. At this positions the liquid surface at the
edges is also horizontal. The contact angle is

R h hR  R h


(A) 0 (B) cos 1   (C) cos 1   (D) sin 1  
 R   R   R 
Solution: (B)

BT-IIT |Camp| FC Rd| PaudRoad|Wanowrie|Aundh|VimanNagar|Deccan|SataraRd|Pimple Saudagar|PCMC|www.bakliwaltutorialsiit.com 2


BT Tests Bakliwal Tutorials - IIT
Q.3 A spherical drop of radius r and density n is falling in air with terminal velocity. The density of air is n0
and its coefficient of viscosity  . The power developed by gravity is
r 2n r 5 27 5 8r 5
(A) n  n0 g2 (B) n n  n 0  (C) 2
r n  n  n 0  g (D) n n  n 0 g 2
 2 8 27
Solution: (D)

2
Q.4 A small hole of area 1 cm is punched on the side of a cylindrical vessel containing water upto a height H =
10 m as shown. The torque of normal force about ‘Centre of the vessel O’ immediately after making the
hole would be :
Assume that tank does not topple. (Take g= 10 m/s-2)

(A) 10 Nm (B) 30 Nm (C) 50 Nm (D) Zero


Density of water is 1000 kg/m3
Solution: (C)

2
Q.5 When a tangential force of 0.02 N is applied on a large wooden plate of area 10 m floating on the surface
of river, plate moves with the constant speed 2 m/s on the river surface. The river is 1 m deep and the water
in contact with the bed is stationary. Assuming constant speed gradient, coefficient of viscosity of water is
(A) 105poise (B) 102 poise (C) 109 poise (D) 1015poise
Solution: (B)

Q.6 A cubical block of volume v and density 3  is placed inside a liquid of density  and attached to a spring
of spring constant k as shown in the figure. Assuming ideal spring and pulley and spring is attached at A
which is at R/2 from center. The compression in the spring at equilibrium is

BT-IIT |Camp| FC Rd| PaudRoad|Wanowrie|Aundh|VimanNagar|Deccan|SataraRd|Pimple Saudagar|PCMC|www.bakliwaltutorialsiit.com 3


BT Tests Bakliwal Tutorials - IIT

4vg 2vg 3 vg vg


(A) (B) (C) (D)
k k k k
Solution: (A)

Q.7 Water is filled in a uniform container of area of cross section A. A hole of cross section a (<< A) is made in
the container at a height of 20 m above the base. Water streams out and hits a small block on surface at
some distance from container. Block is moved on surface in such a way that stream always hits the block.
 a 1 
 .  take g  10m / s 
2
The initial speed of the block (in m/s) is  given 
 A 1000 

(A) 0.01 (B) 0.02 (C) 0.03 (D) 0.04


Solution: (B)
Let height of the liquid above hole be ‘h’. Using Toricelli theorem velocity of efflux will be
2gH
 If x is the height of hole from the ground,
Range of efflux will be
2x
R  2gH
g
R  2 x H [x is given = 20 m]
dR 1 dH
2 x
dt 2 H dt
Volume of container (V) = AH
V = AH

dv AdH

dt dt

dH
a 2gH  A
dt

= 2

BT-IIT |Camp| FC Rd| PaudRoad|Wanowrie|Aundh|VimanNagar|Deccan|SataraRd|Pimple Saudagar|PCMC|www.bakliwaltutorialsiit.com 4


BT Tests Bakliwal Tutorials - IIT
= 2√20. . . √20. √

= 0.02

Q.8 Soap has surface tension  . We make a plain soap film and on the soap film we placed a loop of rubber
band of total length l. the cross-sectional area of the rubber band is S and its Young’s modulus of elasticity
is E. Now the film inside the loop is pierced. The rubber loop acquires a circular shape. Find the tension in
the rubber loop.
E2σls Eσls
(A) 2σl (B) (C) (D) Es-lσ
Es  σl Es  σl
Solution: (C)

SECTION – II
(Multiple Correct Answer(s) Type)
This section contains 8 multiple choice questions. Each question has four choices (A), (B), (C) and (D)
Out of Which ONE OR MORE may be correct.

Q.9 A cubical block is floating in a liquid with one third of it’s volume immersed in the liquid. When the whole
system accelerates upwards with acceleration of g/2.
(A) The fraction of volume immersed in the liquid will change.
(B) The buoyancy force on the block will change.
(C) The buoyancy force will increase by 50 percent.
(D) The pressure in the liquid will increased.
Solution: (BCD)

BT-IIT |Camp| FC Rd| PaudRoad|Wanowrie|Aundh|VimanNagar|Deccan|SataraRd|Pimple Saudagar|PCMC|www.bakliwaltutorialsiit.com 5


BT Tests Bakliwal Tutorials - IIT

Q.10 A vertical capillary tube with inside radius 0.25 mm is submerged into water so that the length of its part
protruding over the water surface is equal to 25 mm. surface tension of water is 73 103 N / m and angle
2
of contact is zero degree for glass and water, acceleration due to gravity is 9.8 m / s . Then choose correct
statement.
(A) R = 0.25 mm (B) h = 59.6 mm (C) R = 0.60 mm (D) h = 25 mm
where R is radius of meniscus and h is height of water in capillary tube.
Solution: (CD)


Q.11 A closed cuboidal-shaped container is completely filled with water and has acceleration a  aiˆ  bjˆ  ckˆ .
Neglect gravity. The pressure will be greatest (among all vertices) at

(A) D if a < 0, b < 0 and c < 0 (B) A if a > 0, b > 0 and c > 0
(C) E if a < 0, b > 0, and c > 0 (D) H if a > 0 b > 0, and c > 0
Solution: (AB)
Conceptual

Q.12 The stress-strain graphs for material A and B are shown in the figure. The graphs are drawn to the same
scale. Choose the correct option(s)

(A) Both materials have same young’s modulus.


(B) The material A has greater Young’s modulus than that of B.
(C) Material A is more ductile than B.
(D) Material B is more brittle than material A.
Solution: (BCD)
Slope and ductile region of material is A greater than material B
BT-IIT |Camp| FC Rd| PaudRoad|Wanowrie|Aundh|VimanNagar|Deccan|SataraRd|Pimple Saudagar|PCMC|www.bakliwaltutorialsiit.com 6
BT Tests Bakliwal Tutorials - IIT
4 3
Q.13 Three identical blocks each of mass m = 1 kg and volume 3  10 m are suspended by massless strings
from a support as shown. Underneath are three identical containers containing the same amount of water
that are placed over the scales. In figure (a), the block is completely out of the water, in figure (b), the block
is completely submerged but not touching the beaker and in figure (c) the block rests on the bottom of the
3
beaker and tension in the string is zero. The scale in figure (a) reads 14 N. Density of water is 1000kg / m
(Take g= 10 m/s-2)

(A) The tension in the string in (b) is 10 N (B) The tension in the string in (b) is 7 N
(C) The reading of the scale in (b) is 17 N (D) The reading of the scale in (c) is 24 N
Solution: (B,C,D)

Q.14 A target is placed at the bottom of a tank of depth 20 m, and filled with a liquid of density 500 gm/cc.
Another object of density 100 gm/cc is dropped onto it, from a height of 45 m from the upper surface of the

liquid. take g = 10 m/s 2 

(A) The second object will collide with the target


(B) The second object does not collide with the target
(C) The least separation between object and target is 12.25 m.
2
(D) The retardation of the object in the liquid is 40 ms .
Solution: (BD)

BT-IIT |Camp| FC Rd| PaudRoad|Wanowrie|Aundh|VimanNagar|Deccan|SataraRd|Pimple Saudagar|PCMC|www.bakliwaltutorialsiit.com 7


BT Tests Bakliwal Tutorials - IIT
Q.15 A cubical block of mass 5 kg and side 10 cm is pressed against a rough wall    0.9 with a force F
passing through the centre of cube inside a swimming pool as shown in the figure.
(Take g= 10 m/s-2) (Neglect Force of air) Then:

(A) The cube will remain in equilibrium if the force F  355 / 9 Newton.
(B) The cube will remain in equilibrium if the force is F  355 / 9 Newton.
(C) The friction force acting on the cube is 40 N if F = 110/3 N.
(D) The friction force acting on the cube is 40 N if F = 50 N.
Solution: (AD)

Q.16 A light wire of length l (figure – 1) is cut into two pieces in two different ways as shown in (figure – 2 & 3).
Different pieces can be arranged in placed of wire as shown and a load can be placed on the massless
hanger. Choose the correct statement(s).

(A) The load required to break the wire B’ is 6 times that required to break B
(B) The stress required to break the wire B and C is same but to break B and B’ is different
(C) The stress required to break C and C’ is same
(D) The load required to break A and B’ is same, but different for B and B’
Solution: (CD)

BT-IIT |Camp| FC Rd| PaudRoad|Wanowrie|Aundh|VimanNagar|Deccan|SataraRd|Pimple Saudagar|PCMC|www.bakliwaltutorialsiit.com 8


BT Tests Bakliwal Tutorials - IIT

SECTION -III
(Numerical Type)
This section contains 4 Numeric Type questions.
Q.17 Ethanol of density   700kg / m3 flows smoothly through a horizontal pipe that tapers in cross-sectional
A1
area from A1  1.2  103 m3 to A 2  . The pressure difference between the wide and the narrow
2
sections of pipe is 4200 Pa. Flow rate of ethanol is Nx10-3 m3/sec, Then Value of N is?
Solution: (0240)

Q.18 Two block of masses m and M are connected by means of a metal wire passing over a frictionless fixed
pulley. The area of cross-section of the wire is 6.5 109 m2 and its breaking stress is 2 109 Nm2 . If m =
1 kg, if the maximum value of M (in kg) for which the wire will not break. Find the value of M.
 g  10 m / s 2

Solution: (0185 – 0186)

Q.19 The difference in water levels in the two communicating capillary tubes of different diameter d = 1 mm and
d = 1.5 mm is K (in millimeter). Surface tension of water = 0.07 N/m and angle of contact between glass
0
and water is 0 . Find the value of K. (Take g= 10 m/s-2 )

BT-IIT |Camp| FC Rd| PaudRoad|Wanowrie|Aundh|VimanNagar|Deccan|SataraRd|Pimple Saudagar|PCMC|www.bakliwaltutorialsiit.com 9


BT Tests Bakliwal Tutorials - IIT

Solution: (0933)

Q.20 A small spherical ball of density same as that of liquid is released from rest in a vessel filled completely
with a liquid and accelerating with acceleration 5 ̂ + 5 ̂ / as shown in the figure. Find the initial
acceleration of ball just after the release with respect to vessel. (Take g= 10 m/s-2)

Solution: (0000)

CHEMISTRY
SECTION – I
Single Correct Answer Type
This section contains 8 multiple choice questions. Each question has four choices (A), (B), (C) and (D) out of
which ONLY ONE is correct.

Q.1 At room temperature, para hydrogen is -


(A) Less stable than ortho hydrogen (B) More stable than ortho hydrogen
(C) As stable as ortho hydrogen (D) Both are unstable at room temperature
Solution: (B)

Q.2 Beryllium hydride is obtained by-


(A) Heating Be in atmosphere of H2 (B) The action of BeCl2 with LiAlH4
(C) The action of Be with CaH2 (D) Dissolving Be in water
Solution: (B)

Q.3 The IUPAC name of the given compound

BT-IIT |Camp| FC Rd| PaudRoad|Wanowrie|Aundh|VimanNagar|Deccan|SataraRd|Pimple Saudagar|PCMC|www.bakliwaltutorialsiit.com 10


BT Tests Bakliwal Tutorials - IIT
CH2–CH3
CH3–CH–CH2–C–OH is -
OH CH2–CH3

(A) 2, 4-Hexanediol (B) 4-Ethyl-2,4-hexanediol


(C) 4, 4-diethylbutane-2,4-diol (D) 3, 3-Diethyl-1,3-butanediol
Solution: (B)
5 6
1 2 3 4
C–C
C–C–C–C–OH
OH C–C

Q.4 The structure of allyl vinyl thioether is -


(A) CH2 = CH – CH2 – S – CH2 – CH = CH2
(B)CH2 = CH – S – CH2 – CH = CH2
(C) CH2 = CH – S – CH = CH2
(D) CH2 = CH – O – CH2 – CH = CH2
Solution: (B)
Vinyl group:CH2 = CH –, Allyl group – CH2 – CH = CH2, Thio: S

Q.5 HA  OH ¯ H2O  A ¯  q1kJ


H   OH ¯ H2O  q2 kJ
The enthalpy of dissociation of weak acid HA is
(A) (q1+ q 2) (B) (q1 - q 2) (C) (q2 - q 1) (D) - (q1+ q 2)
Solution: (C)
Equation 1: HA  OH ¯ H2O  A ¯ , H= -q1

Equation 2: H  OH ¯ H2O , H= - q2

Equation 1 – Equation 2 = HA  H  A ¯ , H= -q1  (q2 )  q2  q1

Q.6 In the following unbalanced redox reaction,


Cu3P  Cr2O72 
 Cu2  H3PO4  Cr3
Equivalent weight of H3PO4 is -
M M M M
(A) (B) (C) (D)
3 6 7 8
Solution: (D)
3 5
C u3 P  
 H 3 P O 4 So change in oxidation state for P = 5-(-3) = 8

Q.7 An ideal gas confined inside an adiabatic container is suddenly taken from state 1 to state 2 by a
two stage process as shown. Calculate change in enthalpy.

BT-IIT |Camp| FC Rd| PaudRoad|Wanowrie|Aundh|VimanNagar|Deccan|SataraRd|Pimple Saudagar|PCMC|www.bakliwaltutorialsiit.com 11


BT Tests Bakliwal Tutorials - IIT

(A)  H = 2  P0 V0 (B)  H =  P0 V 0 (C)  H =  3 P0 V 0 (D) H = 3 P0 V0


 1  1
Solution: (B)
Q0
U  Q  w  w
w  P0 (4V0  V0 )  3P0 V0
H  U  (P2 V2  P1V1 )
 3P0V0  (4P0V0  2P0 V0 )
 3P0V0  2P0 V0
 P0V0

Q.8 A reaction at 300 K with  G 0   1743J consists of 3 mole of A (g), 6 mole of B (g) and 3 mole of
C (g). If A, B and C are in equilibrium in 1 litre container then the reaction may be
 Given : ln 2  0.7, R  8.3J / Kmol
(A) A  B  C (B) A  B  2C (C) 2A  B  C (D) A  B  2C
Solution: (C)

G  RT ln K eq
or,  1743  8.3  300  ln k eq
k eq  2  k eq for reaction 2A  B  Cfrom given data

SECTION – II
(Multiple Correct Answer(s) Type)
This section contains 8 multiple choice questions. Each question has four choices (A), (B), (C) and (D)
Out of Which ONE OR MORE may be correct.

Q.9 Three centre - two electron bonds exist in :


(A) B2H6 (B) Al2(CH3)6 (C) (BeH2)n polymer (D) BCl3
Solution: (ABC)

Q.10 A 2.5 g impure sample containing weak monoacidic base (Mol. wt. = 45) is dissolved in 100 ml
th
1
water and titrated with 0.5 M HCl. When   of the base was neutralized the pH was found to be
5
9 and at equivalent point pH of solution is 4.5.
BT-IIT |Camp| FC Rd| PaudRoad|Wanowrie|Aundh|VimanNagar|Deccan|SataraRd|Pimple Saudagar|PCMC|www.bakliwaltutorialsiit.com 12
BT Tests Bakliwal Tutorials - IIT

Given : All data at 25 C & log 2 = 0.3
Select correct statement(s).

(A) Kb of base is less than 10  6


(B) Concentration of salt (C) at equivalent point is 0.25 M
(C) Volume of HCl is used at equivalent point is 100 ml
(D) Weight percentage of base in given sample is 80%
Solution: (BC)
(A)
BOH  HCl  BCl  H 2O
100 m 0.5 V 20 m
100 M  0.5 V 0
 80 M
14  a
salt
pOH  pKb  log
base
20M
5  pKb  log
80M
 pkb  5  log 0.25
 pkb  5.6
 Kb  2.5 10 6
greater than 10 6

(B) At equivalence point,


1
pH  7   pK b  log C 
2
1
 log C = 5 - 5.6 = - 0.6 = log
4
1
C= = 0.25
4

(C) [HCl] taken = 0.5 M


[Salt] at equivalence point = 0.25 M
HCl is limiting reagent & [Salt] at equivalence point
1
   HCl 
2
So, Volume has exactly doubled.
HCl added = 100 mL

(D) Moles of base = Moles of HCl added


= 100  0.5  10  3
= 0.05
Mass of base = 0.05 × 45
0.05  45
% of base =  100
2.5
= 90 %

BT-IIT |Camp| FC Rd| PaudRoad|Wanowrie|Aundh|VimanNagar|Deccan|SataraRd|Pimple Saudagar|PCMC|www.bakliwaltutorialsiit.com 13


BT Tests Bakliwal Tutorials - IIT
Q.11 For the data given :
Zn(OH)2(s) Zn(OH)2 (aq) ; K1 = 10 –6
Zn(OH)2(aq) Zn(OH) + (aq) + OH– (aq); K2 = 10–7
Zn(OH)+ (aq) Zn+2(aq) + OH–(aq) ; K3 = 10 –4
Zn(OH)2(aq) + OH– (aq) Zn(OH)3– (aq); K4 = 10 3
Zn(OH)3– (aq) + OH– (aq) Zn(OH)4–2 (aq); K5 = 10
The true statement(s) is(are) (s = solubility of Zn(OH)2(s)) -
(A) s = 1 M at pH = 4.5 (B) s = 10–15 M at pH = 13
(C) s = 2 × 10–4 M at pH = 13 (D) s = 2 × 10–4 M at pH = 10
Solution: (AB)
 Zn 2  ( aq )  2OH  , K eq  K1  K 2  K 3  10 17
Zn (OH ) 2 
s  [OH  ]2  10 17
10 17
s
[OH  ]2

Q.12 The wave function of 2s electron is given by


3/2  r 
1 1  r   2 a0 
2s     2  e
4 2  a 0   a0 
Find the correct statement(s).
(A) This orbital has a radial node at r = a0 .

(B)This orbital has a radial node at r = 2a0


(C) The orbital, being from s subshell, does not have any radial node.
(D) The orbital does not have any nodal plane.
Solution: (BD)
r
2 0
When the wavefunction becomes 0, at that point a0
r  2 a0
[ r = 0 and r =  are not considered as nodal points.]
S orbitals do not have any nodal plane as they are spherically symmetric.

Q.13 Which of the following equation(s) is/are correct for the changes shown below?
A, B, C and D are perfect crystals. P = 1 bar throughout.
r1  Sº
mA + nB (298 K) ZC + YD (298 K)
 r S 2º  r S 4º

r 3 Sº
mA + nB (0 K)  ZC + YD (0 K)

(A)  rS1º = (ZSCº + YSDº) – (mSAº + nSBº) (B)  rS2º = + [mSAº + nSBº]
BT-IIT |Camp| FC Rd| PaudRoad|Wanowrie|Aundh|VimanNagar|Deccan|SataraRd|Pimple Saudagar|PCMC|www.bakliwaltutorialsiit.com 14
BT Tests Bakliwal Tutorials - IIT
(C)  rS3º = 0 (D)  rS4º = – [ZSCº + YSDº]
Solution: (AC)
Solution:  rS2º = [mSA (O K) + nSB (O K)] - [mSAº + nSBº] = - [mSAº + nSBº]
(D)  rS4º = [ZSCº + YSDº]– [ZSC (O K) + YSD (O K)] = + [ZSCº + YSDº]

At O K, entropy of A = B = C = D = O [3rd law of thermodynamics]

Q14. The equilibria between 3 gases, A, B and C, can be represented as-


Reaction Equilibrium constant
A(g)  B(g) : K1  ?
B(g)  C(g) : K2  0.4
C(g)  A(g) : K3  0.6
1 mole of A is taken in a closed vessel of volume 1 L. Then the correct statements are-

A) [A]+[B]+[C] = 1 at equilibrium. B) K1 = 1/0.6


C) K1 = 1/0.24 D) log K1 = pK2 + pK3
Solution: (ACD)
1
Equation 2 + Equation 3 => B( g )  A( g ), Keq  K2  K3 
K1
A(g)  B(g)
t=0 1 0
teq , 1 x  z x-y
B(g)  C(g)
teq , x -y y–z
C(g)  A(g)
teq , y–z 1-x+z
[A] + [B] + [C] = (1 – x + z) + (x – y) + (y – z)
=1

Q.15 Select the correct statement(s):


(A) The value of compressibility factor 'Z' for H2 gas is greater than one at room temperature and
pressure.
(B) The real gas behaves as an ideal gas at critical temperature.
(C) NH3 shows lower value for Van der waals constant ‘a’ compared to N2.
P.b
(D) At high pressure, the compressibility factor 'Z' = 1 + for a Van der waals gas.
RT
Solution: (AD)
A, B, D – Theory based
C – Due to hydrogen bonding, intermolecular force is higher in NH3.

Q.16 The correct structure of the compound 2-Amino- 5-bromocyclohexane-1-carboxylic acid is/are-

BT-IIT |Camp| FC Rd| PaudRoad|Wanowrie|Aundh|VimanNagar|Deccan|SataraRd|Pimple Saudagar|PCMC|www.bakliwaltutorialsiit.com 15


BT Tests Bakliwal Tutorials - IIT
Br COOH
COOH
NH2
(A) NH2 (B) (C) (D)
HOOC
Br NH2 Br
NH2

Br COOH
Solution: (AB)

SECTION -III
(Numerical Type)
This section contains 4 Numeric Type questions.
Q.17 When 10 litre of O2 gas at 1 atm is compressed adiabatically to 1/5th of its volume, then pressure
increases to 6.5 atm. find out the work done in the process. (Assuming ideal behavior for O2, give
your answer in L-atm)
Solution: (0750)
P2 V2 – P1 V1
W=
 –1

= 6.5  2 – 10  1 L – atm
7
–1
5
= 7.5 L-atm

Q.18 One mole of Argon is heated using P V 5 / 2  constant. By what amount heat is absorbed during the
process, when temperature changes by T  26K . Calculate in Joule. Divide the calculated heat by
10 and express your answer. ( = 8.3 / )
Solution: (1798)

3
n  1Cv  R PV5/2  cons tan t, n '  5 / 2
2
q  n C T
 
 R  3 R  3 2  5 5  8.3  26
 n Cm,v   T  1  R   26   R  R  26  R  26   179.83J
1 n '  5
 2 1   2 3  6 6
 2

Q.19 KMnO solution is to be standardized by titration against As2O3 (s) . A 0.1097 g sample of
4
As2O3 requires 26.10 ml of the KMnO solution for its oxidation to in acid medium. What
4
is the molarity of the KMnO solution? (Atomic weight of As = 75 u)
4
Solution: (0017)
BT-IIT |Camp| FC Rd| PaudRoad|Wanowrie|Aundh|VimanNagar|Deccan|SataraRd|Pimple Saudagar|PCMC|www.bakliwaltutorialsiit.com 16
BT Tests Bakliwal Tutorials - IIT

Mn 7  5e  Mn 2 (Re duction)


3 5 
As  2As  4e
2 (oxidation)
 Meq.of As 2O 3  meq of KMnO 4
0.1097
198
 1000  26.10  N E As2 O3 M/4 
4
 N KMnO4  0.085
0.085
 M KMnO4   0.017
5

Q.20 The following sequence of reaction occurs in commercial production of aqueous nitric acid.
4NH3(g)  5O2 (g) 4NO(g)  6H2O(l) H = - 904 kJ … (1)
2NO(g)  O2 (g)  2NO2 (g) H = - 112 kJ … (2)
3NO2 (g)  H2O  l   2HNO3  aq  + NO  g  H = - 140 kJ … (3)
Determine the total heat liberated (in kJ) at constant pressure for the production of exactly 1 mole
of aqueous nitric acid from NH3 by this process. Divide the amount by 10 and express your
answer.
Solution: (4930)
3 3 3
1molof HNO3  moles of NO2  mol of NO  mol of NH3
2 2 2
3 1 3 1  3 1
     904      (112)     (140)  493kJ / mol
2 4 2 2  2 3
Heat liberated  493kJ / mol

MATH
SECTION – I
Single Correct Answer Type
This section contains 8 multiple choice questions. Each question has four choices (A), (B), (C) and (D) out of
which ONLY ONE is correct.

Q.1 A ray of light incident at the point (-2, -1) gets reflected from the tangent at (0, -1) to the circle + =
1. The reflected ray touches the circle. The equation of the line along which the incident ray moved is
(A) 4 − 3 + 11 = 0 (B) 4 + 3 + 11 = 0
(C) 3 + 4 + 11 = 0 (D) 4 + 3 + 7 = 0
Solution: (B)

BT-IIT |Camp| FC Rd| PaudRoad|Wanowrie|Aundh|VimanNagar|Deccan|SataraRd|Pimple Saudagar|PCMC|www.bakliwaltutorialsiit.com 17


BT Tests Bakliwal Tutorials - IIT

/
Q.2 If = + and = − then − = ( − ) where k =
(A) 1 (B) 2 (C) 3 (D) 4
Solution: (D)

Q.3 The line lx + my = 1 intersects the circle + = at points A, B, If AB subtends 45° at the origin then
( + )=
(A) 4 − 2√2 (B) 4 − 2√6 (C) 2√6 (D) 4 − √6
Solution: (A)

BT-IIT |Camp| FC Rd| PaudRoad|Wanowrie|Aundh|VimanNagar|Deccan|SataraRd|Pimple Saudagar|PCMC|www.bakliwaltutorialsiit.com 18


BT Tests Bakliwal Tutorials - IIT

Q.4 The number of ways in which 4 distinct balls can be put into 4 boxes labeled a, b, c, d so that exactly one
box remains empty is
(A) 232 (B) 196 (C) 192 (D) 144
Solution: (D)
Any of the four boxes can be empty in ways then 4 balls are to be distributed in the remaining 3 boxes
such that no box remains empty.
= 3 − . 2 + = 36
Total number of ways = 4 × 36 = 144

Q.5 If coefficient of in (1 + ) (1 − + ) is non-zero, then n cannot be of the form


(A) 3r +1 (B) 3r (C) 3r + 2 (D) 4r + 1
Solution: (C)

The exp ression is (1  x)101 (1  x  x 2 )100


 (1  x)((1  x)(1  x  x 2 ))100
 (1  x)(1  x 3 )100
 (1  x) C0  C1x 3  C 2 x 6  ....  C100 x 300 
100 100 100
 (1  x)  n C r x 3r   n C r x 3r   n C r x 3r 1
r 0 r 0 r 0

Hence there will be no term containing 3r  2

BT-IIT |Camp| FC Rd| PaudRoad|Wanowrie|Aundh|VimanNagar|Deccan|SataraRd|Pimple Saudagar|PCMC|www.bakliwaltutorialsiit.com 19


BT Tests Bakliwal Tutorials - IIT
Q.6 If ( + ) + 2 + = 0 and ( + ) + 2 + = 0, ( ≠ 0) then
(A) = + (B) = (C) =− (D) None of these.
Solution: (A)
Given a(p  q) 2  2bpq  c  0 and
a(p  r)2  2bpr  c  0
 q and r satisfy the equation
a(p  x)2  2bpx  c  0
 q and r are the roots of a(p  x) 2  2bpx  c  0
i.e. q and r are the roots of
ax 2  2(ap  bp)x  c  ap 2  0
c  ap 2 c
 ap  product of roots   p2 
a a

Q.7 If a, b, c are positive numbers such that a + b + c = 1, then the minimum value of + + is
(A) 3 (B) 9 (C) 27 (D) None of these
Solution: (C)
1 1 1 1
   and
ab bc ca abc
abc 1 1
 (abc)1/3  abc    27
3 27 abc

Q.8 If | | = 2, | | = 3, | | = 4 and | + + | = 2, then the value of |4 +9 + 16 |


(A) 24 (B) 48 (C) 96 (D) 120
Solution: (B)

SECTION – II
(Multiple Correct Answer(s) Type)
This section contains 8 multiple choice questions. Each question has four choices (A), (B), (C) and (D)
Out of Which ONE OR MORE may be correct.

Q.9 The number of words which can be made from letters of the word INTERMEDIATE is
(A) 907200 if words start with I and end with E
(B) 21600 if vowels and consonants occupy their original places
(C) 43200 if vowels and consonants occur alternatively
(D) 302400 if all the vowels occur together
Solution: (ABC)

BT-IIT |Camp| FC Rd| PaudRoad|Wanowrie|Aundh|VimanNagar|Deccan|SataraRd|Pimple Saudagar|PCMC|www.bakliwaltutorialsiit.com 20


BT Tests Bakliwal Tutorials - IIT

Q.10 The range of values of ‘a’ such that the angle between the pair of tangents drawn from (a, 0) to the circle
+ = 1 satisfies < < , lies in
(A) (1, 2) (B) 1, √2 (C) −√2, −1 (D) −√2, −1 ∪ 1, √2
Solution: (BCD)

Equation of pair of tan gents by SS'  T 2 is (a 2  1)y 2  x 2  2ax  a 2  0


2 h 2  ab 2 (a 2  1)( 1) 2 (a 2  1)
If  be the angle between the tan gents then tan    
ab a2  2 a2  2
If  lies in II quadrant then tan   0
2 (a 2  1)
  0  a2 1  0 & a2  2  0
a2  2
 a 1 and a  2
 a  (  2, 1)  (1, 2)

Q.11 Let S1 , S2, …… be squares such that for each n ≥ l, the length of a side of Sn equals the length of a diagonal
of Sn+1. If the length of a side of S1 is 10 cm. then for which of the following values of n is the area of Sn
less than l sq. cm?
(A) 10 (B) 9 (C) 8 (D) 7
Solution: (ABC)

BT-IIT |Camp| FC Rd| PaudRoad|Wanowrie|Aundh|VimanNagar|Deccan|SataraRd|Pimple Saudagar|PCMC|www.bakliwaltutorialsiit.com 21


BT Tests Bakliwal Tutorials - IIT

Q.12 If sin + cos = + for ∈ [0, ] and y > 0, then

(A) = (B) y = 0 (C) y = 1 (D) =


Solution: (AC)

1 1
y  2 y   2
y y
But sin x  cos x  2
1
 y   2 and sin x  cos x  2
y
 
 y  1 and sin  x    1
 4

or y  1 and x 
4

Q.13 If all the three vertices of an isosceles right angle triangle be rational points and length of base is an integer,
then which of the points is/are always a rational point (A point P (x, y) is a rational point if both x and y are
rational).
(A) centroid (B) incentre (C) circumcentre (D) orthocentre
Solution: (ACD)
Since a is an integer, √2 a is irrational now incentre
√ √
will be ,
√ √
which is clearly an irrational point

BT-IIT |Camp| FC Rd| PaudRoad|Wanowrie|Aundh|VimanNagar|Deccan|SataraRd|Pimple Saudagar|PCMC|www.bakliwaltutorialsiit.com 22


BT Tests Bakliwal Tutorials - IIT

Centroid is , and
Circumcenter is , , which are rational. Clearly orthocentre is also rational.

Q.14 The roots of + + = 0. Where ≠ 0 and coefficients are real, are nonreal complex and
+ < . Then
(A) 4 + > 2 (B) 4 + < 2 (C) +4 >2 (D) +4 < 2
Solution: (BD)

The roots are non-real complex,


⇒ , −4 < 0
Hence ( ) = + + must have the same sign
For all real x. Now (−1) = − + < 0 (given condition)
Thus ( ) < 0 for all ∈ . In particular (−2) < 0.
That is, 4 − 2 + < 0 ⇒ 4 + < 2
Also, − < 0 ⇒ − 2 + 4 < 0

Q.15 If the sum of the coefficients in the expansion of (2 + 3 + ) vanishes then c equals
(A) -2 (B) 2 (C) 1 (D) -1
Solution: (AD)
To find sum of the coefficients in the expansion, we put x = 1. So,
(2 + 3 + ) = 0 ⇒ + 3 + 2 = 0
⇒ = −2, −1

Q.16 If amp ( ) = 0 and | | = | | = 1, then


(A) + =0 (B) =1 (C) = (D) =
Solution: (BC)

SECTION -III
(Numerical Type)
This section contains 4 Numeric Type questions.

Q.17 If the sum S = C + 2 ∙ C + 3 ∙ C + ⋯ + 19 ∙ C can be expressed as 1 + ∙ 2 ,


where p is an odd number. Then find the value of ( + ).
Solution: (2900)

BT-IIT |Camp| FC Rd| PaudRoad|Wanowrie|Aundh|VimanNagar|Deccan|SataraRd|Pimple Saudagar|PCMC|www.bakliwaltutorialsiit.com 23


BT Tests Bakliwal Tutorials - IIT

Q.18 The number of ordered pairs ( , ); , ∈ {1, 2, 3, … … … , 20} such that 3 + 7 is a multiple of 10, is
equal to then find the value of .
Solution: (1000)

Q.19 If + + 1 = 0, then the value of + + + + + + … + + is equal to

Solution: (4800)


Q.20 Let A = log √
8; B = 5 ;C = ;D = 2
Find the value of A + B + (C / D).
Solution: (1406 to 1407)
 30 
log 2  
 2
A  8; B  6; C  1; D  2  15
1 211
A  B  C / D  8  6  1/ 15  14  
15 15

BT-IIT |Camp| FC Rd| PaudRoad|Wanowrie|Aundh|VimanNagar|Deccan|SataraRd|Pimple Saudagar|PCMC|www.bakliwaltutorialsiit.com 24


Date: 9 th February 2020 Bakliwal Tutorials-IIT
Comprehensive 2021 BTest - 9 YOUR STRATEGIC MOVE

Total Time: - 3.00 hrs Sub: - Physics, Chemistry & Math Max. Marks: -300

SET-A
Rules & regulations for BTEST

1. Immediately fill in the particulars on this page of the Test Booklet with Ball Point Pen only.
2. The Answer Sheet (OMR) is provided separately. Please fill in the particulars carefully with Ball Point
Pen.
3. The test is of 3 hours duration.
4. The Test Booklet consists of 75 questions. The maximum marks are 300.
5. There are three parts in the question paper consisting of Physics, Chemistry and Mathematics having 25
questions in each part. Question pattern of question paper is as follows,
New Mains Pattern
Single Correct Answer Type -
This section contains 20 multiple choice questions. (Marking +4, -1)

Numerical Type - (Marking +4, 0)

These questions will have answers lying between 0 to 99.99 and always with xx.xx form meaning the
decimal point will be always after 2 digits. We will provide bubbling for only 4 digits. Hence if the
answer is 99.99 you need to bubble only 9999. A few more cases are as below.
A. Answer 1.54 then Bubble 0154
B. Answer 1.987 will be 0199 due to rounding off of the last digit.
C. Answer 0.1889 will be 0019 due to rounding off.
D. Answer 32.23 will be 3223.
6. No candidate is allowed to carry any textual material, printed or written, bits of papers, pager, mobile
phone, any electronic device, etc. except the BT ID Card inside the examination room/hall.
7. Rough work is to be done on the additional sheets provided
8. On completion of the test, the candidate must immediately hand over the Answer Sheet to the Invigilator
on duty in the Room/Hall. However, the candidates are allowed to take away this Test Booklet with them.
9. The CODE for this Booklet is A. Make sure that the CODE is marked properly on the answer sheet.
10. Do not fold or make any stray mark on the Answer Sheet.

BT-IIT |Camp| FC Rd| PaudRoad|Wanowrie|Aundh|VimanNagar|Deccan|SataraRd|Pimple Saudagar|PCMC|www.bakliwaltutorialsiit.com 1


BT Tests Bakliwal Tutorials - IIT
PHYSICS
SECTION – I
Single Correct Answer Type
This section contains 20 multiple choice questions. Each question has four choices (A), (B), (C) and
(D) out of which ONLY ONE is correct.

Q.1 A particle moves on x-axis according to the equation x  x0 sin 2  t , the motion is simple
harmonic
(A) with amplitude x0 (B)with amplitude 2x0

(C) with time period  2 /   (D) with time period  /  

Solution : (D)
= sin = (1 − cos 2 )= − cos 2

Frequency =2
⟹2 ⁄ ′=2
⟹ =( ⁄ )
Amplitude = /2
∴( )

Q.2 When two simple harmonic motions of same periods, same amplitude, having phase difference
of 3 / 2 , and at right angles to each other are super imposed, the resultant wave form is a
(A) circle (B) parabola (C) ellipse (D) figure of eight
Solution : (A)
The SHM 's are x  a sin t and y  a sin(t  3 / 2)  a cos t
 x 2  y2  a 2
 (A)
Q.3 The equation of a wave disturbance propagating in the positive x-direction is given by
2
y  1/ 1  x 2  at time t = 0 and by y  1/ 1   x  1  at t = 2 seconds, where x and y are in
 
metres. The shape of the wave disturbance does not change during the propagation. The velocity
of the wave is
-1 -1 -1 -1
(A) 1 ms (B) 0.5 ms (C) 1.5 ms (D) 2 ms
Solution : (B)

Writing the general expression for y in terms of x as


1 1
y 2
.At t  0, y  1 / (1  x 2 ). At t  2s, y 
1  (x  vt) 1  [x  v(2)]2

BT-IIT |Camp| FC Rd| PaudRoad|Wanowrie|Aundh|VimanNagar|Deccan|SataraRd|Pimple Saudagar|PCMC|www.bakliwaltutorialsiit.com 2


BT Tests Bakliwal Tutorials - IIT
Comparing with the given equation we get 2v = 1 and v = 0.5 m/s
a b
Q.4 The potential energy between two atoms in a diatomic molecule varies with x as U  12
 6,
x x
where a and b are positive constants. Find the equivalent spring constant for the oscillation of
one atom if the other atom is kept stationary.

1/3 1/3 1/3 1/3


9b 2  4b  9b 2  b  9b 2  2b  9b 2  b 
(A)   (B)   (C)   (D)  
2a  a  a  2a  a  a  a a
Solution : (B)

Q.5 A uniform string of length 20m is suspended from a rigid support. A short wave pulse is
introduced at its lowest end. It starts moving up the string. The time taken to reach the support is
: (take g  10ms 2 )

(A) 2s (B) 2 2s (C) 2s (D) 2 2s

BT-IIT |Camp| FC Rd| PaudRoad|Wanowrie|Aundh|VimanNagar|Deccan|SataraRd|Pimple Saudagar|PCMC|www.bakliwaltutorialsiit.com 3


BT Tests Bakliwal Tutorials - IIT
Solution : (D)

Q.6 A transverse sinusoidal wave moves along a string in the positive x-direction at a speed of 10
cm/s. The wavelength of the wave is 0.5 m and its amplitude is 10 cm. At a particular time t, the
snapshot of the wave is shown in figure. The velocity of point P when its displacement is 5 cm is

√ √ √ √
(A) ̂ / (B) - ̂ / (C) ̂ / (D) − ̂ /

Solution : (A)

Q.7 A pendulum has time period T in air. When it is made to oscillate in water, it acquired a time
period T’ = √2T. The specific gravity of the pendulum bob is equal to
(A) √2 (B) 2 (C) 2√2 (D) None of these
Solution : (B)

The effective acceleration of a bob in water = = 1− ,where d and D are the density of

water and the bob respectively.


=specific gravity of the bob.

Since the period of oscillation of the bob in air and water are given as

BT-IIT |Camp| FC Rd| PaudRoad|Wanowrie|Aundh|VimanNagar|Deccan|SataraRd|Pimple Saudagar|PCMC|www.bakliwaltutorialsiit.com 4


BT Tests Bakliwal Tutorials - IIT
ℓ ℓ
=2 and ′ = 2

( / )
∴ ⁄ ′= = = 1− = 1−

Putting ⁄ ′ = 1/√2, we obtain


1⁄2 = 1 − 1/

⇒ = ⇒ =2

∴( )

Q.8 A disc of radius R and mass M is pivoted at the rim and is set for small oscillations. If simple
pendulum has to have the same period as that of the disc, the length of the simple pendulum
should be
(A) (5/4) R (B) (2/3) R (C) (3/4) R (D) (3/2) R
Solution : (D)

Q.9 Two particles A and B each of mass ‘m’ are describing uniform circular motion of radius ‘R’
with angular velocity  in horizontal x – y plane. The particle ‘A’ is moving in anticlockwise
direction whereas particle B is moving in clockwise direction. If at t = 0 the particle A and
particle B are at the positions shown in figure, then for the centre of mass of the two particles (A
and B):

(A) motion occurs with constant acceleration


2
(B) motion is S.H.M. with time period

BT-IIT |Camp| FC Rd| PaudRoad|Wanowrie|Aundh|VimanNagar|Deccan|SataraRd|Pimple Saudagar|PCMC|www.bakliwaltutorialsiit.com 5
BT Tests Bakliwal Tutorials - IIT

(C) motion is S.H.M. with time period

(D) motion is only oscillatory and not S.H.M.
Solution : (B)

Q.10 An object of mass 0.8 kg is attached to one end of a spring and the system is set into simple
harmonic motion. The displacement x of the object as a function of time is shown in the figure.
With the aid of the data, the magnitude of the object’s acceleration at t = 1.0 is

2 2 2
(A) zero (B) 1.57 m/s (C) 0.197 m/s (D) 0.157 m/s
Solution : (C)

BT-IIT |Camp| FC Rd| PaudRoad|Wanowrie|Aundh|VimanNagar|Deccan|SataraRd|Pimple Saudagar|PCMC|www.bakliwaltutorialsiit.com 6


BT Tests Bakliwal Tutorials - IIT

Q.11 One end of a spring of force constant k is fixed to a vertical wall and the other to a block of mass
m resting on a smooth horizontal surface. There is another wall at a distance x0 from the block.
The spring is then compressed by 2x0 and released. The time taken to strike the wall is

1 k k 2π m π k
(A) π (B) (C) (D)
6 m m 3 k 4 m
Solution : (C)

Q.12 A transverse wave is described by the equation y  y0 sin 2  ft  x / a  . The maximum particle

velocity is equal to four times the wave velocity if a is equal to


(A) πy 0 /4 (B) πy 0 /2 (C) πy0 (D) 2πy0

Solution : (B)
The maximum paritcle velocity of SHM of amplitude and frequency f is 2 . The wave
velocity is
f. For 2fy0 to be equal to 4f,  has to be y 0 / 2
(Here   a)
BT-IIT |Camp| FC Rd| PaudRoad|Wanowrie|Aundh|VimanNagar|Deccan|SataraRd|Pimple Saudagar|PCMC|www.bakliwaltutorialsiit.com 7
BT Tests Bakliwal Tutorials - IIT

Q.13 A block of mass m rigidly attached with a sprig k is compressed through a distance A. If the
block is released, the period of oscillation of the block for a complete cycle is equal to

4π m π m 2π m
(A) (B) (C) (D) None of these
3 k 2 k 3 k
Solution : (A)

Q.14 Which of the following functions represent a travelling wave?


2 2
 x vt  1
(A)  x  vt  (B) ln  x  vt  (C) e (D)
x  vt
Solution : (C)
Although all the four functions are written in the form ( ± ), only (c) among the four
functions in finite everywhere at all times. Hence only (c) represents a travelling wave

Q.15 A uniform rod of length L and mass M is pivoted at the centre. Its two ends are attached to two
springs of equal spring constant k. The springs are fixed to rigid supports as shown in the figure,
and rod is free to oscillate in the horizontal plane. The rod is gently pushed through a small angle
in one direction and released. The frequency of oscillation is

BT-IIT |Camp| FC Rd| PaudRoad|Wanowrie|Aundh|VimanNagar|Deccan|SataraRd|Pimple Saudagar|PCMC|www.bakliwaltutorialsiit.com 8


BT Tests Bakliwal Tutorials - IIT

1 2k 1 k 1 6k 1 24k
(A) (B) (C) (D)
2 M 2 M 2 M 2 M
Solution: (C)

Q.16 A point mass is subjected to two simultaneous sinusoidal displacements in x-direction, ( )=


 2 
sin and x 2 (t)  A sin  t   . Adding a third sinusoidal displacement
 3 

x 3 (t)  B sin  t    brings the mass to a complete rest. The values of B and  are

3 4 5 
(A) 2A, (B) A, (C) 3A, (D) A,
4 3 6 3
Solution: (B)

Resultant amplitude of is A at angle from .

BT-IIT |Camp| FC Rd| PaudRoad|Wanowrie|Aundh|VimanNagar|Deccan|SataraRd|Pimple Saudagar|PCMC|www.bakliwaltutorialsiit.com 9


BT Tests Bakliwal Tutorials - IIT
To make resultant of , and to be zero. should be equal to A at angle = as

shown in figure.
∴ Correct answer is (B)

Q.17 A wave is represented by the equation;


y  Asin(10x 15t   / 3)
Where, x is in metre and t is in second. The expression represents
(A) a wave traveling in the positive x-direction with a velocity 1.5 m/s
(B) a wave travelling in the negative x-direction with a velocity 1.5 m/s
(C) a wave travelling in the negative x-direction with a wavelength 0.1 m
(D) a wave travelling in the positive x-direction with a wavelength 0.2 m
Solution: (B)

Q.18 One end of a long metallic wire of length Lis tied to the ceiling. The other end is tied to a
massless spring of spring constant k. A mass m hangs freely from the free end of the spring. The
area of cross-section and the Young’s modulus of the wire are A and Y respectively. If the mass
is slightly pulled down and released, it will oscillate with a time period

m(YA  kL)
(A) 2(m / k)1/2 (B) 2
YAk
(C) 2(mYA / kL)1/2 (D) 2(mL / YA)1/2
Solution: (B)

BT-IIT |Camp| FC Rd| PaudRoad|Wanowrie|Aundh|VimanNagar|Deccan|SataraRd|Pimple Saudagar|PCMC|www.bakliwaltutorialsiit.com 10


BT Tests Bakliwal Tutorials - IIT

Q.19 For a particle executing SHM the displacement x is given by = cos . Identify the graph
which represents the variation of potential energy (PE) as s function of time t and displacement
x.

(A) I, III (B) II, IV (C) II, III (D) I, IV


Solution: (A)

Q.20 A spring of stiffness constant k and natural length is cut into two parts of length 3 /4 and /4
respectively, and an arrangement is made as shown in the figure. If the mass is slightly displaced,
find the time period of oscillation.

BT-IIT |Camp| FC Rd| PaudRoad|Wanowrie|Aundh|VimanNagar|Deccan|SataraRd|Pimple Saudagar|PCMC|www.bakliwaltutorialsiit.com 11


BT Tests Bakliwal Tutorials - IIT
 3m  3m  6m  6m
(A) (B) (C) (D)
2 k 4 k 2 k 4 k
Solution: (A)

SECTION -II
(Numerical Type)
This section contains 5 questions.

Q.21 In the figure shown, mass of the plank is m and that of the solid cylinder is 8m. Springs are light.
The plank is slightly displaced from equilibrium and then released. Find the period of small
oscillations (in seconds) of the plank. There is no slipping at any contact point. The ratio of the
m 2
mass of the plank and stiffness of the spring i.e.,  .
K 2

Solution : (04.00)

BT-IIT |Camp| FC Rd| PaudRoad|Wanowrie|Aundh|VimanNagar|Deccan|SataraRd|Pimple Saudagar|PCMC|www.bakliwaltutorialsiit.com 12


BT Tests Bakliwal Tutorials - IIT

Q.22 Two blocks lie on each other and are connected to a spring as shown in Figure.

block A placed on block B of mass 6 kg so that the system’s period is 0.75 s. Assuming no
slipping, what should be the minimum value of coefficient of static friction  , for which block A

BT-IIT |Camp| FC Rd| PaudRoad|Wanowrie|Aundh|VimanNagar|Deccan|SataraRd|Pimple Saudagar|PCMC|www.bakliwaltutorialsiit.com 13


BT Tests Bakliwal Tutorials - IIT
will not slip relative to block B, if block B is displaced 50 mm from equilibrium position and
released?
Solution : (00.35 to 00.36)

Q.23 Consider a liquid which fills a uniform U-tube, as shown in the figure upto a height h. Find the
angular frequency of small oscillations of the liquid in the U-tube. (Consider h = 10 cm)

Solution : (10.00)

BT-IIT |Camp| FC Rd| PaudRoad|Wanowrie|Aundh|VimanNagar|Deccan|SataraRd|Pimple Saudagar|PCMC|www.bakliwaltutorialsiit.com 14


BT Tests Bakliwal Tutorials - IIT
Q.24 A hemispherical shell of mass M and radians R made to vibrate as shown in figure. Find its
frequency of oscillation. (Radius = 30 cm,   3.14 )

Solution : (0.79 – 0.80)

Q.25 A progressive wave on a string having linear mass density ρ is represented by

 2 
y  A sin  x   t  where y is in mm. Find the total energy  in μJ  passing through origin
  

from t = 0 to t  .  Take: ρ = 3 ×10-3 kg/m ; A = 1 mm ; ω = 100 rad/sec ; λ = 16 cm 
2 
Solution : (06.00)

CHEMISTRY
SECTION – I
Single Correct Answer Type
This section contains 20 multiple choice questions. Each question has four choices (A), (B), (C) and
(D) out of which ONLY ONE is correct.

BT-IIT |Camp| FC Rd| PaudRoad|Wanowrie|Aundh|VimanNagar|Deccan|SataraRd|Pimple Saudagar|PCMC|www.bakliwaltutorialsiit.com 15


BT Tests Bakliwal Tutorials - IIT
Q.1 Atoms A occupies, corners, face centres and atoms B occupies octahedral voids. Then the
structure is

The co-ordination number of A, B is


(A) 4, 6 (B) 6, 4 (C) 4, 4 (D) 6, 6
Solution : (D)
Q.2 The number of primary, secondary and tertiary amines possible with the molecular formula
C3 H9 N respectively.
(A) 1, 2, 2 (B) 1, 2, 1 (C) 2, 1, 1 (D) 3, 0, 1
Solution : (C)

Q.3

Shows which type of isomerism


(A)Functional group isomerism (B) Geometrical isomerism
(C) Metamerism (D) Position isomerism
Solution : (C)

Q.4 What is the correct IUPAC name of the following compound

BT-IIT |Camp| FC Rd| PaudRoad|Wanowrie|Aundh|VimanNagar|Deccan|SataraRd|Pimple Saudagar|PCMC|www.bakliwaltutorialsiit.com 16


BT Tests Bakliwal Tutorials - IIT

(A) 2E, 4E, 6Z 4-methyl oct – 2, 4, 6 triene (B) 2E, 4Z, 6Z 5-methyl oct – 2, 4, 6 triene
(C) 2Z, 4Z, 6Z 5-methyl oct – 2, 4, 6 triene (D) 2E, 4Z, 6E 4-methyl oct – 2, 4, 6 triene
Solution : (D)

Q.5 Which is amorphous solid?


(A) Rubber (B) Plastic (C) Glass (D) All
Solution : (D)
Amorphous solids neither have ordered arrangement (i.e. no definite shape) nor have sharp
melting point like crystals, but when heated, they become pliable until they assume the properties
usually related to liquid. Therefore, they are regarded as super cooled liquids.

Q.6 What is the simplest formula of a solid whose cubic unit cell has the atom A at each corner, the
atom B at each face centre and a C atom at the body centre?
(A) AB2C (B) A 2BC (C) AB3C (D) ABC3

Solution : (C)
An atom at the corner of a cube is shared among 8 unit cells. As there are 8 corners in a cube,
1
number of corner atom (A) per unit cell = 8 × = 1.
8

Q.7 Which of the following molecule or molecular ion has highest number of total electrons in
antibonding molecular orbitals
(A) O2 (B) N 2 (C) N 2  (D) O2 2

Solution : (D)

BT-IIT |Camp| FC Rd| PaudRoad|Wanowrie|Aundh|VimanNagar|Deccan|SataraRd|Pimple Saudagar|PCMC|www.bakliwaltutorialsiit.com 17


BT Tests Bakliwal Tutorials - IIT
Q.8 The species that do not contain peroxide ions are
(A) PbO2 (B) H 2O2 (C) SrO2 (D) BaO2

Solution : (A)

Q.9 One mole of a gas is heated at constant pressure to raise its temperature by 1° C . The work done
in Joules is
(A) -4.3 (B) -8.314 (C) -16.62 (D) Unpredictable
Solution : (B)

Q.10 Gases X, Y, Z, P and Q have the van der Waal’s constants ‘a’ and ‘b’ (in CGS units) as shown
below
X Y Z P Q
a 6 6 20 0.05 30
b 0.025 0.15 0.1 0.02 0.2
The gas with the highest critical temperature is :
(A) P (B) Q (C) Y (D) X
Solution : (D)

Q.11 Which of the following statements is INCORRECT :


 rNa + = 95 pm ; rCl- = 181pm

(A) The coordination number of each type of ion in CsCl is 8.


(B) A metal that crystallizes in BCC structure has a coordination number 12.
(C) A unit cell of an ionic crystal shares some of its ions with other unit cells
(D) The length of the unit cell in NaCl is 552 pm.
Solution : (B)

BT-IIT |Camp| FC Rd| PaudRoad|Wanowrie|Aundh|VimanNagar|Deccan|SataraRd|Pimple Saudagar|PCMC|www.bakliwaltutorialsiit.com 18


BT Tests Bakliwal Tutorials - IIT
Q.12 Analysis shows that nickel oxide has the formula Ni0.98O . What fractions of the nickel exist as

Ni 2+ and Ni 3+ ?
(A) 96%, 4% (B) 84%, 16% (C) 36%, 64% (D) 4%, 96%
Solution : (A)

Q.13 The IP1 , IP2 , IP3 , IP4 and IP5 of an element are 7.1, 14.3, 34.5, 46.8, 162.2 eV respectively. The

element is likely to be
(A) Na (B) Si (C) F (D) Ca
Solution : (B)

Q.14 Which of the following have zero dipole moment?


(A) benzene 1, 4- diol (B) trans-1, 2-dichloro ethene
(C) cis-1,2-dichloro ethene (D) 1,1-dichloro ethene
Solution : (B)

Q.15 The temperature of a definite amount of an ideal monoatomic gas becomes four times in a
reversible process for which heat exchange is zero. Which of the following is correct relation
between the final and initial parameters of gas?
1
(A) Vf = 8Vi (B) Pf = 32Pi (C) Vf = 16Vi (D) Pf = Pi
16
Solution : (B)
1
 V1   T2  γ-1 3/2
  =   = 3 = 8
 V2   T1 

BT-IIT |Camp| FC Rd| PaudRoad|Wanowrie|Aundh|VimanNagar|Deccan|SataraRd|Pimple Saudagar|PCMC|www.bakliwaltutorialsiit.com 19


BT Tests Bakliwal Tutorials - IIT
γ
P1 T  1-γ 5/3
-5/2 1
=  2 =  3 -2/3 =  3 =
P2  T1  32

Q.16 Which of the following pair will not produce dihydrogen gas-
(A) Cu + HCl  dil. (B) Fe + H2SO4 (C) Mg + steam (D) Na + alcohol

Solution : (A)
The less active metals which lie below hydrogen in electrochemical series can’t displaced
hydrogen gas from dilute mineral acid.

Q.17 Calculate the weight of CaO required to remove the hardness of 106 L of water containing 1.62 g
of Ca  HCO3 2 in 1.0 L.

(A) 56 Kg (B) 5.6 ×105 g/ml (C) 104 g (D) 1.62g


Solution : (B)

Q.18 Which among the following reactions will be favored at low pressure?
(A) N 2  g  + O2  g   2NO  g  (B) H 2  g  + I 2  g   2HI  g 

(C) PCl5  g   PCl3  g  + Cl2  g  (D) N 2  g  + 3H 2  g   2NH3  g 

Solution : (C)

Q.19 The unit cell of a metallic element of atomic mass 108 and density 10.5 g /cm3 is a cube with
edge length of 409 pm. The structure of the crystal lattice is
(A) FCC (B) BCC (C) HCP (D) None of these
Solution : (A)

BT-IIT |Camp| FC Rd| PaudRoad|Wanowrie|Aundh|VimanNagar|Deccan|SataraRd|Pimple Saudagar|PCMC|www.bakliwaltutorialsiit.com 20


BT Tests Bakliwal Tutorials - IIT

Q.20 An aqueous solution contains 10% ammonia by mass and has a density of 0.99 g/cm3 . What is


the pH of this solution? Ka for NH+4 = 5.0 ×10-10 M 
(A) 12.0 (B) 9.3 (C) 9.0 (D) 2.0
Solution : (A)

SECTION -II
(Numerical Type)
This section contains 5 questions.

Q.21 x = no of compounds which can show to geometrical isomerism.

BT-IIT |Camp| FC Rd| PaudRoad|Wanowrie|Aundh|VimanNagar|Deccan|SataraRd|Pimple Saudagar|PCMC|www.bakliwaltutorialsiit.com 21


BT Tests Bakliwal Tutorials - IIT

OH
C N
N N

CMe C C CMe

Then x is

Solution : (08.00)

Q.22 The structure of diamond and silicon is same and can be described as atoms taking corners and
faces of a cube and also taking positions of alternate tetrahedral voids. What is the value of
 density  diamond  
 
 density  silicon  

At. Mass (g / mol) Covalent Bond Length (Ǻ)


Carbon : 12 C – C = 1.50
Silicon : 28 Si – Si = 2.25

BT-IIT |Camp| FC Rd| PaudRoad|Wanowrie|Aundh|VimanNagar|Deccan|SataraRd|Pimple Saudagar|PCMC|www.bakliwaltutorialsiit.com 22


BT Tests Bakliwal Tutorials - IIT
Solution : (01.45)

Q.23 How many pair(s) of geometrical isomers are possible with C6 H12 (only in open chain structures)

Solution : (04.00)

Q.24 Total number of geometrical isomer of following compound is :

Solution : (02.00)

Q.25 What would be the percent hydrolysis of 0.10 (M) N 2 H 5Cl , a salt containing the acid ion

conjugate to hydrazine base? Given : Kb for N2 H4 = 9.6 ×10-7

BT-IIT |Camp| FC Rd| PaudRoad|Wanowrie|Aundh|VimanNagar|Deccan|SataraRd|Pimple Saudagar|PCMC|www.bakliwaltutorialsiit.com 23


BT Tests Bakliwal Tutorials - IIT
Solution : (00.03)

MATH
SECTION – I
Single Correct Answer Type
This section contains 20 multiple choice questions. Each question has four choices (A), (B), (C) and
(D) out of which ONLY ONE is correct.

Q.1 If z1  8  2i z2  2  2i z3  8  6i , where i = √−1 forms a triangle then orthocenter of the

triangle is
(A) 8  2i (B) 8  6i (C) 2  2i (D)  2  2i
Solution (A)

Q.2 Given family of straight lines a  3 x  4 y  6   b  x  y  2   0 , a, b  R . The line of family,

situated at greatest distance from point P  2, 3  has equation

(A) 4 x  3 y  8  0 (B) 5 x  3 y  10  0 (C) 15 x  8 y  30  0 (D) 2 x  4 y  5  0


Solution (A)

BT-IIT |Camp| FC Rd| PaudRoad|Wanowrie|Aundh|VimanNagar|Deccan|SataraRd|Pimple Saudagar|PCMC|www.bakliwaltutorialsiit.com 24


BT Tests Bakliwal Tutorials - IIT

Q.3 If tangents are drawn from any point of the part of the line x  y  3 which lies outside the circle
x 2  y 2  9 , then the line of chord of contact always passes through point
(A) (3, 5) (B) (3, 3) (C) (5, 3) (D) (5, - 3)
Solution (B)

BT-IIT |Camp| FC Rd| PaudRoad|Wanowrie|Aundh|VimanNagar|Deccan|SataraRd|Pimple Saudagar|PCMC|www.bakliwaltutorialsiit.com 25


BT Tests Bakliwal Tutorials - IIT
Q.4 The Equation of chord of minimum length passing through (1, 2) of circle
x 2  y 2  4 x  2 y  4  0 is ax  by  c  0 (c  0) then a  b  c
(A) 8 (B)  8 (C) 1 (D)  1
Solution (C)

Q.5 20  10 C0   17  10 C1   14  10 C2   _ _ _ _ _ 7  10 C9   10 

(A) 10.29  1 (B) 9.210 (C) 10.210 (D) 5.210


Solution (D)

39
20
Q.6 If 1  3x  2 x 2   a0  a1 x  a2 x 2     a40 x 40 then find a  k
k 1

(A) -1 (B) 1 (C) 0 (D) 2 20

Solution (A)

BT-IIT |Camp| FC Rd| PaudRoad|Wanowrie|Aundh|VimanNagar|Deccan|SataraRd|Pimple Saudagar|PCMC|www.bakliwaltutorialsiit.com 26


BT Tests Bakliwal Tutorials - IIT

11
Q.7 Coefficient of x 4 in expansion 1  x 2  1  5 x  9 x 2       41x10 

(A) 11 C2  4. 11C1  3 (B) 167

(C) 154 (D) 171


Solution (D)

1 0 3
Q.8 Q.8 If A    such that A  ()A  ()I  O Find    = ? (,   R)
1 2 
(A) – 1 (B) 1 (C) 13 (D) – 13
Solution: (C)

BT-IIT |Camp| FC Rd| PaudRoad|Wanowrie|Aundh|VimanNagar|Deccan|SataraRd|Pimple Saudagar|PCMC|www.bakliwaltutorialsiit.com 27


BT Tests Bakliwal Tutorials - IIT

 0 4 1 
Q.9 Matrix V   4 0 5  is
 1 5 0 
(A) Orthogonal (B) Idempotent (C) Skew symmetric (D) Symmetric
Solution: (C)

7x  6y  2z  0
Q.10 If 3x  4y  2z  0 system of linear equations are given then pick the correct option:
x  2y  6z  0
(A) No solution exists (B) Only trivial solution exists
(C) Infinite non trivial solutions for x  2z (D) Infinite non trivial solutions for y  2z
Solution: (C)

BT-IIT |Camp| FC Rd| PaudRoad|Wanowrie|Aundh|VimanNagar|Deccan|SataraRd|Pimple Saudagar|PCMC|www.bakliwaltutorialsiit.com 28


BT Tests Bakliwal Tutorials - IIT

 1 1 1 
adjB
Q.11 If A   0 2 3 and B = (adj A) and C = 5A, then find value of
C
 2 1 0 
(A) 1 (B) 5 (C) 25 (D) 125
Solution: (A)

Q.12 If A and B are two matrixes such that, AB = O then, which of the following statements are
always true ?
(A) Either A = O or B = O
(B) A = 0 and B = O, both are compulsorily true
(C) Not necessary that either A = O or B = O
(D) A  O and B  O , both are compulsorily true
Solution: (C)

 
Q.13 If in a  ABC , a  2cms , B  , C  then side c = ?
6 4
8 7
(A) (B) (C) 2  2 3 (D) 2 3  2
7 8
Solution: (D)

BT-IIT |Camp| FC Rd| PaudRoad|Wanowrie|Aundh|VimanNagar|Deccan|SataraRd|Pimple Saudagar|PCMC|www.bakliwaltutorialsiit.com 29


BT Tests Bakliwal Tutorials - IIT

Q.14 If in  ABC , sin A : sin B : sin C  7 : 8 : 9 then cos A : cos B : cos C equals
(A) 11 : 6 : 14 (B) 6 : 11 : 14 (C) 28 : 22 : 10 (D) 14 : 11 : 6
Solution: (D)

Area of Circumcircle of polygon 4


Q.15 Recognize the regular polygon if 
Area of Incircle of polygon 3
(A) Pentagon (B) Hexagon (C) Heptagon (D) Octagon
Solution: (B)

BT-IIT |Camp| FC Rd| PaudRoad|Wanowrie|Aundh|VimanNagar|Deccan|SataraRd|Pimple Saudagar|PCMC|www.bakliwaltutorialsiit.com 30


BT Tests Bakliwal Tutorials - IIT

Q.16 If z is a complex number, z  2 , and a  max  z  4  and b  min  z  4  find a  b 


(A) 6 (B) 4 (C) 8 (D) 10
Solution: (C)

z 1  i
Q.17 If is purely imaginary then z lies on: (z is a complex number)
z  i 1
(A) Straight line (B) Line segment (C) Circle (D) Ray
Solution: (C)

BT-IIT |Camp| FC Rd| PaudRoad|Wanowrie|Aundh|VimanNagar|Deccan|SataraRd|Pimple Saudagar|PCMC|www.bakliwaltutorialsiit.com 31


BT Tests Bakliwal Tutorials - IIT

Q.18 Given the equation x 9  x 5  x 4  1  0 , x  C then find the number of distinct complex roots
whose real part is non-negative
(A) 9 (B) 7 (C) 5 (D) 3
Solution: (C)

Q.19 If area of triangle formed by z, − z, z − z is 18 unit 2 where i = √−1. Find absolute value of z.
(z is a complex number)
(A) 8 (B) 4 (C) 10 (D) 6
Solution: (D)

BT-IIT |Camp| FC Rd| PaudRoad|Wanowrie|Aundh|VimanNagar|Deccan|SataraRd|Pimple Saudagar|PCMC|www.bakliwaltutorialsiit.com 32


BT Tests Bakliwal Tutorials - IIT

3 
Q.20 The number of point(s) of intersection of curves arg (z  3i)  and arg (2z  1  2i)  is
4 4
(A) 0 (B) 1 (C) 2 (D) Infinite
Solution: (A)

BT-IIT |Camp| FC Rd| PaudRoad|Wanowrie|Aundh|VimanNagar|Deccan|SataraRd|Pimple Saudagar|PCMC|www.bakliwaltutorialsiit.com 33


BT Tests Bakliwal Tutorials - IIT

SECTION -II
(Numerical Type)
This section contains 5 questions.

100 100 201


Q.21 If a n is a positive term of a GP such that   a2n 1   200 ,   a2n   100 then its  an  1000  k  k
n 1 n 1 n 2

=?
Solution (00.30)

BT-IIT |Camp| FC Rd| PaudRoad|Wanowrie|Aundh|VimanNagar|Deccan|SataraRd|Pimple Saudagar|PCMC|www.bakliwaltutorialsiit.com 34


BT Tests Bakliwal Tutorials - IIT

Q.22 If  ,   C are distinct roots of equation x 2  x  1  0 then find 2 152  2 104 , where

represents absolute value.


Solution (02.00)

Q.23 The total number of solution of equation tan x  sec x  2 cos x lying in  0, 4  is/are

Solution (04.00)

BT-IIT |Camp| FC Rd| PaudRoad|Wanowrie|Aundh|VimanNagar|Deccan|SataraRd|Pimple Saudagar|PCMC|www.bakliwaltutorialsiit.com 35


BT Tests Bakliwal Tutorials - IIT

n!
Q.24 If n Cr  and n Cr 1  10, nCr  45, nCr 1  120 then r equals
r!(n  r)!
Solution (02.00)

3x
Q.25      
If equation  2    1 x 2  2 2  4 x  3 2  9  3  0 is an identity in  , then
2
=

Solution (04.50)

BT-IIT |Camp| FC Rd| PaudRoad|Wanowrie|Aundh|VimanNagar|Deccan|SataraRd|Pimple Saudagar|PCMC|www.bakliwaltutorialsiit.com 36

You might also like